TRANSFORMER
!
Working Principle of
Transformer
Transformer Construction
Core-type Transformers
Shell-type Transformers
E.M.F. Equation of
Transformer
Voltage Transformation
Ratio
Transformer with losses
Equivalent Resistance
Magnetic Leakage
Transformer with Resistance
and Leakage Reactance
Total Approximate Voltage
Drop in Transformer
Exact Voltage Drop
Separation of Core Losses
Short-Circuit or Impedance
Test
Why Transformer Rating in
KVA?
Regulation of a Transformer
Percentage Resistance,
Reactance and Impedance
Kapp Regulation Diagram
Sumpner or Back-to-back-
Test
Efficiency of a Transformer
Auto-transformer
+0)26-4
Learning Objectives
To overcome losses, the electricity from a
generator is passed through a step-up
transformer, which increases the voltage.
Throughout the distribution system, the voltages
are changed using step-down transformers to
voltages suitable to the applications at industry
and homes.
Ç
CONTENTS
CONTENTS
CONTENTS
CONTENTS
1116
Electrical Technology
Fig. 32.1
f
Lam
inated Core
Prim
ary
Secondary
32.1. Working Principle of a Transformer
A transformer is a static (or stationary) piece of apparatus by means of which electric power in
one circuit is transformed into electric power of the same frequency in another circuit. It can raise or
lower the voltage in a circuit but with a correspond-
ing decrease or increase in current. The physical
basis of a transformer is mutual induction between
two circuits linked by a common magnetic flux. In
its simplest form, it consists of two inductive coils
which are electrically separated but magnetically
linked through a path of low reluctance as shown
in Fig. 32.1. The two coils possess high mutual
inductance. If one coil is connected to a source of
alternating voltage, an alternating flux is set up in
the laminated core, most of which is linked with
the other coil in which it produces mutually-in-
duced e.m.f. (according to Faraday’s Laws of Electromagnetic Induction e = MdI/dt). If the second
coil circuit is closed, a current flows in it and so electric energy is transferred (entirely magnetically)
from the first coil to the second coil. The first coil, in which electric energy is fed from the a.c. supply
mains, is called primary winding and the other from which energy is drawn out, is called secondary
winding. In brief, a transformer is a device that
1. transfers electric power from one circuit to another
2. it does so without a change of frequency
3. it accomplishes this by electromagnetic induction and
4. where the two electric circuits are in mutual inductive influence of each other.
32.2. Transformer Construction
The simple elements of a transformer
consist of two coils having mutual
inductance and a laminated steel core. The
two coils are insulated from each other and
the steel core. Other necessary parts are :
some suitable container for assembled core
and windings ; a suitable medium for
insulating the core and its windings from
its container ; suitable bushings (either of
porcelain, oil-filled or capacitor-type) for
insulating and bringing out the terminals
of windings
from the
tank.
In all
types of
transformers,
the core is constructed of transformer sheet steel laminations assembled
to provide a continuous magnetic path with a minimum of air-gap
included. The steel used is of high silicon content, sometimes heat
treated to produce a high permeability and a low hysteresis loss at the
iron core
secondary
coil
secondary
coil
110/120
volts
220/240
volts
110/120
volts
primary
coil
220/240
volts
Principle of transformer
Fig. 32.2
Transformer
1117
usual operating flux densities. The eddy current loss is minimised by
laminating the core, the laminations being insulated from each other by a
light coat of core-plate varnish or by an oxide layer on the surface. The
thickness of laminations varies from 0.35 mm for a frequency of 50 Hz to
0.5 mm for a frequency of 25 Hz. The core laminations (in the form of
strips) are joined as shown in Fig. 32.2. It is seen that the joints in the
alternate layers are staggered in order to avoid the presence of narrow
gaps right through the cross-section of the core. Such staggered joints
are said to be ‘imbricated’.
Constructionally, the transformers are of two
general types, distinguished from each other
merely by the manner in which the primary
and secondary coils are placed around the
laminated core. The two types are known as (i) core-type and (ii) shell-
type. Another recent development is spiral-core or wound-core type, the
trade name being spirakore transformer.
In the so-called core type transformers, the windings surround a
considerable part of the core whereas in shell-type transformers, the core
surrounds a considerable portion of the windings as shown schematically
in Fig. 32.3 (a) and (b) respectively.
Fig. 32.3 Fig. 32.4
In the simplified diagram for the core type transformers [Fig. 32.3 (a)], the primary and secondary
winding are shown located on the opposite legs (or limbs) of the core, but in actual construction,
these are always interleaved to reduce leakage flux. As shown in Fig. 32.4, half the primary and half the
secondary winding have been placed side by side or concentrically on each limb, not primary on one
limb (or leg) and the secondary on the other.
Lam
ination
Coils
Butt
Joint
Fig. 32.5 Fig. 32.6
In both core and shell-type transformers, the individual laminations are cut in the form of long
strips of L’s, Es and Is as shown in Fig. 32.5. The assembly of the complete core for the two types of
transformers is shown in Fig. 32.6 and Fig. 32.7.
Low
voltage
High
voltage
Normal Operation
Oil
Iron Core
Magnetic Flux
Ground
Core-type transformer
Leads
Primary
Winding
Secondary Winding
Paper Insulation
Laminated
core
Shell-Type transformer
1118
Electrical Technology
As said above, in order to avoid high reluctance at the joints where the laminations are butted against
each other, the alternate layers are stacked differently to eliminate these joints as shown in Fig. 32.6 and
Fig. 32.7.
Butt Joint
Fig. 32.7
32.3.32.3.
32.3.32.3.
32.3.
CorCor
CorCor
Cor
e-type e-type
e-type e-type
e-type
TT
TT
T
ransfransf
ransfransf
ransf
oror
oror
or
mermer
mermer
mer
ss
ss
s
The coils used are form-wound and
are of the cylindrical type. The general
form of these coils may be circular or oval
or rectangular. In small size core-type
transformers, a simple rectangular core is
used with cylindrical coils which are either
circular or rectangular in form. But for
large-size core-type transformers, round
or circular cylindrical coils are used which are so
wound as to fit over a cruciform core section as
shown in Fig. 32.8(a). The circular cylindrical coils
are used in most of the core-type transformers
because of their mechanical strength. Such
cylindrical coils are wound in helical layers with the
different layers insulated from each other by paper,
cloth, micarta board or cooling ducts. Fig. 32.8(c)
shows the general arrangement of these coils with
respect to the core. Insulating cylinders of fuller
board are used to separate the cylindrical windings
from the core and from each other. Since the low-
voltage (LV) winding is easiest to insulate, it is placed
nearest to the core (Fig. 32.8).
Core
L.V.H.V.
H.V.
L.V
. L.V.
H.V
.
()b
()c
Insulating
Cylinder
H.V. Winding
L.V. Winding
Fig. 32.8
Coil Coil
Coil
Coil Coil
Single-Phase Transformer Cores
2-leg core 3-leg core 4-leg core
Fig. 32.8 (a)
Transformer
1119
Because of laminations and insulation, the net or effective core area is reduced, due allowance for
which has to be made (Ex. 32.6). It is found that, in general, the reduction in core sectional area due to the
presence of paper, surface oxide etc. is of the order of 10% approximately.
As pointed out above, rectangular cores with rectangular cylindrical coils can be used for small-size
core-type transformers as shown in Fig. 32.9 (a) but for large-sized transformers, it becomes wasteful to
use rectangular cylindrical coils and so circular cylindrical coils are preferred. For such purposes, square
cores may be used as shown in Fig. 32.9 (b) where circles represent the tubular former carrying the coils.
Obviously, a considerable amount of useful space is still wasted. A common improvement on square core
is to employ cruciform core as in Fig. 32.9 (c) which demands, at least, two sizes of core strips. For very
large transformers, further core-stepping is done as in Fig. 32.9 (d) where at least three sizes of core plates
are necessary. Core-stepping not only gives high space factor but also results in reduced length of the mean
turn and the consequent I
2
R loss. Three stepped core is the one most commonly used although more steps
may be used for very large transformers as in Fig. 32.9 (e). From the geometry of Fig. 32.9, it can be shown
that maximum gross core section for Fig. 32.9 (b) is 0.5 d
2
and for Fig. 32.9 (c) it is 0.616 d
2
where d is the
diameter of the cylindrical coil.
Fig. 32.9
32.4. Shell-type Transformers
In these case also, the coils are form-would but are multi-layer disc type usually wound in the
form of pancakes. The different layers of such multi-layer discs are insulated from each other by
paper. The complete winding consists of stacked discs with insulation space between the coils–the
spaces forming horizontal cooling and insulating ducts. A shell-type transformer may have a simple
rectangular form as shown in Fig. 32.10 or it may have distributed form as shown in Fig. 32.11.
Fig. 32.10
A very commonly-used shell-type transformer is the one known as Berry Transformer–so called
after the name of its designer and is cylindrical in form. The transformer core consists of laminations
arranged in groups which radiate out from the centre as shown in section in Fig. 32.12.
1120
Electrical Technology
It may be pointed out that cores and coils of transformers must be provided with rigid mechanical
bracing in order to prevent movement and possible insulation damage. Good bracing reduces vibration and
the objectionable noise–a humming sound–during operation.
The spiral-core transformer employs the newest development in core construction. The core is as-
sembled of a continuous strip or ribbon of transformer steel wound in the form of a circular or elliptical
cylinder. Such construction allows the core flux to follow the grain of the iron. Cold-rolled steel of high
silicon content enables the designer to use considerably higher operating flux densities with lower loss per
kg. The use of higher flux density reduces the weight per kVA. Hence, the advantages of such construction
are (i) a relatively more rigid core (ii) lesser weight and size per kVA rating (iii) lower iron losses at higher
operating flux densities and (iv) lower cost of manufacture.
()b
()a
Cylindrical
Winding
M
agnetic
Core
Fig. 32.11 Fig. 32.12
Transformers are generally housed in tightly-fitted sheet-metal ; tanks filled with special insulating oil*.
This oil has been highly developed and its function is two-fold. By circulation, it not only keeps the coils
reasonably cool, but also provides the transformer with additional insulation not obtainable when the
transformer is left in the air.
In cases where a smooth tank surface does not provide sufficient cooling area, the sides of the tank are
corrugated or provided with radiators mounted on the sides. Good transformer oil should be absolutely free
from alkalies, sulphur and particularly from moisture. The presence of even an
extremely small percentage of moisture in the oil is highly detrimental from the insulation viewpoint
because it lowers the dielectric strength of the oil considerably. The importance of avoiding moisture
in the transformer oil is clear from the fact that even an addition of 8 parts of water in 1,000,000 reduces
the insulating quality of the oil to a value generally recognized as below standard. Hence, the tanks
are sealed air-tight in smaller units. In the case of large-sized transformers where complete air-tight
construction is impossible, chambers known as breathers are provided to permit the oil inside the tank
to expand and contract as its temperature increases or decreases. The atmospheric moisture is
entrapped in these breathers and is not allowed to pass on to the oil. Another thing to avoid in the oil
is sledging which is simply the decomposition of oil with long and continued use. Sledging is caused
principally by exposure to oxygen during heating and results in the formation of large deposits of dark
and heavy matter that eventually clogs the cooling ducts in the transformer.
No other feature in the construction of a transformer is given more attention and care than the
insulating materials, because the life on the unit almost solely depends on the quality, durability and
handling of these materials. All the insulating materials are selected on the basis of their high quality
and ability to preserve high quality even after many years of normal use.
* Instead of natural mineral oil, now-a-days synthetic insulating fluids known as ASKARELS (trade name) are
used. They are non-inflammable and, under the influence of an electric arc, do not decompose to produce
inflammable gases. One such fluid commercially known as PYROCLOR is being extensively used because
it possesses remarkable stability as a dielectric and even after long service shows no deterioration through
sledging, oxidation, acid or moisture formation. Unlike mineral oil, it shows no rapid burning.
Transformer
1121
All the transformer leads are brought out of their cases through suitable bushings. There are many
designs of these, their size and construction depending on the voltage of the leads. For moderate voltages,
porcelain bushings are used to insulate the leads as they come out through the tank. In general, they look
almost like the insulators used on the transmission lines. In high voltage installations, oil-filled or capacitor-
type bushings are employed.
The choice of core or shell-type construction is usually determined by cost, because similar character-
istics can be obtained with both types. For very high-voltage transformers or for multiwinding design, shell-
type construction is preferred by many manufacturers. In this type, usually the mean length of coil turn is
longer than in a comparable core-type design. Both core and shell forms are used and the selection is
decided by many factors such as voltage rating, kVA rating, weight, insulation stress, heat distribution etc.
Another means of classifying the transformers is according to the type of cooling employed. The
following types are in common use :
(a) oil-filled self-cooled (b) oil-filled water-cooled (c) air-blast type
Small and medium size distribution transformers–so called because of their use on distribution
systems as distinguished from line transmission–are of type (a). The assembled windings and cores
of such transformers are mounted in a welded, oil-tight steel tank provided with steel cover. After
putting the core at its proper place, the tank is filled with purified, high quality insulating oil. The oil
serves to convey the heat from the core and the windings to the case from where it is radiated out to
the surroundings. For small size, the tanks are usually smooth-surfaced, but for larger sizes, the cases
are frequently corrugated or fluted to get greater heat radiation area without increasing the cubical
capacity of the tank. Still larger sizes are provided with radiators or pipes.
Construction of very large self-cooled transformers is expensive, a more economical form of
construction for such large transformers is provided in the oil-immersed, water-cooled type. As
before, the windings and the core are immersed in the oil, but there is mounted near the surface of oil,
a cooling coil through which cold water is kept circulating. The heat is carried away by this water. The
largest transformers such as those used with high-voltage transmission lines, are constructed in this
manner.
Oil-filled transformers are built for outdoor duty and as these require no housing other than their
own, a great saving is thereby effected. These transformers require only periodic inspection.
For voltages below 25,000 V, transformers can be built for cooling by means of an air-blast. The
transformer is not immersed in oil, but is housed in a thin sheet-metal box open at both ends through
which air is blown from the bottom to the top by means of a fan or blower.
32.5. Elementary Theory of an Ideal Transformer
An ideal transformer is one which has no losses i.e. its windings have no ohmic resistance, there is no
magnetic leakage and hence which has no I
2
R and core losses. In other words, an ideal transformer consists
of two purely inductive coils wound on a loss-free core. It may, however, be noted that it is impossible
to realize such a transformer in practice, yet for convenience, we will start with such a trans-
former and step by step approach an actual transformer.
Fig. 32.13
1122
Electrical Technology
Consider an ideal transformer [Fig. 32.13 (a)] whose secondary is open and whose primary is con-
nected to sinusoidal alternating voltage V
1
. This potential difference causes an alternating current to flow in
the primary. Since the primary coil is purely inductive and there is no output (secondary being open) the
primary draws the magnetising current I
µ
only. The function of this current is
merely to magnetise the core, it is small in magnitude and lags V
1
by 90°. This alternating current I
µ
produces an alternating flux φ which is, at all
times, proportional to the current (assuming
permeability of the magnetic circuit to be
constant) and, hence, is in phase with it. This
changing flux is linked both with the primary
and the secondary windings. Therefore, it
produces self-induced e.m.f. in the primary.
This self-induced e.m.f. E
1
is, at every in-
stant, equal to and in opposition to V
1
. It is
also known as counter e.m.f. or back e.m.f.
of the primary.
Similarly, there is produced in the sec-
ondary an induced e.m.f. E
2
which is
known as mutually induced e.m.f. This
e.m.f. is antiphase with V
1
and its magni-
tude is proportional to the rate of change
of flux and the number of secondary turns.
The instantaneous values of applied
voltage, induced e.m.fs, flux and
magnetising current are shown by sinu-
soidal waves in Fig. 32.13 (b). Fig. 32.13
(c) shows the vectorial representation of the effective values of the above quantities.
32.6. E.M.F. Equation of a Transformer
Let N
1
= No. of turns in primary
N
2
= No. of turns in secondary
Φ
m
= Maximum flux in core in webers
= B
m
× A
f = Frequency of a.c. input in Hz
As shown in Fig. 32.14, flux increases from its zero value to
maximum value Φ
m
in one quarter of the cycle i.e. in 1/4 f second.
Average rate of change of flux =
1/4
m
f
Φ
=4 f Φ
m
Wb/s or volt
Now, rate of change of flux per turn means induced e.m.f. in volts.
Average e.m.f./turn = 4 f Φ
m
volt
If flux Φ varies sinusoidally, then r.m.s. value of induced e.m.f. is obtained by multiplying the average
value with form factor.
Form factor =
r.m.s. value
=1.11
average value
r.m.s. value of e.m.f./turn = 1.11 × 4 f Φ
m
= 4.44 f Φ
m
volt
Now, r.m.s. value of the induced e.m.f. in the whole of primary winding
= (induced e.m.f/turn) × No. of primary turns
E
1
=4.44 f N
1
Φ
m
= 4.44 f N
1
B
m
A ...(i)
Fig. 32.14
Cycle
Tim
e
m
T
4
f
T
=
1
/
f
Step-up transformer
Primary coil
Secondary coil
If the primary coil
has 3 loops and the
secondary coil has
30, the voltage is
stepped up 10
times.
Primary coil
Secondary coil
If the primary coil
has 30 loops and
the secondary coil
has 3, the voltage
is stepped down 10
times.
Step-down transformer
Step-up transformer
Transformer
1123
Similarly, r.m.s. value of the e.m.f. induced in secondary is,
E
2
= 4.44 f N
2
Φ
m
= 4.44 f N
2
B
m
A ...(ii)
It is seen from (i) and (ii) that E
1
/N
1
= E
2
/N
2
= 4.44 f Φ
m
. It means that e.m.f./turn is the same in both
the primary and secondary windings.
In an ideal transformer on no-load, V
1
= E
1
and E
2
= V
2
where V
2
is the terminal voltage
(Fig. 32.15).
32.7 Voltage Transformation Ratio (K)
From equations (i) and (ii), we get
2
1
E
E
=
2
1
N
K
N
=
This constant K is known as voltage transformation
ratio.
(i) If N
2
> N
1
i.e. K > 1, then transformer is called step-up
transformer.
(ii) If N
2
< N
1
i.e. K < 1, then transformer is known as
step-down transformer.
Again, for an ideal transformer, input VA = output VA .
V
1
I
1
= V
2
I
2
or
21
12
1
IV
IVK
==
Hence, currents are in the inverse ratio of the (voltage) transformation ratio.
Example 32.1. The maximum flux density in the core of a 250/3000-volts, 50-Hz single-phase
transformer is 1.2 Wb/m
2
. If the e.m.f. per turn is 8 volt, determine
(i) primary and secondary turns (ii) area of the core.
(Electrical Engg.-I, Nagpur Univ. 1991)
Solution. (i) E
1
= N
1
× e.m.f. induced/turn
N
1
= 250/8 = 32; N
2
= 3000/8 = 375
(ii) We may use E
2
= 4.44 f N
2
B
m
A
3000 = 4.44 × 50 × 375 × 1.2 × A; A = 0.03m
2
.
Example 32.2. The core of a 100-kVA, 11000/550 V, 50-Hz, 1-ph, core type transformer has a
cross-section of 20 cm × 20 cm. Find (i) the number of H.V. and L.V. turns per phase and (ii) the e.m.f.
per turn if the maximum core density is not to exceed 1.3 Tesla. Assume a stacking factor of 0.9.
What will happen if its primary voltage is increased by 10% on no-load ?
(Elect. Machines, A.M.I.E. Sec. B, 1991)
Solution. (i) B
m
=1.3 T, A = (0.2 × 0.2) × 0.9 = 0.036 m
2
11,000 = 4.44 × 50 × N
1
× 1.3 × 0.036, N
1
= 1060
550 = 4.44 × 50 × N
2
× 1.3 × 0.036; N
2
= 53
or, N
2
= KN
1
= (550/11,000) × 1060 = 53
(ii) e.m.f./turn = 11,000/1060 = 10.4 V or 550/53 = 10.4 V
Keeping supply frequency constant, if primary voltage is increased by 10%, magnetising current will
increase by much more than 10%. However, due to saturation, flux density will increase only marginally and
so will the eddy current and hysteresis losses.
Example 32.3. A single-phase transformer has 400 primary and 1000 secondary turns. The
net cross-sectional area of the core is 60 cm
2
. If the primary winding be connected to a 50-Hz supply
at 520 V, calculate (i) the peak value of flux density in the core (ii) the voltage induced in the
secondary winding. (Elect. Engg-I, Pune Univ. 1989)
V
1
E
1
E
2
V
2
Fig. 32.15
1124
Electrical Technology
Solution. K = N
2
/N
1
= 1000/400 = 2.5
(i) E
2
/E
1
= K
Ε
2
=
ΚΕ
1
= 2.5 × 520 = 1300 V
(ii) E
1
= 4.44 f N
1
B
m
A
or 520 = 4.44 × 50 × 400 × B
m
× (60 × 10
4
) B
m
= 0.976 Wb/m
2
Example 32.4. A 25-kVA transformer has 500 turns on the primary and 50 turns on the second-
ary winding. The primary is connected to 3000-V, 50-Hz supply. Find the full-load primary and
secondary currents, the secondary e.m.f. and the maximum flux in the core. Neglect leakage drops
and no-load primary current. (Elect. & Electronic Engg., Madras Univ. 1985)
Solution. K = N
2
/N
1
= 50/500 = 1/10
Now, full-load I
1
= 25,000/3000 = 8.33 A. F.L. I
2
= I
1
/K = 10 × 8.33 = 83.3 A
e.m.f. per turn on primary side = 3000/500 = 6 V
secondary e.m.f. = 6 × 50 = 300 V (or E
2
= KE
1
= 3000 × 1/10 =300 V)
Also, E
1
=4.44 f N
1
Φ
m
; 3000 = 4.44 × 50 × 500 × Φ
m
∴ Φ
m
= 27 mWb
Example 32.5. The core of a three phase, 50 Hz, 11000/550 V delta/star, 300 kVA, core-type
transformer operates with a flux of 0.05 Wb. Find
(i) number of H.V. and L.V. turns per phase. (ii) e.m.f. per turn
(iii) full load H.V. and L.V. phase-currents. (Bharathithasan Univ. April 1997)
Solution. Maximum value of flux has been given as 0.05 Wb.
(ii) e.m.f. per turn = 4.44 f φ
m
=4.44 × 50 × 0.05 = 11.1 volts
(i) Calculations for number of turns on two sides :
Voltage per phase on delta-connected primary winding = 11000 volts
Voltage per phase on star-connected secondary winding = 550/1.732 = 317.5 volts
T
1
= number of turns on primary, per phase
= voltage per phase/e.m.f. per turn
= 11000/11.1 = 991
T
2
= number of turns on secondary, per phase
= voltage per phase/e.m.f. per turn
= 317.5/11.1 = 28.6
Note : (i) Generally, Low-voltage-turns are calculated first, the figure is rounded off to next higher even
integer. In this case, it will be 30. Then, number of turns on primary side is calculated by turns-ratio.
In this case, T
1
= T
2
(V
1
/V
2
) = 30 × 11000/317.5 = 1040
This, however, reduces the flux and results into less saturation. This, in fact, is an elementary
aspect in Design-calculations for transformers. (Explanation is added here only to overcome a doubt
whether a fraction is acceptable as a number of L.V. turns).
(ii) Full load H.V. and L.V. phase currents :
Output per phase = (300/3) = 100 kVA
H.V. phase-current =
100 1000
9.1 Amp
11,000
×
=
L.V. phase-current = (100 × 1000/317.5) = 315 Amp
Example 32.6. A single phase transformer has 500 turns in the primary and 1200 turns in the
secondary. The cross-sectional area of the core is 80 sq. cm. If the primary winding is connected to
a 50 Hz supply at 500 V, calculate (i) Peak flux-density, and (ii) Voltage induced in the secondary.
(Bharathiar University November 1997)
Transformer
1125
Solution. From the e.m.f. equation for transformer,
500 = 4.44 × 50 × φ
m
× 500
φ
m
= 1/222 Wb
(i) Peak flux density, B
m
= φ
m
/ (80 × 10
4
) = 0.563 wb/m
2
(ii) Voltage induced in secondary is obtained from transformation ratio or turns ratio
2
1
V
V
=
2
1
N
N
or V
2
= 500 × 1200/500 = 1200 volts
Example 32.7. A 25 kVA, single-phase transformer has 250 turns on the primary and 40 turns on
the secondary winding. The primary is connected to 1500-volt, 50 Hz mains. Calculate
(i) Primary and Secondary currents on full-load, (ii) Secondary e.m.f., (iii) maximum flux in the core.
(Bharathiar Univ. April 1998)
Solution. (i) If V
2
= Secondary voltage rating, = secondary e.m.f.,
2
1500
V
=
40
250
, giving V
2
= 240 volts
(ii) Primary current = 25000/1500 = 16.67 amp
Secondary current = 25000/240 = 104.2 amp
(iii) If φ
m
is the maximum core-flux in Wb,
1500 = 4.44 × 50 × φ
m
× 250, giving φ
m
= 0.027 Wb or 27 mWb
Example 32.8. A single-phase, 50 Hz, core-type transformer has square cores of 20 cm side.
Permissible maximum flux-density is 1 Wb/m
2
. Calculate the number of turns per Limb on the High
and Low-voltage sides for a 3000/220 V ratio. (Manonmaniam Sundaranar Univ. April 1998)
Solution. E.M.F. equation gives the number of turns required on the two sides. We shall first
calculate the L.V.-turns, round the figure off to the next higher even number, so that given maximum
flux density is not exceeded. With the corrected number of L.V. turns, calculate H.V.-turns by
transformation ratio. Further, there are two Limbs. Each Limb accommodates half-L.V. and half H.V.
winding from the view-point of reducing leakage reactance.
Starting with calculation for L.V. turns, T
2
,
4.44 × 50 × [(20 × 20 × 10
4
) × 1] ×
Τ
2
= 220
T
2
= 220/8.88 = 24.77
Select T
2
=26
T
1
/T
2
= V
1
/V
2
T
1
= 26 × 3000/220 = 354, selecting the nearest even integer.
Number of H.V. turns on each Limb = 177
Number of L.V. turns on each Limb = 13
32.8. Transformer with Losses but no Magnetic Leakage
We will consider two cases (i) when such a transformer is on no load and (ii) when it is loaded.
32.9. Transformer on No-load
In the above discussion, we assumed an ideal transformer i.e. one in which there were no core losses
and copper losses. But practical conditions require that certain modifications be made in the foregoing
1126
Electrical Technology
theory. When an actual transformer is put on load, there is iron loss in the core and copper loss in the
windings (both primary and secondary) and these losses are not entirely negligible.
Even when the transformer is on no-load, the primary input current is not wholly reactive. The primary
input current under no-load conditions has to supply (i) iron losses in the core i.e. hysteresis loss and eddy
current loss and (ii) a very small amount of copper loss in primary (there being no Cu loss in secondary as
it is open). Hence, the no-load primary input current I
0
is not at 90° behind V
1
but lags it by an angle φ
0
<
90°. No-load input power
W
0
= V
1
I
0
cos φ
0
where cos φ
0
is primary power factor under no-load conditions. No-load
condition of an actual transformer is shown vectorially in Fig. 32.16.
As seen from Fig. 32.16, primary current I
0
has two components :
(i) One in phase with V
1
. This is known as active or working or
iron loss component I
w
because it mainly supplies the iron loss plus
small quantity of primary Cu loss.
I
w
= I
0
cos φ
0
(ii) The other component is in quadrature with V
1
and is known as
magnetising component I
µ
because its function is to sustain the
alternating flux in the core. It is wattless.
I
µ
= I
0
sin φ
0
Obviously, I
0
is the vector sum of I
w
and I
µ
, hence I
0
= (I
µ
2
+ I
ω
2
).
The following points should be noted carefully :
1. The no-load primary current I
0
is very small as compared to the full-load primary current. It is
about 1 per cent of the full-load current.
2. Owing to the fact that the permeability of the core varies with the instantaneous value of the
exciting current, the wave of the exciting or magnetising current is not truly sinusoidal. As such it
should not be represented by a vector because only sinusoidally varying quantities are represented
by rotating vectors. But, in practice, it makes no appreciable difference.
3. As I
0
is very small, the no-load primary Cu loss is negligibly small which means that no-load
primary input is practically equal to the iron loss in the transformer.
4. As it is principally the core-loss which is responsible for shift in the current vector, angle φ
0
is
known as hysteresis angle of advance.
Example 32.9. (a) A 2,200/200-V transformer draws a no-load primary current of 0.6 A and
absorbs 400 watts. Find the magnetising and iron loss currents.
(b) A 2,200/250-V transformer takes 0.5 A at a p.f. of 0.3 on open circuit. Find magnetising
and working components of no-load primary current.
Solution. (a) Iron-loss current
=
no-load input in watts
400
primary voltage 2,200
==
0.182 A
Now I
0
2
= I
w
2
+ I
µ
2
Magnetising component I
µ
=
22
(0.6 0.182)
−=
0.572 A
The two components are shown in Fig. 29.15.
Fig. 32.16
V
1
i
0
I
w
E
1
E
2
0
I
Transformer
1127
(b) I
0
= 0.5 A, cos φ
0
= 0.3 I
w
= I
0
cos φ
0
= 0.5 × 0.3 = 0.15 A
I
µ
=
22
0.5 0.15 0.476 A
−=
Example 32.10. A single-phase transformer has 500 turns on the primary and 40 turns on the
secondary winding. The mean length of the magnetic path in the iron core is 150 cm and the joints
are equivalent to an air-gap of 0.1 mm. When a p.d. of 3,000 V is applied to the primary, maximum
flux density is 1.2 Wb/m
2
. Calculate (a) the cross-sectional area of the core (b) no-load secondary
voltage (c) the no-load current drawn by the primary (d) power factor on no-load. Given that
AT/cm for a flux density of 1.2 Wb/m
2
in iron to be 5, the corresponding iron loss to be 2 watt/kg at
50 Hz and the density of iron as 7.8 gram/cm
3
.
Solution. (a) 3,000 = 4.44 × 50 × 500 × 1.2 × A A = 0.0225 m
2
= 225 cm
2
This is the net cross-sectional area. However, the gross area would be about 10% more to allow
for the insulation between laminations.
(b) K = N
2
/N
1
= 40/500 = 4/50
N.L. secondary voltage = KE
1
= (4/50) × 3000 = 240 V
(c) AT per cm = 5 AT for iron core = 150 × 5 = 750
AT for air-gap =
7
0
1.2
0.0001 95.5
410
B
Hl l
= × =
µ
π×
Total AT for given B
max
= 750 + 95.5 = 845.5
Max. value of magnetising current drawn by primary = 845.5/500 = 1.691 A
Assuming this current to be sinusoidal, its r.m.s. value is I
µ
=
1.691/ 2 1.196 A
=
Volume of iron = length × area = 150 × 225 = 33,750 cm
3
Density = 7.8 gram/cm
3
Mass of iron = 33,750 × 7.8/1000 = 263.25 kg
Total iron loss = 263.25 × 2 = 526.5 W
Iron loss component of no-load primary current I
0
is I
w
= 526.5/3000 = 0.176 A
I
0
=
22 2 2
1.196 0.176
uw
II
+= + =
0.208 A
(d) Power factor,cos φ
0
= I
w
/I
0
= 0.176/1.208 = 0.1457
Example 32.11. A single-phase transformer has 1000 turns on the primary and 200 turns on the
secondary. The no load current is 3 amp. at a p.f. of 0.2 lagging. Calculate the primary current and
power-factor when the secondary current is 280 Amp at a p.f. of 0.80 lagging.
(Nagpur University, November 1997)
Solution. V
2
is taken as reference. cos
1
0.80 = 36.87°
I
2
= 280 ∠−36.87º amp
I’
2
= (280/5) ∠−36.87º amp
φ = cos
1
0.20 = 78.5°, sin φ = 0.98
I
1
= I
0
+ I’
2
= 3(0.20 j 0.98) + 56 (0.80 j 0.60)
=0.6 j 2.94 + 44.8 j 33.6
= 45.4 j 2.94 + 44.8 j 33.6
= 45.4 j 36.54 = 58.3 38.86º
Thus I lags behind the supply voltage by an angle of 38.86°.
1128
Electrical Technology
Tutorial Problems 32.1
1. The number of turns on the primary and secondary windings of a 1−φ transformer are 350 and 35
respectively. If the primary is connected to a 2.2 kV, 50-Hz supply, determine the secondary voltage
on no-load. [220 V] (Elect. Engg.-II, Kerala Univ. 1980)
2. A 3000/200-V, 50-Hz, 1-phase transformer is built on a core having an effective cross-sectional area
of 150 cm
2
and has 80 turns in the low-voltage winding. Calculate
(a) the value of the maximum flux density in the core
(b) the number of turns in the high-voltage winding. [(a) 0.75 Wb/m
2
(b) 1200]
3. A 3,300/230-V, 50-Hz, 1-phase transformer is to be worked at a maximum flux density of 1.2 Wb/m
2
in the core. The effective cross-sectional area of the transformer core is 150 cm
2
. Calculate suitable
values of primary and secondary turns. [830; 58]
4. A 40-kVA, 3,300/240-V, 50 Hz, 1-phase transformer has 660 turns on the primary. Determine
(a) the number of turns on the secondary
(b) the maximum value of flux in the core
(c) the approximate value of primary and secondary full-load currents.
Internal drops in the windings are to be ignored. [(a) 48 (b) 22.5 mWb (c) 12.1 A; 166.7 A]
5. A double-wound, 1-phase transformer is required to step down from 1900 V to 240 V, 50-Hz. It is
to have 1.5 V per turn. Calculate the required number of turns on the primary and secondary windings
respectively.
The peak value of flux density is required to be not more than 1.2 Wb/m
2
. Calculate the required
cross-sectional area of the steel core. If the output is 10 kVA, calculate the secondary current.
[1,267; 160; 56.4 cm
2
; 41.75 A]
6. The no-load voltage ratio in a 1-phase, 50-Hz, core-type transformer is 1,200/440. Find the number
of turns in each winding if the maximum flux is to be 0.075 Wb. [24 and 74 turns]
7. A 1-phase transformer has 500 primary and 1200 secondary turns. The net cross-sectional area of
the core is 75 cm
2
. If the primary winding be connected to a 400-V, 50 Hz supply, calculate.
(i) the peak value of flux density in the core and (ii) voltage induced in the secondary winding.
[0.48 Wb/m
2
; 60 V]
8. A 10-kVA, 1-phase transformer has a turn ratio of 300/23. The primary is connected to a 1500-V,
60 Hz supply. Find the secondary volts on open-circuit and the approximate values of the currents
in the two windings on full-load. Find also the maximum value of the flux. [115 V; 6.67 A; 87 A;
11.75 mWb]
9. A 100-kVA, 3300/400-V, 50 Hz, 1 phase transformer has 110 turns on the secondary. Calculate the
approximate values of the primary and secondary full-load currents, the maximum value of flux in the
core and the number of primary turns.
How does the core flux vary with load ? [30.3 A; 250 A; 16.4 mWb; 907]
10. The no-load current of a transformer is 5.0 A at 0.3 power factor when supplied at 230-V, 50-Hz. The
number of turns on the primary winding is 200. Calculate (i) the maximum value of flux in the core (ii)
the core loss (iii) the magnetising current. [5.18 mWb; 345 W; 4.77 A]
11. The no-load current of a transformer is 15 at a power factor of 0.2 when connected to a 460-V, 50-Hz
supply. If the primary winding has 550 turns, calculate
(a) the magnetising component of no-load current
(b) the iron loss
(c) the maximum value of the flux in the core. [(a) 14.7 A (b) 1,380 W (c) 3.77 mWb]
12. The no-load current of a transformer is 4.0 A at 0.25 p.f. when supplied at 250-V, 50 Hz. The number
of turns on the primary winding is 200. Calculate
(i) the r.m.s. value of the flux in the core (assume sinusoidal flux)
(ii) the core loss
(iii) the magnetising current. [(i) 3.96 mWb (ii) 250 W (iii) 3.87 A]
Transformer
1129
13. The following data apply to a single- phase transformer:
output : 100 kVA, secondary voltage; 400 V; Primary turns: 200; secondary turns: 40; Neglecting the
losses, calculate: (i) the primary applied voltage (ii) the normal primary and secondary currents (iii)
the secondary current, when the load is 25 kW at 0.8 power factor.
(Rajiv Gandhi Technical University, Bhopal 2000) [(i) 2000 V, (ii) 50 amp, (iii) 78.125 amp]
32.10. Transformer on Load
When the secondary is loaded, the secondary current I
2
is set up. The
magnitude and phase of I
2
with respect to V
2
is determined by the charac-
teristics of the load. Current I
2
is in phase with V
2
if load is non-inductive, it
lags if load is inductive and it leads if load is capacitive.
The secondary current sets up its own m.m.f. (=N
2
I
2
) and hence its
own flux Φ
2
which is in opposition to the main primary flux Φ which is
due to I
0
. The secondary ampere-turns N
2
I
2
are known as
demagnetising amp-turns. The opposing secondary flux Φ
2
weakens
the primary flux Φ momentarily, hence primary back e.m.f. E
1
tends to be
reduced. For a moment V
1
gains the upper hand over E
1
and hence
causes more current to flow in primary.
Let the additional primary current be I
2
. It is known as load
component of primary current. This current is antiphase with I
2
. The
additional primary m.m.f. N
1
I
2
sets up its own flux Φ
2
which is in
opposition to Φ
2
(but is in the same direction as Φ) and is equal to it in
magnitude. Hence, the two cancel each other out. So, we find that the
magnetic effects of secondary current I
2
are immediately neutralized by
the additional primary current I
2
which is brought into existence exactly
at the same instant as I
2
. The whole process is illustrated in Fig. 32.17.
Hence, whatever the load conditions, the net flux passing through
the core is approximately the same as at no-load. An important deduction is that due to the constancy
of core flux at all loads, the core loss is also practically the same under all load conditions.
As Φ
2
= Φ
2
′ ∴ N
2
I
2
= N
1
I
2
′∴I
2
=
2
22
1
N
IKI
N
×=
Hence, when transformer is on load, the primary winding has two currents in it; one is I
0
and the other
is I
2
which is anti-phase with I
2
and K times in magnitude. The total primary current is the vector sum
of I
0
and I
2
.
0
0
I
2
I
0
I
1
I
2
E
2
k=1
()a
()b
2
1
0
I'
2
I
0
V
1
=
I
1
I
2
E
2
k=1
()c
Negligible
0
1
2
0
I'
2
I
0
I
1
I
2
E
2
k=1
V
V
1
Fig. 32.18
Fig. 32.17
2
2
V
1
V
1
V
1
I
2
I
2
V
1
V
2
I
0
I
1
I
2
I
0
I
2
I
0
Load
Load
Load
1130
Electrical Technology
In Fig. 32.18 are shown the vector diagrams for a load transformer when load is non-inductive and
when it is inductive (a similar diagram could be drawn for capacitive load). Voltage transformation
ratio of unity is assumed so that primary vectors are equal to the secondary vectors. With reference
to Fig. 32.18 (a), I
2
is secondary current in phase with E
2
(strictly speaking it should be V
2
). It causes
primary current I
2
which is anti-phase with it and equal to it in magnitude (K = 1). Total primary
current I
1
is the vector sum of I
0
and I
2
and lags behind V
1
by an angle φ
1
.
In Fig. 32.18 (b) vectors are drawn for an inductive load. Here I
2
lags E
2
(actually V
2
) by φ
2
.
Current I
2
is again antiphase with I
2
and equal to it in magnitude. As before, I
1
is the vector sum of I
2
and I
0
and lags behind V
1
by φ
1
.
It will be observed that φ
1
is slightly greater than φ
2
. But if we neglect I
0
as compared to I
2
as in
Fig. 32.18 (c), then φ
1
= φ
2
. Moreover, under this assumption
N
1
I
2
= N
2
I
1
= N
1
I
2
21 2
22 1
IIN
K
IIN
== =
It shows that under full-load conditions, the ratio of primary and secondary currents is constant.
This important relationship is made the basis of current transformer–a transformer which is used with
a low-range ammeter for measuring currents in circuits where the direct connection of the ammeter is
impracticable.
Example 32.12. A single-phase transformer with a ratio of 440/110-V takes a no-load current
of 5A at 0.2 power factor lagging. If the secondary supplies a current of 120 A at a p.f. of 0.8 lagging,
estimate the current taken by the primary. (Elect. Engg. Punjab Univ. 1991)
Solution. cos φ
2
=0.8, φ
2
= cos
1
(0.8) = 36°54
cos φ
0
=0.2 ∴φ
0
= cos
1
(0.2) = 78° 30
Now K = V
2
/V
1
= 110/440 = 1/4
I
2
= KI
2
= 120 × 1/4 = 30 A
I
0
=5 A.
Angle between I
0
and I
2
= 78° 30′ − 36° 54= 41° 36
Using parallelogram law of vectors (Fig. 32.19) we get
I
1
=
22
(5 30 2 5 30 cos 41 36 )
++××× °
= 34.45 A
The resultant current could also have been found by resolving
I
2
and I
0
into their X and Y -components.
Example 32.13. A transformer has a primary winding of
800 turns and a secondary winding of 200 turns. When the load
current on the secondary is 80 A at 0.8 power factor lagging,
the primary current is 25 A at 0.707 power factor lagging.
Determine graphically or otherwise the no-load current of the
transformer and its phase with respect to the voltage.
Solution. Here K = 200/800 = 1/4; I
2
= 80 × 1/4 = 20 A
φ
2
= cos
1
(0.8) = 36.9°; φ
1
= cos
1
(0.707) = 45°
As seen from Fig. 32.20, I
1
is the vector sum of I
0
and I
2
′. Let
I
0
lag behind V
1
by an angle φ
0
.
I
0
cos φ
0
+ 20 cos 36.9° = 25 cos 45°
Fig. 32.19
36 54'
o
36 54'
o
78 30'
o
I'
2
=
30A
I
1
V
1
I
0
I
2
=
120A
Fig. 32.20
36.9
o
36.9
o
45
o
I
2
=
80A
I'
2
=
20A
V
1
I
1
E
2
I
0
0
Transformer
1131
I
0
cos φ
2
= 25 × 0.707 20 × 0.8
= 1.675 A
I
0
sin φ
0
+ 20 sin 36.9° = 25 sin 4
I
0
sin φ
0
= 25 × 0.707 20 × 0.6
= 5.675 A
tan φ
0
= 5.675/1.675 = 3.388
∴φ
0
= 73.3°
Now, I
0
sin φ
0
= 5.675
I
0
= 5.675/sin 73.3° = 5.93 A
Example 32.14. A single phase transformer takes 10 A on no load at p.f. of 0.2 lagging. The
turns ratio is 4 : 1 (step down). If the load on the secondary is 200 A at a p.f. of 0.85 lagging. Find
the primary current and power factor.
Neglect the voltage-drop in the winding. (Nagpur University November 1999)
Solution. Secondary load of 200 A, 0.85 lag is reflected as 50 A, 0.85 lag in terms of the primary
equivalent current.
I
0
= 10 ∠−φ
0
, where φ
0
= cos
1
0.20 = 78.5° lagging
=2 j 9.8 amp
I
2
= 50 ∠−φ
L
where φ
L
= cos
1
0.85 = 31.8°, lagging
I
2
= 42.5 j 26.35
Hence primary current I
1
= I
0
+ I
2
=2 j 9.8 + 42.5 j 26.35
= 44.5 j 36.15
|I
1
| = 57.333 amp, φ = 0.776 Lag.
φ =
1
44.5
cos 39.10 lagging
57.333
The phasor diagram is shown in Fig. 32.21.
Tutorial Problems 32.2
1. The primary of a certain transformer takes 1 A at a power factor of 0.4 when it is connected across
a 200-V, 50-Hz supply and the secondary is on open circuit. The number of turns on the primary is
twice that on the secondary. A load taking 50 A at a lagging power factor of 0.8 is now connected
across the secondary. What is now the value of primary current ? [25.9 A]
2. The number of turns on the primary and secondary windings of a single-phase transformer are 350
and 38 respectively. If the primary winding is connected to a 2.2 kV, 50-Hz supply, determine
(a) the secondary voltage on no-load,
(b) the primary current when the secondary current is 200 A at 0.8 p.f. lagging, if the no-load current
is 5 A at 0.2 p.f. lagging,
(c) the power factor of the primary current. [239 V; 25-65 A; 0.715 lag]
3. A 400/200-V, 1-phase transformer is supplying a load of 25 A at a p.f. of 0.866 lagging. On no-load
the current and power factor are 2 A and 0.208 respectively. Calculate the current taken from the
supply. [13.9 A lagging V1 by 36.1°]
4. A transformer takes 10 A on no-load at a power factor of 0.1. The turn ratio is 4 : 1 (step down). If
I
O
I
2
I
1
O
L
V
(Ref)
Fig. 32.21
1132
Electrical Technology
a load is supplied by the secondary at 200 A and p.f. of 0.8, find the primary current and power factor
(internal voltage drops in transformer are to be ignored). [57.2 A; 0.717 lagging]
5. A 1-phase transformer is supplied at 1,600 V on the h.v. side and has a turn ratio of 8 : 1. The
transformer supplies a load of 20 kW at a power factor of 0.8 lag and takes a magnetising current of
2.0 A at a power factor of 0.2. Calculate the magnitude and phase of the current taken from the h.v.
supply. [17.15 A ; 0.753 lag] (Elect. Engg. Calcutta Univ. 1980)
6. A 2,200/200-V, transformer takes 1 A at the H.T. side on no-load at a p.f. of 0.385 lagging. Calculate
the iron losses.
If a load of 50 A at a power of 0.8 lagging is taken from the secondary of the transformer, calculate the
actual primary current and its power factor. [847 W; 5.44 A; 0.74 lag]
7. A 400/200-V, I-phase transformer is supplying a load of 50 A at a power factor of 0.866 lagging. The
no-load current is 2 A at 0.208 p.f. lagging. Calculate the primary current and primary power factor.
[26.4 A; 0.838 lag] (Elect. Machines-I, Indore Univ. 1980)
32.11. Transformer with Winding Resistance but No Magnetic Leakage
An ideal transformer was supposed to possess no resistance, but in an actual transformer, there
is always present some resistance of the primary and secondary windings. Due to this resistance,
there is some voltage drop in the two windings. The result is that :
(i) The secondary terminal voltage V
2
is vectorially less than the secondary induced e.m.f. E
2
by an amount I
2
R
2
where R
2
is the resistance of the secondary winding. Hence, V
2
is equal to the
vector difference of E
2
and resistive voltage drop I
2
R
2
.
V
2
= E
2
I
2
R
2
...vector difference
(ii) Similarly, primary induced e.m.f. E
1
is equal to the vector difference of V
1
and I
1
R
1
where R
1
is the resistance of the primary winding.
E
1
= V
1
I
1
R
1
...vector difference
I' KI
2
2
=
I
1
E
1
E
2
V
2
I
2
I
0
V
1
IR
11
IR
22
()a
I'
2
I
1
E
2
I
2
V
2
I
0
V
1
IR
11
IR
22
()c
_
E
1
2
I' KI
2
2
=
I
1
E
2
V
2
I
2
I
0
V
1
IR
11
IR
22
()b
_
E
1
2
1
Fig. 32.22
The vector diagrams for non-inductive, inductive and capacitive loads are shown in Fig. 32.22 (a),
(b) and (c) respectively.
32.12. Equivalent Resistance
In Fig. 32.23 a transformer is shown whose primary and secondary windings have resistances of
R
1
and R
2
respectively. The resistances have been shown external to the windings.
Transformer
1133
It would now be shown that the resistances of the
two windings can be transferred to any one of the two
windings. The advantage of concentrating both the
resistances in one winding is that it makes calculations
very simple and easy because one has then to work in
one winding only. It will be proved that a resistance of
R
2
in secondary is equivalent to R
2
/K
2
in primary. The
value R
2
/K
2
will be denoted by R
2
′− the equivalent
secondary resistance as referred to primary.
The copper loss in secondary is I
2
2
R
2
. This loss is supplied by primary which takes a current of I
1
.
Hence if R
2
is the equivalent resistance in primary which would have caused the same loss as R
2
in
secondary, then
I
1
2
R
2
= I
2
2
R
2
or R
2
= (I
2
/I
1
)
2
R
2
Now, if we neglect no-load current I
0
, then I
2
/I
1
= I/K
*
. Hence, R
2
= R
2
/K
2
Similarly, equivalent primary resistance as referred to secondary is R
1
= K
2
R
1
In Fig. 32.24, secondary resistance has been transferred to primary side leaving secondary circuit
resistanceless. The resistance R
1
+ R
2
= R
1
+ R
2
/K
2
is known as the equivalent or effective resistance
of the transformer as referred to primary and may be designated as R
01
.
R
01
= R
1
+ R
2
= R
1
+ R
2
/ K
2
Similarly, the equivalent resistance of the transformer as referred to secondary is
R
02
= R
2
+ R
1
= R
2
+ K
2
R
1
.
This fact is shown in Fig. 32.25 where all the resistances of the transformer has been concentrated
in the secondary winding.
R
2
R
02
K
R
2
1
R'
1
=
R
1
R
01
R
2
K
2
R'
2
R
2
=
Fig. 32.24 Fig. 32.25
It is to be noted that
1. a resistance of R
1
in primary is equivalent to K
2
R
1
in secondary. Hence, it is called equivalent
resistance as referred to secondary i.e. R
1
.
2. a resistance of R
2
in secondary is equivalent to R
2
/K
2
in primary. Hence, it is called the
equivalent secondary resistance as referred to primary i.e. R
2
.
3. Total or effective resistance of the transformer as referred to primary is
R
01
= primary resistance + equivalent secondary resistance as referred to primary
= R
1
+ R
2
= R
1
+ R
2
/K
2
4. Similarly, total transformer resistance as referred to secondary is,
R
02
= secondary resistance + equivalent primary resistance as referred to secondary
= R
2
+ R
1
= R
2
+ K
2
R
1
Fig. 32.23
R
1
I
1
I
2
R
2
* Actually I
2
# 2/I
2
= I/K and not I
2
# 2/I
1
. However, if I
0
is neglected, then I
2
= I
1
.
1134
Electrical Technology
Note : It is important to remember that
(a) when shifting any primary resistance to the secondary, multiply it by K
2
i.e. (transformation ratio)
2
.
(b) when shifting secondary resistance to the primary, divide it by K
2
.
(c) however, when shifting any voltage from one winding to another only K is used.
32.13. Magnetic Leakage
In the preceding discussion, it has been assumed that
all the flux linked with primary winding also links the
secondary winding. But, in practice, it is impossible to
realize this condition. It is found, however, that all the
flux linked with primary does not link the secondary but
part of it i.e. Φ
L
1
completes its magnetic circuit by passing
through air rather than around the core, as shown in Fig.
32.26. This leakage flux is produced when the m.m.f. due
to primary ampere-turns existing between points a and
b, acts along the leakage paths. Hence, this flux is known as primary leakage flux and is proportional
to the primary ampere-turns alone because the secondary turns do not link the magnetic circuit of Φ
L
1
.
The flux Φ
L
1
is in time phase with I
1
. It induces an e.m.f. e
L
1
in primary but not in secondary.
Similarly, secondary ampere-turns (or m.m.f.) acting across points c and d set up leakage flux Φ
L
2
which is linked with secondary winding alone (and not with primary turns). This flux Φ
L
2
is in time
phase with I
2
and produces a self-induced e.m.f. e
L
2
in secondary (but not in primary).
At no load and light loads, the primary and secondary ampere-turns are small, hence leakage fluxes
are negligible. But when load is increased, both primary and secondary windings carry huge currents.
Hence, large m.m.f.s are set up which, while acting on leakage paths, increase the leakage flux.
As said earlier, the leakage flux linking with each winding, produces a self-induced e.m.f. in that
winding. Hence, in effect, it is equivalent to a small
choker or inductive coil in series with each winding
such that voltage drop in each series coil is equal to
that produced by leakage flux. In other words, a
transformer with magnetic leakage is equivalent to
an ideal transformer with inductive coils connected
in both primary and secondary circuits as shown in
Fig. 32.27 such that the internal e.m.f. in each inductive
coil is equal to that due to the corresponding leakage
flux in the actual transformer.
X
1
= e
L1
/I
1
and X
2
= e
L2
/I
2
The terms X
1
and X
2
are known as primary and secondary leakage reactances respectively.
Following few points should be kept in mind :
1. The leakage flux links one or the other winding but not both, hence it in no way contributes
to the transfer of energy from the primary to the secondary winding.
2. The primary voltage V
1
will have to supply reactive drop I
1
X
1
in addition to I
1
R
1
. Similarly E
2
will have to supply I
2
R
2
and I
2
X
2
.
3. In an actual transformer, the primary and secondary windings are not placed on separate legs
or limbs as shown in Fig. 32.27 because due to their being widely separated, large primary and
secondary leakage fluxes would result. These leakage fluxes are minimised by sectionalizing and
interleaving the primary and secondary windings as in Fig. 32.6 or Fig. 32.8.
Fig. 32.26
Load
V
1
a
b
L
1
c
d
l
2
Fig. 32.27
Transformer
1135
32.14. Transformer with Resistance and Leakage Reactance
In Fig. 32.28 the primary and secondary windings of a transformer with reactances taken out of
the windings are shown. The primary impedance is given by
Z
1
=
22
11
()
RX
+
Similarly, secondary impedance is
given by
Z
2
=
22
22
()
RX+
The resistance and leakage reactance of
each winding is responsible for some voltage
drop in each winding. In primary, the leakage
reactance drop is I
1
X
1
(usually 1 or 2% of V
1
).
Hence
V
1
= E
1
+ I
1
(R
1
+ jX
1
) = E
1
+ I
1
Z
1
Similarly, there are I
2
R
2
and I
2
X
2
drops in secondary which combine with V
2
to give E
2
.
E
2
= V
2
+ I
2
(R
2
+ jX
2
) = V
2
+ I
2
Z
2
The vector diagram for such a transformer for different kinds of loads is shown in Fig. 32.29. In
these diagrams, vectors for resistive drops are drawn parallel to current vectors whereas reactive
drops are perpendicular to the current vectors. The angle φ
1
between V
1
and I
1
gives the power factor
angle of the transformer.
It may be noted that leakage reactances can also be transferred from one winding to the other in
the same way as resistance.
X
2
= X
2
/K
2
and X
1
= K
2
X
1
and X
01
= X
1
+ X
2
= X
1
+ X
2
/K
2
and X
02
= X
2
+ X
1
= X
2
+ K
2
X
1
V
2
I
1
I
2
V
1
E
1
I
0
I'
2
E
2
IX
11
IR
11
IZ
11
IR
22
IX
22
IZ
22
V
2
I
1
I
2
V
1
E
1
I
0
I'
2
E
2
IX
11
IR
11
IZ
11
IR
22
IX
22
IZ
22
V
2
I
1
I
2
V
1
E
1
I
0
I'
2
E
2
IX
11
IR
11
IZ
11
IR
22
IX
22
IZ
22
Fig. 32.29
Fig. 32.28
X
1
R
1
X
2
R
2
Z
2
Z
1
1136
Electrical Technology
R+R'
12
X+X'
12
R
01
Z
01
X
01
R+R'
21
X+X'
21
R
02
Z
02
X
02
Fig. 32.30 (a) Fig. 32.30 (b)
It is obvious that total impedance of the transformer as referred to primary is given by
Z
01
=
22
01 01
()
RX
+
...Fig. 32.30 (a)
and Z
02
=
22
02 02
()
RX
+
...Fig. 32.30 (b)
Example 32.15. A 30 kVA, 2400/120-V, 50-Hz transformer has a high voltage winding resistance
of 0.1 and a leakage reactance of 0.22. The low voltage winding resistance is 0.035 and the
leakage reactance is 0.012 . Find the equivalent winding resistance, reactance and impedance
referred to the (i) high voltage side and (ii) the low-voltage side.
(Electrical Machines-I, Bangalore Univ. 1987)
Solution. K = 120/2400 = 1/20; R
1
= 0.1 , X
1
= 0.22
R
2
= 0.035 and X
2
= 0.012
(i) Here, high-voltage side is, obviously, the primary side. Hence, values as referred to primary side
are
R
01
= R
1
+ R
2
= R
1
+ R
2
/K
2
= 0.1 + 0.035/(1/20)
2
= 14.1
X
01
= X
1
+ X
2
= X
1
+ X
2
/K
2
= 0.22 + 0.12/(1/20)
2
= 5.02
Z
01
=
22 2 2
01 01
14.1 5.02
RX
+= + =
1
15
(ii) R
02
= R
2
+ R
1
= R
2
+ K
2
R
1
= 0.035 + (1/20)
2
× 0.1 = 0.03525
X
02
= X
2
+ X
1
= X
2
+ K
2
X
1
= 0.012 + (1/20)
2
× 0.22 = 0.01255
Z
02
=
22 2 2
02 02
0.0325 0.01255
RX
+= +
=
0.0374
(or Z
02
= K
2
Z
01
= (1/20)
2
× 15 = 0.0375 )
Example 32.16. A 50-kVA, 4,400/220-V transformer has R
1
= 3.45 , R
2
= 0.009 . The values
of reactances are X
1
= 5.2 and X
2
= 0.015 . Calculate for the transformer (i) equivalent resis-
tance as referred to primary (ii) equivalent resistance as referred to secondary (iii) equivalent reac-
tance as referred to both primary and secondary (iv) equivalent impedance as referred to both pri-
mary and secondary (v) total Cu loss, first using individual resistances of the two windings and
secondly, using equivalent resistances as referred to each side.
(Elect. Engg.-I, Nagpur Univ. 1993)
Solution. Full-load I
1
= 50,000/4,400 = 11.36 A (assuming 100% efficiency)
Full-load I
2
= 50,000/2220 = 227 A; K = 220/4,400 = 1/20
(i) R
01
=
2
1
22
0.009
3.45 3.45 3.6
(1 / 2 0)
+= + = +=
R
R
K
7.05
(ii) R
02
= R
2
+ K
2
R
1
= 0.009 + (1/20)
2
× 3.45 = 0.009 + 0.0086 = 0.0176
Also, R
02
= K
2
R
01
= (1/20)
2
× 7.05 = 0.0176 (check)
Transformer
1137
(iii) X
01
= X
1
+ X
2
= X
1
+ X
2
/K
2
= 5.2 + 0.015/(1/20)
2
= 11.2
X
02
= X
2
+ X
1
= X
2
+ K
2
X
1
= 0.015 + 5.2/20
2
= 0.028
AlsoX
02
= K
2
X
01
= 11.2/400 = 0.028 (check)
(iv) Z
01
=
22 2 2
01 01
( ) (7.05 11.2)
RX
+= + =
13.23
Z
02
=
22 2 2
02 02
( ) (0.0176 0.028)
RX
+= + =
0.03311
AlsoZ
02
= K
2
Z
01
= 13.23/400 = 0.0331 (check)
(v) Cu loss = I
1
2
R
2
+ I
2
2
R
2
= 11.36
2
× 3.45 + 227
2
× 0.009 = 910 W
AlsoCu loss = I
1
2
R
01
= 11.36
2
× 7.05 = 910 W
= I
2
2
R
02
= 227
2
× 0.0176 = 910 W
Example 32.17. A transformer with a 10 : 1 ratio and rated at 50-kVA, 2400/240-V, 50-Hz is
used to step down the voltage of a distribution system. The low tension voltage is to be kept
constant at 240 V.
(a) What load impedance connected to low-tension size will be loading the transformer fully at
0.8 power factor (lag) ?
(b) What is the value of this impedance referred to high tension side ?
(c) What is the value of the current referred to the high tension side ?
(Elect. Engineering-I, Bombay Univ. 1987)
Solution. (a) F. L. I
2
= 50,000/240 = 625/3 A; Z
2
=
240
(625/ 3)
=
1.142
(b) K = 240/2400 = 1/10
The secondary impedance referred to primary side is
Z
2
= Z
2
/K
2
= 1.142/(1/10)
2
= 114.2
(c) Secondary current referred to primary side is I
2
= KI
2
= (1/10) × 625/3 = 20.83 A
Example 32.18. The full-load copper loss on the h.v. side of a 100-kVA, 11000/317-V, 1-phase
transformer is 0.62 kW and on the L.V. side is 0.48 kW.
(i) Calculate R
1
, R
2
and R
3
in ohms (ii) the total reactance is 4 per cent, find X
1
, X
2
and X
3
in
ohms if the reactance is divided in the same proportion as resistance.
(Elect. Machines A.M.I.E,. Sec. B, 1991)
Solution. (i) F.L. I
1
= 100 × 10
3
/11000 = 9.1 A. F.L. I
2
= 100 × 10
3
/317 = 315.5 A
Now, I
1
2
R
1
= 0.62 kW or R
1
= 620/9.1
2
= 7.5
I
2
2
R
2
= 0.48 kW, R
2
= 480/315.5
2
= 0.00482
R
2
= R
2
/K
2
= 0.00482 × (11,000/317)
2
= 5.8
% reactance =
101 01
01
1
9.1
100 or 4 100, 48.4
11000
IX X
X
V
××
×= ×=
X
1
+ X
2
= 48.4 . Given R
1
/R
2
= X
1
/X
2
or (R
1
+ R
2
)/R
2
=(X
1
+ X
2
)/X
2
(7.5 + 5.8)/5.8 = 48.4/X
2
′ ∴ X
2
= 21.1
X
1
= 48.4 21.1 = 27.3 , X
2
= 21.1 × (317/11000)
2
= 0.175
Example 32.19. The following data refer to a I-phase transformer :
Turn ratio 19.5 : 1 ; R
1
= 25 ; X
1
= 100 ; R
2
= 0.06 ; X
2
= 0.25 . No-load current =
1.25 A leading the flux by 30°.
The secondary delivers 200 A at a terminal voltage of 500 V and p.f. of 0.8 lagging. Determine
1138
Electrical Technology
by the aid of a vector diagram, the primary applied voltage, the primary p.f. and the efficiency.
(Elect. Machinery-I, Madras Univ. 1989)
Solution. The vector diagram is similar to Fig. 30.28 which has been redrawn as Fig. 32.31. Let us
take V
2
as the reference vector.
V
2
= 500 0° = 500 + j0
I
2
= 200 (0.8 j 0.6) = 160 j 120
Z
2
= (0.06 + j 0.25)
E
2
= V
2
+ I
2
Z
2
= (500 + j 0) + (160 j 120) (0.06 + j 0.25)
= 500 + (39.6 + j 32.8) = 539.6 + j 32.8 = 541 3.5°
Obviously, β =3.5°
E
1
= E
2
/K = 19.5 E
2
= 19.5 (539.6 + j 32.8)
= 10,520 + j 640
∴ − E
1
=10,520 j 640 = 10,540 183.5°
I
2
= I
2
K = (160 + j 120)/19.5
= 8.21 + j 6.16
As seen from Fig. 32.31, I
0
leads V
2
by an angle
= 3.5 ° + 90° + 30° = 123.5°
I
0
= 1.25 123.5°
= 1.25 (cos 123.5° + j sin 123.5°)
= 1.25 (cos 56.5° + j sin 56.5°)
= 0.69 + j 1.04
I
1
= I
2
+ I
0
= (8.21 + j 6.16) + (0.69 + j 1.04)
= 8.9 + j 7.2 = 11.45 141°
V
2
=
E
1
+ I
1
Z
1
= 10,520 j 640 + (8.9 + j 7.5) (25 + j 100)
= 10,520 j 640 942 j 710
= 11,462 j 1350
= 11,540 186.7°
Phase angle between V
1
and I
1
is = 186.7° 141° = 45.7°
primary p.f. = cos 45.7° = 0.698 (lag)
No-load primary input power = V
1
I
0
sin φ
0
= 11,540 × 1.25 × cos 60° = 7,210 W
R
02
= R
2
+ K
2
R
1
= 0.06 + 25/19.5
2
= 0.1257
Total Cu loss as referred to secondary = I
2
2
R
02
= 200
2
× 0.1257 = 5,030 W
Output = V
2
I
2
cos φ
2
= 500 × 200 × 0.8 = 80,000 W
Total losses = 5030 + 7210 = 12,240 W
Input = 80,000 + 12,240 = 92,240 W
η = 80,000/92,240 = 0.8674 or 86.74%
Example 32.20. A 100 kVA, 1100/220 V, 50 Hz, single-phase transformer has a leakage impedance
of (0.1 + 0/40) ohm for the H.V. winding and (0.006 + 0.015) ohm for the L.V. winding. Find the
equivalent winding resistance, reactance and impedance referred to the H.V. and L.V. sides.
(Bharathiar Univ. Nov. 1997)
2
1
V
2
V
1
I
2
I'
2
I
1
E
1
I
0
IX
22
IX
11
IR
22
IR
11
30
o
Fig. 32.31
Transformer
1139
Solution. Turns ratio = (N
1
/N
2
) = (V
1
/ V
2
) = 1100/220 = 5
(i) Referred to H.V. side :
Resistance = r
1
+ r
2
= 0.1 + (25 × 0.006) = 0.25 ohm
Reactance = x
1
+ x
2
= 0.4 + (25 × 0.015) = 0.775 ohm
Impedance = (0.25
2
+ 0.775
2
)
0.5
= 0.8143 ohm
(ii) Referred to L.V. side :
Resistance = 0.25/25 = 0.01
(or resistance = 0.006 + (0.1/25) = 0.01 ohm)
Reactance = 0.775/25 = 0.031 ohm
Impedance = 0.8143/25 = 0.0326 ohm
32.15. Simplified Diagram
The vector diagram of Fig. 32.29 may be
considerably simplified if the no-load current
I
0
is neglected. Since I
0
is 1 to 3 per cent of
full-load primary current I
1
, it may be neglected
without serious error. Fig. 32.32 shows the
diagram of Fig. 32.29 with I
0
omitted altogether.
In Fig. 32.32, V
2
, V
1
, φ
2
are known, hence
E
2
can be found by adding vectorially I
2
R
2
and I
2
X
2
to V
2
. Similarly, V
1
is given by the
vector addition of I
1
R
1
and I
1
X
1
to E
1
. All the
voltages on the primary side can be transferred
to the secondary side as shown in figure, where
the upper part of the diagram has been rotated
through 180°. However, it should be noted
that each voltage or voltage drop should be
multiplied by transformation ratio K.
The lower side of the diagram has been
shown separately in Fig. 32.34 laid horizontally
where vector for V
2
has been taken along
X-axis.
It is a simple matter to find transformer regulation as shown in Fig. 32.34 or Fig. 32.35.
It may be noted that V
2
= KV
1
I
2
(R
02
+ jX
02
) = KV
1
I
2
Z
02
.
1
2
I
2
V
2
IR
22
IX
22
KI X
11
KI R
11
K
V
1
0
=
K
IX
2
21
=
K
IR
2
21
1
2
I
2
V
2
I(X +
K
X)
22 1
2
I(R +
K
R)
22 1
2
IZ
220
K
V
1
0
A
B
C
Fig. 32.33 Fig. 32.34
Fig. 32.32
* Also, V
1
= (V
2
+ I
2
Z
02
) #3/K
1
2
1
IX
22
IX
11
IR
11
K
V
1
KI X
11
K
IR
11
I
2
V
2
V
1
E
1
E
2
IR
22
E=
K
E
21
I=
I
/K
12
0
K
K
1140
Electrical Technology
32.16. Total Approximate Voltage Drop
in a Transformer
When the transformer is on no-load, then V
1
is ap-
proximately equal to E
1
. Hence E
2
= KE
1
= KV
1
. Also,
E
2
=
0
V
2
where
0
V
2
is secondary terminal voltage on no-
load, hence no-load secondary terminal voltage is KV
1
.
The secondary voltage on load is V
2
. The difference be-
tween the two is I
2
Z
02
as shown in Fig. 32.35. The ap-
proximate voltage drop of the transformer as referred to
secondary is found thus :
With O as the centre and radius OC draw an arc cutting OA produced at M. The total voltage drop I
2
Z
02
= AC = AM which is approximately equal to AN. From B draw BD perpendicular on OA produced.
Draw CN perpendicular to OM and draw BL parallel to OM.
Approximate voltage drop
= AN = AD +DN
= I
2
R
02
cos φ + I
2
X
02
sin φ
where φ
1
= φ
2
= φ (approx).
This is the value of approximate voltage drop for a lagging power factor.
The different figures for unity and leading power factors are shown in Fig. 32.36 (a) and (b)
respectively.
I
2
V
2
IR
202
IX
102
IZ
20
2
KV = V
102
0
B
A
C
()a

12
2
I
2
V
2
IX
10
2
IZ
202
IR
20
2
K
V
=V
102
0
B
A
C
()b
Fig. 32.36
The approximate voltage drop for leading power factor becomes
(I
2
R
02
cos φ ± I
2
X
02
sin φ)
In general, approximate voltage drop is (I
2
R
02
cos φ ± I
2
X
02
sin φ)
It may be noted that approximate voltage drop as referred to primary is
(I
1
R
01
cos φ ± I
1
X
01
sin φ)
% voltage drop in secondary is=
202 2 02
02
cos sin
100
IR IX
V
φ± φ
×
=
202 2 02
02 02
100 100
cos sin
IR IX
VV
×
φ± φ
= v
r
cos φ ± v
x
sin φ
where v
r
=
202 1 01
02 1
100 100
percentage resistive drop =
IR I R
VV
=
v
x
=
202 1 01
02 1
100 100
percentage reactive drop =
=
IX I X
VV
Fig. 32.35

12
2
I
2
V
2
IR
20
2
IX
10
2
IZ
202
K
V
=V
102
0
N
M
B
A
D
L
C
Transformer
1141
32.17. Exact Voltage Drop
With reference to Fig. 32.35, it is to be noted that exact voltage drop is AM and not AN. If we add the
quantity NM to AN, we will get the exact value of the voltage drop.
Considering the right-angled triangle OCN, we get
NC
2
= OC
2
ON
2
= (OC + ON) (OC ON) = (OC + ON) (OM ON) = 2 OC × NM
NM = NC
2
/2.OC Now, NC = LC LN = LC BD
NC = I
2
X
02
cos φ − I
2
R
02
sin φ ∴ NM =
2
202 202
02
(cos sin)
2
IX IR
V
φ− φ
For a lagging power factor, exact voltage drop is
= AN + NM = (I
2
R
02
cos φ + I
2
X
02
sin φ) +
2
202 202
02
(cos sin)
2
IX IR
V
φ− φ
For a leading power factor, the expression becomes
=(I
2
R
02
cos φ − I
2
X
02
sin φ) +
2
202 202
02
(cos sin)
2
IX IR
V
φ+ φ
In general, the voltage drop is
=(I
2
R
02
cos φ ± I
2
R
02
sin φ) +
2
202 202
02
(cos sin)
2
IX IR
V
φ± φ
Percentage drop is
=
2
202202 202202
2
02
02
( cos sin ) 100 ( cos sin ) 100
2
φ± φ × φ φ ×
+
m
IR IX IX IR
V
V
= (v
r
cos φ ± v
x
sin φ) + (1/200) (v
x
cos φ
m
v
r
sin φ)
2
The upper signs are to be used for a lagging power factor and the lower ones for a leading power
factor.
Example 32.21. A 230/460-V transformer has a primary resistance of 0.2 and reactance
of 0.5 and the corresponding values for the secondary are 0.75 and 1.8 respectively. Find
the secondary terminal voltage when supplying 10 A at 0.8 p.f. lagging.
(Electric. Machines-II, Bangalore Univ. 1991)
Solution. K = 460/230 = 2; R
02
= R
2
+ K
2
R
1
= 0.75 + 2
2
× 0.2 = 1.55
X
02
= X
2
+ K
2
X
1
= 1.8 + 2
2
× 0.5 = 3.8
Voltage drop = I
2
(R
02
cos φ + X
02
sin φ) = 10 (1.55 × 0.8 + 3.8 × 0.6) = 35.2V
Secondary terminal voltage = 460 35.2 = 424.8 V
Example 32.22. Calculate the regulation of a transformer in which the percentage resistance
drops is 1.0% and percentage reactance drop is 5.0% when the power factor is (a) 0.8 lagging
(b) unity and (c) 0.8 leading. (Electrical Engineering, Banaras Hindu Univ. 1988)
Soluion. We will use the approximate expression of Art 30.16.
(a) p.f. = cos φ = 0.8 lag µ=v
r
cos φ + v
x
sin φ = 1 × 0.8 + 5 × 0.6 = 3.8%
(b) p.f. = cos φ = 1 µ=1 × 1 + 5 × 0 = 1%
(c) p.f. = cos φ = 0.8 lead µ=1 × 0.8 5 × 0.6 =
2.2%
Example 32.23. A transformer has a reactance drop of 5% and a resistance drop of 2.5%. Find
the lagging power factor at which the voltage regulation is maximum and the value of this
regulation. (Elect. Engg. Punjab Univ. 1991)
Solution. The percentage voltage regulation (µ) is given by
1142
Electrical Technology
µ=v
r
cos φ + v
x
sin φ
where v
r
is the percentage resistive drop and v
x
is the percentage reactive drop.
Differentiating the above equation, we get
d
d
µ
φ
= v
r
sin φ + v
x
cos φ
For regulation to be maximum, dµ/dφ = 0 ∴ − v
r
sin φ + v
x
cos φ = 0
or tan φ = v
x
/v
r
= 5/2.5 = 2 ∴ φ = tan
1
(2) = 63.5° Now, cos φ = 0.45 and sin φ = 0.892
Maximum percentage regulation = (2.5 × 0.45) + (5 × 0.892) = 5.585
Maximum percentage regulation is 5.585 and occurs at a power factor of 0.45 (lag).
Example 32.24. Calculate the percentage voltage drop for a transformer with a percentage
resistance of 2.5% and a percentage reactance of 5% of rating 500 kVA when it is delivering
400 kVA at 0.8 p.f. lagging. (Elect. Machinery-I, Indore Univ. 1987)
Solution. % drop =
(% ) cos (% ) sin
ff
RI XI
II
φφ
+
where I
f
is the full-load current and I the actual current.
% drop =
(% )
(% )
rating rating
XkVAR
RkW
kVA kVA
+
In the present case, kW = 400 × 0.8 = 320 and kVAR = 400 × 0.6 = 240
% drop =
2.5 320 5 240
500 500
××
+=
4%
32.18. Equivalent Circuit
The transformer shown diagrammatically in Fig. 32.37 (a) can be resolved into an equivalent
circuit in which the resistance and leakage reactance of the transformer are imagined to be external to
the winding whose only function then is to transform the voltage (Fig. 32.37 (b)). The no-load
E
1
I
1
V
1
V
2
R
1
R
2
Z
1
Z
2
I
1
I
0
I
2
I'
2
X
1
X
2
V
1
E
1
E
2
V
2
X
0
R
0
I
w
I
I
2
E
2
()a
()b
Z
L
Ideal
Transform
er
Fig. 32.37
current I
0
is simulated by pure inductance X
0
taking the magnetising component I
µ
and a non-inductive
resistance R
0
taking the working component I
w
connected in parallel across the primary circuit.
The value of E
1
is obtained by subtracting vectorially I
1
Z
1
fromV
1
. The value of X
0
= E
1
/I
0
and of
R
0
= E
1
/I
w
. It is clear that E
1
and E
2
are related to each other by expression
E
2
/E
1
= N
2
/N
1
= K.
To make transformer calculations simpler, it is preferable to transfer voltage, current and impedance
Transformer
1143
either to the primary or to the secondary. In that case, we would have to work in one winding only which is
more convenient.
The primary equivalent of the secondary induced voltage is E
2
= E
2
/K = E
1
.
Similarly, primary equivalent of secondary terminal or output voltage is V
2
= V
2
/K.
Primary equivalent of the secondary current is I
2
= KI
2
.
For transferring secondary impedance to primary K
2
is used.
R
2
= R
2
/K
2
, X
2
= X
2
/K
2
, Z
2
= Z
2
/K
2
The same relationship is used for shifting an external load impedance to the primary.
The secondary circuit is shown in Fig. 32.38 (a) and its equivalent primary values are shown in Fig.
32.38 (b).
E
2
V
2
R
2
Z
2
I
2
X
2
Z
L
()a
Load
E' =E
21
I'
2
V' =V /K
22
R' =R /K
22
2
X' =X /K
22
2
Z' =Z /K
LL
2
()b
Fig. 32.38
The total equivalent circuit of the transformer is obtained by adding in the primary impedance as shown
in Fig. 32.39. This is known as the exact equivalent circuit but it presents a somewhat harder circuit
problem to solve. A simplification can be made by transferring the exciting circuit across the terminals as in
Fig. 32.40 or in Fig. 32.41 (a). It should be noted that in this case X
0
= V
1
/I
µ
.
E=
E
21
V
L
I
2
I
2
R
2
X
2
Z
2
Z
L
I
1
I
0
E
1
I
1
X
0
V
1
R
0
R
1
Z
1
X
1
I
w
Load
E=
E
21
I
2
I'
2
R
2
Z
L
I
1
I
1
X
0
V
1
I
0
R
0
R
1
X
1
X
2
Load
Fig. 32.39 Fig. 32.40
Further simplification may be achieved by omitting I
0
altogether as shown in Fig. 32.41(b).
From Fig. 32.39 it is found that total impedance between the input terminal is
Z = Z
1
+ Z
m
| | (Z
2
+ Z
L
) =
2
1
2
()
()
mL
mL
′+

+

+′+

ZZ Z
Z
ZZZ
where Z
2
= R
2
+ jX
2
and Z
m
= impedance of the exciting circuit.
This is so because there are two parallel circuits, one having an impedance of Z
m
and the other
having Z
2
and Z
L
in series with each other.
V
1
=
2
11
2
()
()
mL
mL
′+

+

+′+

ZZ Z
IZ
ZZZ
1144
Electrical Technology
V
2
I
2
I'
2
X
1
X
2
X
01
Z
L
I
1
I
1
I
0
X
0
V
1
I
0
R
0
R
1
R
01
R
2
Load
I=I
12
I=I
12
X
01
Z
01
R
01
Z
L
V
1
V
2
Load
Fig. 32.41 (a) Fig. 32.41 (b)
Example 32.25. The parametres of a 2300/230 V, 50-Hz transfomer are given below :
R
1
= 0.286 R
2
= 0.319 R
0
= 250
X
1
= 0.73 X
2
= 0.73 X
0
= 1250
The secondary load impedance Z
L
= 0.387 + j 0.29. Solve the exact equivalent circuit with
normal voltage across the primary.
Solution. K = 230/2300 = 1/10; Z
L
= 0.387 + j 0.29
Z
L
= Z
L
/K
2
= 100 (0.387 + j 0.29) = 38.7 + j 29 = 48.4 36.8°
Z
2
+ Z
L
= (38.7 + 0.319) + j(29 + 0.73) = 39.02 + j29.73 = 49.0 37.3°
Y
m
= (0.004 j 0.0008); Z
m
= 1/Y
m
= 240 + j48 = 245 11.3°
Z
m
+ (Z
2
+ Z
L
) = (240 + j48) + (39 + j29.7) = 290 15.6°
I
1
=
1
2
1
2
2300 0
()
0.286 0.73 41.4 33
()
mL
mL
j
∠°
=

′+
++°

+
′+
V
ZZ Z
Z
ZZ Z
=
2300 0
54.8 33.7
42 33.7
∠°
=∠°
∠°
Now I
2
=
1
2
245 11.3
54.8 33.7
( ) 290 15.6
m
Lm
∠°
×=°×
′+ + °
Z
I
ZZ Z
= 54.8 ∠ − 33.7° × 0.845 ∠− 4.3° = 46.2 ∠ − 38°
I
0
=
2
1
2
()
49 37.3
54.8 33.7
( ) 290 15.6
L
mL
′+
∠°
×=°×
+′+ ∠°
ZZ
I
ZZZ
=54.8 ∠−33.7° × 0.169 21.7° = 9.26 ∠−12°
Input power factor = cos 33.7° = 0.832 lagging
Power input = V
1
I
1
cos φ
1
= 2300 × 54.8 × 0.832 = 105 kW
Power output = 46.2
2
× 38.7 = 82.7 kW
Primary Cu loss = 54.8
2
× 0.286 = 860 W
Secondary Cu loss = 46.2
2
× 0.319 = 680 W; Core loss = 9.26
2
× 240 = 20.6 kW
η = (82.7/105) × 100 = 78.8%; V
2
= I
2
Z
L
= 46.2 × 48.4 = 2,240V
Regulation =
2300 2240
100
2240
×=
2.7%
Example 32.26. A transformer has a primary winding with a voltage-rating of 600 V. Its
secondary-voltage rating is 1080 V with an additional tap at 720 V. An 8 kW resistive load is
connected across 1080-V output terminals. A purely inductive load of 10kVA is connected across
the tapping point and common secondary terminal so as to get 720 V. Calculate the primary current
and its power-factor. Correlate it with the existing secondary loads. Neglect losses and magnetizing
current. (Nagpur University, Winter 1999)
Transformer
1145
Solution. Loads are connected as shown in Fig. 32.42.
I
r
2
=
8000
7.41 at unity p.f.
1080
=
I
L
2
= 10000/720 = 13.89 at zero lagging p.f.
These are reflected on to the primary sides with appropriate ratios of turns, with corresponding power-
factors. If the corresponding transformed currents are represented by the above symbols modified by
dashed superscripts,
I
r
2
= 7.41 × 1080/600 = 13.34 A at unity p.f.
I
L
2
= 13.89 × 720/600 = 16.67 A at zero lag. p.f.
Hence, I
r
2
=[I
r
2
2
+ I
L
2
2
]
0.5
= 21.35 A, at 0.625 lag p.f.
Fig. 32.42
Correlation : Since losses and magnetizing current are ignored, the calculations for primary current
and its power-factor can also be made with data pertaining to the two Loads (in kW/kVAR), as supplied by
the 600 V source.
S = Load to be supplied : 8 kW at unity p.f. and 10 kVAR lagging
Thus, S = P + jQ = 8 j 10 kVA
S =(8
2
+ 10
2
)
0.5
= 12.8 kVA
Power factor = cos φ = 8/12.8 = 0.625 lag
Primary current = 12.8 × 1000/600 = 21.33 A
32.19. Transformer Tests
As shown in Ex 32.25, the performance of a transformer can be calculated on the basis of its equivalent
circuit which contains (Fig. 32.41) four main parameters, the
equivalent resistance R
01
as referred to primary (or secondary
R
02
), the equivalent leakage reactance X
01
as referred to
primary (or secondary X
02
), the core-loss conductance G
0
(or resistance R
0
) and the magnetising susceptance B
0
(or
reactance X
0
). These constants or parameters can be easily
determined by two tests (i) open-circuit test and (ii) short-
circuit test. These tests are very economical and convenient,
because they furnish the required information without actually
loading the transformer. In fact, the testing of very large a.c.
machinery consists of running two tests similar to the open
and short-circuit tests of a transformer.
Small transformer
1146
Electrical Technology
32.20. Open-circuit or No-load Test
The purpose of this test is to determine
no-load loss or core loss and no-load I
0
which
is helpful in finding X
0
and R
0
.
One winding of the transformer –
whichever is convenient but usually high
voltage winding – is left open and the other
is connected to its supply of normal voltage
and frequency. A wattmeter W, voltmeter V
and an ammeter A are connected in the low-
voltage winding i.e. primary winding in the
present case. With normal voltage applied to the primary, normal flux will be set up in the core, hence
normal iron losses will occur which are recorded by the wattmeter. As the primary no-load current I
0
(as
measured by ammeter) is small (usually 2 to 10% of rated load current), Cu loss is negligibly small in primary
and nil in secondary (it being open). Hence, the wattmeter reading represents practically the core loss under
no-load condition (and which is the same for all loads as pointed out in Art. 32.9).
It should be noted that since I
0
is itself very small, the pressure coils of the wattmeter and the
voltmeter are connected such that the current in them does not pass through the current coil of the
wattmeter.
Sometimes, a high-resistance voltmeter is connected across the secondary. The reading of the voltmeter
gives the induced e.m.f. in the secondary winding. This helps to find transformation ratio K.
The no-load vector diagram is shown in Fig. 32.16. If W is the wattmeter reading (in Fig. 32.43),
then
W = V
1
I
0
cos φ
0
cos φ
0
= W/V
1
I
0
I
µ
= I
0
sin φ
0
, I
w
= I
0
cos φ
0
X
0
= V
1
/I
µ
and R
0
= V
1
/I
w
Or since the current is practically all-exciting current when a transformer is on no-load (i.e. I
0
I
µ
) and
as the voltage drop in primary leakage impedance is small*, hence the exciting admittance Y
0
of the trans-
former is given by I
0
= V
1
Y
0
or Y
0
= I
0
/V
1
.
The exciting conductance G
0
is given by W = V
1
2
G
0
or G
0
= W/V
1
2
.
The exciting susceptance B
0
=
22
00
()
YG
Example. 32.27. In no-load test of single-phase transformer, the following test data were
obtained :
Primary voltage : 220 V ; Secondary voltage : 110 V ;
Primary current : 0.5 A ; Power input : 30 W.
Find the following :
(i) The turns ratio (ii) the magnetising component of no-load current (iii) its working (or loss)
component (iv) the iron loss.
Resistance of the primary winding = 0.6 ohm.
Draw the no-load phasor diagram to scale. (Elect. Machine A.M.I.E. 1990)
Solution. (i) Turn ratio, N
1
/N
2
= 220/110 = 2
(ii) W = V
1
I
0
cos φ
0
;cos φ
0
= 30/220 × 0.5 = 0.273 ; sin φ
0
= 0.962
I
µ
= I
0
sin φ
0
= 0.5 × 0.962 = 0.48 A
* If it is not negligibly small, then I
0
= E
1
Y
0
i.e. instead of V
1
we will have to use E
1
.
I
0
V
1
E=V
22
W
V
A
Low
High
Fig. 32.43
Transformer
1147
(iii) I
w
= I
0
cos φ
0
= 0.5 × 0.273 = 0.1365 A
(iv) Primary Cu loss = I
0
2
R
1
= 0.5
2
× 0.6 = 0.15 W
Iron loss = 30 0.15 = 29.85 W
Example 32.28. A 5 kVA 200/1000 V, 50 Hz, single-phase transformer gave the following test
results :
O.C. Test (L.V. Side) : 2000 V, 1.2 A, 90 W
S.C. Test (H.V. Side) : 50 V, 5A, 110 W
(i) Calculate the parameters of the equivalent circuit referred to the L.V. side.
(ii) Calculate the output secondary voltage when delivering 3 kW at 0.8 p.f. lagging, the input
primary voltage being 200 V. Find the percentage regulation also.
(Nagpur University, November 1998)
Solution. (i) Shunt branch parameters from O.C. test (L.V. side) :
R
0
= V
2
/P
i
= 200
2
/90 = 444 ohms, I
ao
= 200/444 = 0.45 amp
I
µ
= (1.2
2
0.45
2
)
0.5
= 1.11 amp, X
m
= 200/1.11 = 180.2 ohms
All these are referred to L.V. side.
(ii) Series-branch Parameters from S.C test (H.V side) :
Since the S.C. test has been conducted from H.V. side, the parameters will refer to H.V. side.
They should be converted to the parameters referred to L.V. side by transforming them suitably.
From S.C. Test readings, Z = 50/5 = 10 ohms
R = 110/25 = 4.40 ohms, X = (10
2
4.4
2
)
0.5
= 8.9 ohms
These are referred to H.V. side.
For referring these to L.V. side, transform these using the ratio of turns, as follows :
r
1
= 4.40 × (200/1000)
2
= 0.176 ohm
x
1
= 8.98 × (200/1000)
2
= 0.36 ohm
Equivalent circuit can be drawn with R
0
and X
m
calculated above and r
1
and x
1
as above.
L.V. Current at rated load= 5000/200 = 25 A
L.V. Current at 3 kW at 0.8 lagging p.f. = (3000/0.80)/200 = 18.75 A
Regulation at this load = 18.75 (r
1
cos φ + x
1
sin φ)
= 18.75 (0.176 × 0.80 + 0.36 × 0.6)
= + 6.69 Volts = + (6.69/200) × 100% = + 3.345%
This is referred to L.V. side, and positive sign means voltage drop.
Regulation in volts ref. to H.V. side = 6.69 × 1000/200 = 33.45 V
With 200 V across primary (i.e. L.V. side), the secondary (i.e. H.V. side)
terminal voltage = 1000 33.45 = 966.55 V
Note : Since approximate formula for voltage regulation has been used, the procedure is simpler, and faster.
32.21. Separation of Core Losses
The core loss of a transformer depends upon the frequency and the maximum flux density when
the volume and the thickness of the core laminations are given. The core loss is made up of two parts
(i) hysteresis loss W
h
= PB
1.6
max
f as given by Steinmetz’s empirical relation and (ii) eddy current loss W
e
= QB
2
max
f
2
where Q is a constant. The total core-loss is given by
W
i
= W
h
+ W
e
= PB
1.6
max
f
2
+ QB
2
max
f
2
If we carry out two experiments using two different frequencies but the same maximum flux
density, we should be able to find the constants P and Q and hence calculate hysteresis and eddy
current losses separately.
1148
Electrical Technology
Example 32.29. In a transformer, the core loss is found to be 52 W at 40 Hz and 90 W at 60 Hz
measured at same peak flux density. Compute the hysteresis and eddy current losses at 50 Hz.
(Elect. Machines, Nagpur Univ. 1993)
Solution. Since the flux density is the same in both cases, we can use the relation
Total core loss W
i
= Af + Bf
2
or W
i
/f = A + Bf
52/40 = A + 40B and 90/60 = A + 60B ; A = 0.9 and B = 0.01
At 50 Hz, the two losses are
W
h
= A
f
= 0.9 × 50 = 45 W ; W
e
= Bf
2
= 0.01 × 50
2
= 25 W
Example 32.30. In a power loss test on a 10 kg specimen of sheet steel laminations, the maximum
flux density and waveform factor are maintained constant and the following results were obtained:
Frequency (Hz) 25 40 50 60 80
Total loss (watt) 18.5365066104
Calculate the eddy current loss per kg at a frequency of 50 Hz.
(Elect. Measur. A.M.I.E. Sec B, 1991)
Solution. When flux density and wave form factor remain constant, the expression for iron loss
can be written as
W
i
= Af + Bf
2
or W
i
/f = A + Bf
The values of W
i
/ f for different frequencies are as under :
f 25 40 50 60 80
W
i
/ f 0.74 0.9 1.0 1.1 1.3
The graph between f and W
i
/f has been plotted in Fig. 32.44. As seen from it, A = 0.5 and
B = 0.01
Eddy current loss at 50 Hz = Bf
2
= 0.01 × 50
2
= 25 W
Eddy current loss/kg = 25/10 = 2.5 W
Example 32.31. In a test for the determination of the losses of a
440-V, 50-Hz transformer, the total iron losses were found to be 2500 W
at normal voltage and frequency. When the applied voltage and
frequency were 220 V and 25 Hz, the iron losses were found to be 850 W.
Calculate the eddy-current loss at normal voltage and frequency.
(Elect. Inst. and Meas. Punjab Univ. 1991)
Solution. The flux density in both cases is the same because in
second case voltage as well as frequency are halved. Flux density remaining the same, the eddy current loss
is proportional to f
2
and hysteresis loss f.
Hysteresis loss f = Af and eddy current loss f
2
= Bf
2
where A and B are constants.
Total iron loss W
i
= Af + Bf
2
i
W
f
= A + Bf ...(i)
Now, when f = 50 Hz : W
i
= 2500 W
and when f = 25 Hz ; W
i
= 850 W
Using these values in (i) above, we get, from Fig. 32.44
2,500/50 = A + 50 B and 850/25 = A + 25 B B = 16/25 = 0.64
Hence, at normal p.d. and frequency
eddy current loss = Bf
2
= 0.64 × 50
2
= 1600 W
Hystersis loss = 2500 1600 = 900 W
Fig. 32.44
W
i
f
A
B
f
Transformer
1149
Example 32.32. When a transformer is connected to a 1000-V, 50-Hz supply the core loss is
1000 W, of which 650 is hysteresis and 350 is eddy current loss. If the applied voltage is raised to
2,000 V and the frequency to 100 Hz, find the new core losses.
Solution. Hysteresis loss W
h
1.6 1.6
max max
BfPBf
=
Eddy current loss W
e
B
2
max
f
2
= QB
2
max
f
2
From the relation E = 4.44 fNB
max
A volt, we get B
max
E/f
Putting this value of B
max
in the above equations, we have
W
h
=
2
1.6 0.6
E
PfPEf
f

=


and W
e
=
2
22

=


E
QfQE
f
In the first case, E = 1000 V, f = 50 Hz, W
h
= 650 W, W
e
= 350 W
650 = P × 1000
1.6
× 50
0.6
P = 650 × 1000
1.6
× 50
0.6
Similarly, 350 = Q × 1000
2
Q = 350 × 1000
2
Hence, constants P and Q are known.
Using them in the second case, we get
W
h
= (650 × 1000
1.6
× 50
0.6
) × 2000
1.6
× 100
0.6
= 650 × 2 = 1,300 W
W
e
= (350 × 1000
2
) × 2,000
2
= 350 × 4 = 1,400 W
Core loss under new condition is = 1,300 + 1,400 = 2700 W
Alternative Solution
Here, both voltage and frequency are doubled, leaving the flux density unchanged.
With 1000 V at 50 Hz
W
h
= Af or 650 = 50 A ; A = 13
W
e
= Bf
2
or 350 = B × 50
2
; B = 7/50
With 2000 V at 100 Hz
W
h
= Af = 13 × 100 = 1300 W and
W
e
= Bf
2
= (7/50) × 100
2
= 1400 W
New core loss = 1300 + 1400 = 2700 W
Example 32.33. A transformer with normal voltage impressed has a flux density of 1.4 Wb/m
2
and a core loss comprising of 1000 W eddy current loss and 3000 W hysteresis loss. What do these
losses become under the following conditions ?
(a) increasing the applied voltage by 10% at rated frequency.
(b) reducing the frequency by 10% with normal voltage impressed.
(c) increasing both impressed voltage and frequency by 10 per cent.
(Electrical Machinery-I, Madras Univ. 1985)
Solution. As seen from Ex. 32.32
W
h
= PE
1.6
f
0.6
and W
e
= QE
2
From the given data, we have 3000 = PE
1.6
f
0.6
...(i)
and 1000 = QE
2
...(ii)
where E and f are the normal values of primary voltage and frequency.
(a) Here voltage becomes = E + 10% E = 1.1 E
The new hysteresis loss is W
h
= P (1.1 E)
1.6
f
0.6
...(iii)
Dividing Eq. (iii) by (i), we get
3000
h
W
=1.1
1.6
; W
h
= 3000 × 1.165 = 3495 W
1150
Electrical Technology
The new eddy-current loss is
W
e
= Q (1.1. E)
2
1000
e
W
= 1.1
2
W
e
= 1000 × 1.21 = 1210 W
(b) As seen from Eq. (i) above eddy-current loss would not be effected. The new hysteresis loss is
W
h
= PE
1.6
(0.9 f)
0.6
...(iv)
From (i) and (iv), we get
3000
h
W
= 0.9
0.6
, W
h
= 3000 × 1.065 = 3,196 W
(c) In this case, both E and f are increased by 10%. The new losses are as under :
W
h
= P (1.1 E)
1.6
(1.1 f)
0.6
3000
h
W
=1.1
1.6
× 1.1
0.6
= 1.165 × 0.944
W
h
= 3000 × 1.165 × 0.944 = 3,299 W
As W
e
is unaffected by changes in f, its value is the same as found in (a) above i.e. 1210 W
Example 32.34. A transformer is connected to 2200 V, 40 Hz supply. The core-loss is 800 watts
out of which 600 watts are due to hysteresis and the remaining, eddy current losses. Determine the
core-loss if the supply voltage and frequency are 3300 V and 60 Hz respectively.
(Bharathiar Univ. Nov. 1997)
Solution. For constant flux density (i.e. constant V/f ratio), which is fulfilled by 2200/40 or
3300/60 figures in two cases,
Core-loss = A f + B f
2
First term on the right-hand side represents hysteresis-loss and the second term represents the
eddy-current loss.
At 40 Hz, 800 = 600 + eddy current loss.
Thus, A f = 600, or A = 15
B f
2
= 200, or B = 200/1600 = 0.125
At 60 Hz, core-loss = 15 × 60 + 0.125 × 60
2
= 900 + 450
= 1350 watts
32.22 Short-Circuit or Impedance Test
This is an economical method for determining the following :
(i) Equivalent impedance (Z
01
or Z
02
),
leakage reactance (X
01
or X
02
) and
total resistance (R
01
or R
02
) of the transformer
as referred to the winding in which the mea-
suring instruments are placed.
(ii) Cu loss at full load (and at any de-
sired load). This loss is used in calculating
the efficiency of the transformer.
(iii) Knowing Z
01
or Z
02
, the total
voltage drop in the transformer as referred
to primary or secondary can be calculated and hence regulation of the transformer determined.
In this test, one winding, usually the low-voltage winding, is solidly short-circuited by a thick conductor
(or through an ammeter which may serve the additional purpose of indicating rated load current) as shown
in Fig. 32.45.
Fig. 32.45
I
0
V
1
L.V.
Supply
W
V
A
Low
High
Short Circuit
Transformer
1151
V' =0
1
X'
2
R'
2
X
1
R
1
E=E'
12
I=I
12
V
1
V' =0
2
V
1
R
01
Z
01
X
01
I=I'
12
Short Circuit
Fig. 32.46
A low voltage (usually 5 to 10% of normal primary voltage) at correct frequency (though for Cu losses
it is not essential) is applied to the primary and is cautiously increased till full-load currents are flowing both
in primary and secondary (as indicated by the respective ammeters).
Since, in this test, the applied voltage is a small percentage of the normal voltage, the mutual flux Φ
produced is also a small percentage of its normal value (Art. 32.6). Hence, core losses are very small with
the result that the wattmeter reading represent the full-load Cu loss or I
2
R loss for the whole transformer i.e.
both primary Cu loss and secondary Cu loss. The equivalent circuit of the transformer under short-circuit
condition is shown in Fig. 32.46. If V
sc
is the voltage required to circulate rated load currents, then Z
01
=
V
sc
/I
1
Also W = I
1
2
R
01
R
01
= W/I
1
X
01
=
()
22
01 01
ZR
In Fig. 32.47 (a) the equivalent circuit
vector diagram for the short-circuit test is
shown. This diagram is the same as shown
in Fig. 32.34 except that all the quantities
are referred to the primary side. It is obvious
that the entire voltage V
SC
is consumed in
the impedance drop of the two windings.
If R
1
can be measured, then knowing
R
01
, we can find R
2
= R
01
R
1
. The impedance triangle can then be divided into the appropriate
equivalent triangles for primary and secondary as shown in Fig. 32.47 (b).
32.23. Why Transformer Rating in kVA ?
As seen, Cu loss of a transformer depends on current and iron loss on voltage. Hence, total
transformer loss depends on volt-ampere (VA) and not on phase angle between voltage and current
i.e. it is independent of load power factor. That is why rating of transformers is in kVA and not in kW.
Example 32.35. The primary and secondary windings of a 30 kVA 76000/230, V, 1-phase
transformer have resistance of 10 ohm and 0.016 ohm respectively. The reactance of the transformer
referred to the primary is 34 ohm. Calculate the primary voltage required to circulate full-load
current when the secondary is short-circuited. What is the power factor on short circuit ?
(Elect. Machines AMIE Sec. B 1991)
Solution. K = 230/6000 = 23/600, X
01
= 34 ,
R
01
= R
1
+ R
2
/K
2
= 10 + 0.016 (600/23)
2
= 20.9
V=
I
Z
SC 1 01
IR
12
IR
11
I
1
IX
11
IX
12
A
B
C
D
E
()a
Z
0
1
R
2
R
2
R
1
X
1
Z
1
X
2
Z
2
D
E
()b
Fig. 32.47
1152
Electrical Technology
Z
01
=
22 22
01 01
20.9 34 40
+= +=
RX
F.L., I
1
= 30,000/6000 = 5 A ; V
SC
= I
1
Z
01
= 5 × 40 = 200 V
Short circuit p.f. = R
01
/Z
01
= 20.9/40 = 0.52
Example 32.36. Obtain the equivalent circuit of a 200/400-V, 50-Hz, 1-phase transformer from
the following test data :
O.C test : 200 V, 0.7 A, 70 W – on L.V. side
S.C. test : 15 V, 10 A, 85 W – on H.V. side
Calculate the secondary voltage when delivering 5 kW at 0.8 p.f. lagging, the primary voltage
being 200V. (Electrical Machinery-I, Madras Univ. 1987)
Solution. From O.C. Test
V
1
I
0
cos φ
0
= W
0
200 × 0.7 × cos φ
0
=70
cos φ
0
= 0.5 and sin φ
0
= 0.866
I
w
= I
0
cos φ
0
= 0.7 × 0.5 = 0.35 A
I
µ
= I
0
sin φ
0
= 0.7 × 0.866 = 0.606 A
R
0
= V
1
/I
w
= 200/0.35 = 571.4
X
0
= V
1
/I
µ
= 200/0.606 = 330
As shown in Fig. 32.48, these values refer to primary
i.e. low-voltage side.
From S.C. Test
It may be noted that in this test, instruments have been placed in the secondary i.e. high-voltage
winding whereas the low-voltage winding i.e. primary has been short-circuited.
Now, as shown in Art. 32.32
Z
02
= V
sc
/I
2
= 15/10 = 1.5 ; K = 400/200 = 2
Z
01
= Z
02
/K
2
= 1.5/4 = 0.375
Also I
2
2
R
02
= W ; R
02
= 85/100 = 0.85
R
01
= R
02
/K
2
= 0.85/4 = 0.21
X
01
=
22 2 2
01 01
0.375 0.21 0.31
ZR
−= =
Output kVA = 5/0.8 ; Output current I
2
= 5000/0.8 × 400 = 15.6 A
This value of I
2
is approximate because V
2
(which is to be calculated as yet) has been taken equal
to 400 V (which, in fact, is equal to E
2
or
0
V
2
).
Now, Z
02
=1.5 , R
02
= 0.85 Ω∴X
02
=
22
1.50.851.24
−=
Total transformer drop as referred to secondary
= I
2
(R
02
cos φ
2
+ X
02
sin φ
2
) = 15.6 (0.85 × 0.8 + 1.24 × 0.6) = 22.2 V
V
2
= 400 22.2 = 377.8 V
Example 32.37. Starting from the ideal transformer, obtain the approximate equivalent circuit
of a commercial transformer in which all the constants are lumped and represented on one side.
A 1-phase transformer has a turn ratio of 6. The resistance and reactance of primary winding
are 0.9 and 5 respecitvely and those of the secondary are 0.03 and 0.13 respectively. If
330 -V at 50-Hz be applied to the high voltage winding with the low-voltage winding short-
circuited, find the current in the low-voltage winding and its power factor. Neglect magnetising
current.
Fig. 32.48
Transformer
1153
Solution. Here K = 1/6 ; R
01
= R
1
+ R
2
= 0.9 + (0.03 × 36) = 1.98
X
01
= X
1
+ X
2
= 5 + (0.13 × 36) = 9.68
Z
01
=
()
22
9.68 1.98
+
= 9.9 ; V
SC
= 330 V
Full-load primary current I
1
= V
sc
/Z
01
= 330/0.9 = 100/3 A
As I
0
is negligible, hence I
1
= I
2
= 100/3 A. Now, I
2
= KI
2
F.L. secondary current I
2
= I
2
K = (100/3) × 6 = 200 A
Now, Power input on short-circuit = V
SC
I
1
cos φ
SC
= Cu loss = I
1
2
R
01
(100/3)
2
× 1.98 = 330 × (100/3) × cos φ
SC
; cos φ
SC
= 0.2
Example 32.38. A 1-phase, 10-kVA, 500/250-V, 50-Hz transformer has the following constants:
Reactance : primary 0.2 ; secondary 0.5
Resistance : primary 0.4 ; secondary 0.1
Resistance of equivalent exciting circuit referred to primary, R
0
= 1500
Reactance of equivalent exciting circuit referred to primary, X
0
= 750
What would be the reading of the instruments when the transformer is connected for the open-
circuit and short-circuit tests ?
Solution. While solving this question, reference may please be made to Art. 30.20 and 30.22.
O.C. Test
I
µ
= V
1
/X
0
= 500/750 = 2/3 A ; I
w
= V
1
/R
0
= 500/1500 = 1/3 A
I
0
=
22
1/3) (2/3)

+

= 0.745 A
No-load primary input = V
1
I
w
= 500 × 1/3 = 167 W
Instruments used in primary circuit are : voltmeter, ammeter and wattmeter, their readings being
500 V, 0.745 A and 167 W respectively.
S.C. Test
Suppose S.C. test is performed by short-circuiting the l.v. winding i.e. the secondary so that all
instruments are in primary.
R
01
= R
1
+ R
2
= R
1
+ R
2
/K
2
; Here K = 1/2 R
01
= 0.2 + (4 × 0.5) = 2.2
Similarly, X
01
= X
1
+ X
2
= 0.4 + (4 × 0.1) = 0.8
Z
01
=
22
(2.2 0.8 )
+
= 2.341
Full-load primary current
I
1
= 10,000/500 = 20 A V
SC
= I
1
Z
01
= 20 × 2.341 = 46.8 V
Power absorbed = I
2
1
R
01
= 20
2
× 2.2 = 880 W
Primary instruments will read : 46.8 V, 20 A, 880 W.
Example 32.39. The efficiency of a 1000-kVA, 110/220 V, 50-Hz, single-phase transformer, is
98.5 % at half full-load at 0.8 p.f. leading and 98.8 % at full-load unity p.f. Determine (i) iron loss
(ii) full-load copper loss and (iii) maximum efficiency at unity p.f.
(Elect. Engg. AMIETE Sec. A Dec. 1991)
Solution. Output at F.L. unity p.f. = 1000 × 1 = 1000 kW
F.L. input = 1000/0.988 = 1012.146 kW
F.L. lossses = 1012.146 1000 = 12.146 kW
If F.L. Cu and iron losses are x and y respectively then
x + y = 12.146 kW ...(i)
1154
Electrical Technology
Input at half F.L. 0.8 p.f. = 500 × 0.8/0.985 = 406.091 kW
Total losses at half F.L. = 406.091 400 = 6.091 kW
Cu loss at half-load = x (1/2)
2
= x/4
x/4 + y = 6.091 ...(ii)
From Eqn. (i) and (ii), we get (i) x = 8.073 kW and (ii) y = 4.073 kW
(iii) kVA for η
max
= 1000 ×
4.073/8.073
= 710.3 kVA
Output at u.p.f. = 710.3 × 1 = 710.3 kW
Cu loss = iron loss = 4.037 kW ; Total loss = 2 × 4.037 = 8.074 kW
∴η
max
= 710.3/(710.3 + 8.074) = 0.989 or 98.9 %
Example 32.40. The equivalent circuit for a 200/400-V step-up transformer has the following
parameters referred to the low-voltage side.
Equivalent resistance = 0.15 ; Equivalent reactance = 0.37
Core-loss component resistance = 600 ; Magnetising reactance = 300
When the transformer is supplying a load at 10 A at a power factor of 0.8 lag, calculate (i) the
primary current (ii) secondary terminal voltage. (Electrical Machinery-I, Bangalore Univ. 1989)
Solution. We are given the following :
R
01
= 0.15 , X
01
= 0.37 ; R
0
= 600 , X
0
= 300
Using the approximate equivalent circuit of Fig. 32.41, we have,
I
µ
= V
1
/X
0
= 200/300 = (2/3) A
I
w
= V
1
/R
0
= 200/600 = (1/3) A
I
0
=
22 2 2
(2/ 3) (1/3)
w
II
µ
+= +
= 0.745 A
As seen from Fig. 32.49
tan θ =
1/3 1
2/3 2
w
I
I
µ
==
; θ = 26.6º
∴φ
0
= 90º 26.6º = 63.4º ; Angle between I
0
and
I
2
= 63.4º 36.9º = 26.5º; K = 400/200 = 2
I
2
= KI
2
= 2 × 10 = 20 A
(i) I
1
= (0.745
2
+ 20
2
+ 2 × 0.745 × 20 × cos 26.5º)
1/2
= 20.67 A
(ii) R
02
= K
2
R
01
= 2
2
× 0.15 = 0.6
X
02
= 22 × 0.37 = 1.48
Approximate voltage drop
= I
2
(R
02
cos φ + X
02
sin φ)
= 10 (0.6 × 0.8 + 1.48 × 0.6) = 13.7 V
Secondary terminal voltage = 400 13.7
= 386.3 V
Example 32.41. The low voltage winding of a 300-kVA, 11,000/2500-V, 50-Hz transformer has
190 turns and a resistance of 0.06. The high-voltage winding has 910 turns and a resistance of
1.6
. When the l.v. winding is short-circuited, the full-load current is obtained with 550-V applied
to the h.v. winding. Calculate (i) the equivalent resistance and leakage reactance as referred to h.v.
side and (ii) the leakage reactance of each winding.
Solution. Assuming a full-load efficiency of 0.985, the full-load primary current is
I
1
V
1
36.9
o
63.4
o
36.9
o
'
*
I=
KI
22
I
0
I
w
I=
10A
2
I
Fig. 32.49
Transformer
1155
= 300,000/0.985 × 11,000 = 27.7A
Z
01
= 550/27.7 = 19.8 ; R
2
= R
2
/K
2
= 0.06 (910/190)
2
= 1.38
R
01
= R
1
+ R
2
= 1.6 + 1.38 = 2.98
X
01
=
()
22 2 2
01 01
(19.8 2.98 )
ZR
−=
= 19.5
Let us make another assumption that for each winding the ratio (reactance/resistance) is the same, then
X
1
= 19.5 × 1.6/2.98 = 10.5
X
2
= 19.5 × 1.38/2.98 = 9.0 ; X
2
= 9(190/910)
2
= 0.39
(a) R
01
= 2.98
; X
01
= 19.5
(b) X
1
= 10.5
: X
2
= 0.39
Example 32.42. A 230/115 volts, single phase transformer is supplying a load of 5 Amps, at
power factor 0.866 lagging. The no-load current is 0.2 Amps at power factor 0.208 lagging. Calcu-
late the primary current and primary power factor.
(Nagpur University Summer 2000)
Solution. L.V. current of 5 amp is referred to as a
2.5 amp current on the primary (= H.V.) side, at 0.866
lagging p.f. To this, the no load current should be added,
as per the phasor diagram in Fig. 32.50. The phase
angle of the load-current is 30º lagging. The no load
current has a phase angle of 80º lagging. Resultant of
these two currents has to be worked out. Along the
reference, active components are added.
Active components of currents =
2.5 × 0.866 + 0.2 × 0.208
= 2.165 + 0.0416
= 2.2066 amp
Along the perpendicular direction, the reactive components get added up.
Reactive component = 2.5 × 0.5 + 0.2 × 0.9848
= 1.25 + 0.197 = 1.447 amp.
I
1
= 2.2066 j 1.447
φ =tan
1
1.447
2.2066
= 33.25º ; as shown
Tutorial Problems 32.3
1. The S.C. test on a 1-phase transformer, with the primary winding short-circuited and 30 V applied to
the secondary gave a wattmeter reading of 60 W and secondary current of 10 A. If the normal applied
primary voltage is 200, the transformation ratio 1 :2 and the full-load secondary current 10 A, calculate
the secondary terminal p.d. at full-load current for (a) unity power factor (b) power factor 0.8 lagging.
If any approximations are made, they must be explained. [394 V, 377.6 V]
2. A single-phase transformer has a turn ratio of 6, the resistances of the primary and secondary windings
are 0.9 and 0.025 respectively and the leakage reactances of these windings are 5.4 and 0.15
respectively. Determine the voltage to be applied to the low-voltage winding to obtain a current of
100 A in the short-circuited high voltage winding. Ignore the magnetising current. [82 V]
3. Draw the equivalent circuit for a 3000/400-V, I-phase transformer on which the following test results
were obtained. Input to high voltage winding when 1.v. winding is open-circuited : 3000 V, 0.5 A, 500
Fig. 32.50. Phasor diagram for Currents
30°
Ref
2.5 A
I
1
0.2 A
80°
1156
Electrical Technology
W. Input to 1.v. winding when h.v. winding is short-circuited : 11 V, 100 A, 500 W. Insert the
appropriate values of resistance and reactance.
[R
0
= 18,000
, X
0
= 6,360
, R
01
= 2.81
, X
01
= 5.51
] (I.E.E. London)
4. The iron loss in a transformer core at normal flux density was measured at frequencies of 30 and 50
Hz, the results being 30 W and 54 W respectively. Calculate (a) the hysteresis loss and (b) the eddy
current loss at 50 Hz. [44 W, 10 W]
5. An iron core was magnetised by passing an alternating current through a winding on it. The power
required for a certain value of maximum flux density was measured at a number of different frequencies.
Neglecting the effect of resistance of the winding, the power required per kg of iron was 0.8 W at 25
Hz and 2.04 W at 60 Hz. Estimate the power needed per kg when the iron is subject to the same
maximum flux density but the frequency is 100 Hz. [3.63 W]
6. The ratio of turns of a 1-phase transformer is 8, the resistances of the primary and secondary
windings are 0.85 and 0.012 respectively and leakage reactances of these windings are 4.8 and
0.07 respectively. Determine the voltage to be applied to the primary to obtain a current of 150 A
in the secondary circuit when the secondary terminals are short-circuited. Ignore the magnetising
current. [176.4 W]
7. A transformer has no-load losses of 55 W with a primary voltage of 250 V at 50 Hz and 41 W with
a primary voltage of 200 V at 40 Hz. Compute the hysteresis and eddy current losses at a primary
voltage of 300 volts at 60 Hz of the above transformer. Neglect small amount of copper loss at no-
load. [43.5 W ; 27 W] (Elect. Machines AMIE Sec. B. (E-3) Summer 1992)
8. A 20 kVA, 2500/250 V, 50 Hz, 1-phase transformer has the following test results :
O.C. Test (1.v. side) : 250 V, 1.4 A, 105 W
S.C. Test (h.v. side) : 104 V, 8 A, 320 W
Compute the parameters of the approximate equivalent circuit referred to the low voltage side and
draw the circuit. (R
0
= 592.5
; X
0
= 187.2
; R
02
= 1.25
; X
12
= 3
)
(Elect. Machines A.M.I.E. Sec. B Summer 1990)
9. A 10-kVA, 2000/400-V, single-phase transformer has resistances and leakage reactances as follows :
R
1
= 5.2 , X
1
= 12.5 , R
2
= 0.2 , X
2
= 0.5
Determine the value of secondary terminal voltage when the transformer is operating with rated
primary voltage with the secondary current at its rated value with power factor 0.8 lag. The no-load
current can be neglected. Draw the phasor diagram. [376.8 V] (Elect. Machines, A.M.I.E. Sec B, 1989)
10. A 1000-V, 50-Hz supply to a transformer results in 650 W hysteresis loss and 400 W eddy current
loss. If both the applied voltage and frequency are doubled, find the new core losses.
[W
h
= 1300 W ; W
e
= 1600 W] (Elect. Machine, A.M.I.E. Sec. B, 1993)
11. A 50 kVA, 2200/110 V transformer when tested gave the following results :
O.C. test (L.V. side) : 400 W, 10 A, 110 V.
S.C. test (H.V. side) : 808 W, 20.5 A, 90 V.
Compute all the parameters of the equivalent ckt. referred to the H.V. side and draw the resultant ckt.
(Rajiv Gandhi Technical University, Bhopal 2000)
[Shunt branch : R
0
= 12.1 k-ohms, X
m
= 4.724 k-ohms Series branch : r = 1.923 ohms, x = 4.39 ohms]
32.24. Regulation of a Transformer
1. When a transformer is loaded with a constant primary voltage, the secondary voltage
decreases* because of its internal resistance and leakage reactance.
Let
0
V
2
= secondary terminal voltage at no-load.
* Assuming lagging power factor. It will increase if power factor is leading.
Transformer
1157
= E
2
= EK
1
= KV
1
because at no-load the impedance drop is negligible.
V
2
= secondary terminal voltage on full-load.
The change in secondary terminal voltage from no-load to full-load is =
0
V
2
V
2
. This change divided
by
0
V
2
is known as regulation ‘down’. If this change is divided by V
2
, i.e., full-load secondary terminal
voltage, then it is called regulation ‘up’.
% regn ‘down’ =
02 2
02
100
VV
V
×
and % regn ‘up’ =
02 2
2
100
VV
V
×
In further treatment, unless stated otherwise, regulation is to be taken as regulation ‘down’.
We have already seen in Art. 32.16 (Fig. 32.35) that the change in secondary terminal voltage from no-
load to full-load, expressed as a percentage of no-load secondary voltage is,
= ν
r
cos φ ± ν
x
sin φ (approximately)
Or more accurately
=(ν
r
cos φ ± ν
x
sin φ) +
1
200
(ν
x
cos φ " ν
r
sin φ)
2
% regn = ν
r
cos φ ± ν
x
sin φ ...approximately.
The lesser this value, the better the transformer, because a good transformer should keep its
secondary terminal voltage as constant as possible under all conditions of load.
(2) The regulation may also be explained in terms of primary values.
In Fig. 32.51 (a) the approximate equivalent circuit of a transformer is shown and in Fig. 32.51 (b), (c)
and (d) the vector diagrams corresponding to different power factors are shown.
The secondary no-load terminal voltage as referred to primary is E
2
= E
2
/K = E
1
= V
1
and if the
secondary full-load voltage as referred to primary is V
2
(= V
2
/K) then
% regn =
12
1
100
VV
V
×
Fig. 32.51
From the vector diagram, it is clear that if angle between V
1
and V
2
is neglected, then the value of
numerical difference V
1
V
2
is given by (I
1
R
01
cos φ + I
1
X
01
sin φ) for lagging p.f.
% regn =
101 1 01
1
cos sin
100
IR IX
V
φ+ φ
×
= ν
r
cos φ + ν
x
sin φ
where
101
1
100
IR
V
×
= ν
r
and
101
1
100
IX
V
×
= ν
x
...Art. 32.16
As before, if angle between V
1
and V
2
is not negligible, then
% regn = (ν
r
cos φ ± ν
x
sin φ) +
1
200
(ν
x
cos φ " ν
r
sin φ)
2
(3) In the above definitions of regulation, primary voltage was supposed to be kept constant and
the changes in secondary terminal voltage were considered.
1158
Electrical Technology
As the transformer is loaded, the secondary terminal voltage falls (for a lagging p.f.). Hence, to keep
the output voltage constant, the primary voltage must be increased. The rise in primary voltage required to
maintain rated output voltage from no-load to full-load at a given power factor expressed as percentage of
rated primary voltage gives the regulation of the transformer.
Suppose primary voltage has to be raised from its rated value V
1
to V
1
, then
% regn. =
11
1
100
VV
V
×
Example 32.43. A-100 kVA transformer has 400 turns on the primary and 80 turns on the
secondary. The primary and secondary resistances are 0.3
and 0.01
respectively and the
corresponding leakage reactances are 1.1 and 0.035
respectively. The supply voltage is 2200 V.
Calculate (i) equivalent impedance referred to primary and (ii) the voltage regulation and the sec-
ondary terminal voltage for full load having a power factor of 0.8 leading.
(Elect. Machines, A.M.I.E. Sec. B, 1989)
Solution. K = 80/400 = 1/5, R
1
= 0.3 , R
01
= R
1
+ R
2
/K
2
= 0.3 + 0.01/(1/5)
2
= 0.55
X
01
= X
1
+ X
2
/K
2
= 1.1 + 0.035/(1/5)
2
= 1.975
(i) Z
01
= 0.55 + j 1.975 = 2.05 74.44º
(ii) Z
02
= K
2
Z
01
= (1/5)
2
(0.55 + j 1.975) = (0.022 + j 0.079)
No-load secondary voltage = KV
1
= (1/5) × 2200 = 440 V, I
2
= 10 × 10
3
/440 = 227.3 A
Full-load voltage drop as referred to secondary
= I
2
(R
02
cos φ − X
02
sin φ)
= 227.3 (0.022 × 0.8 0.079 × 0.6 ) = 6.77 V
% regn. = 6.77 × 100/440 = 1.54
Secondary terminal voltage on load = 440 (6.77) = 446.77 V
Example 32.44. The corrected instrument readings obtained from open and short-circuit tests
on 10-kVA, 450/120-V, 50-Hz transformer are :
O.C. test : V
1
= 120 V; I
1
= 4.2 A; W
1
= 80 W; V
1
, W
1
and I
1
were read on the low-voltage side.
S.C. test : V
1
= 9.65 V; I
1
= 22.2 A ; W
1
= 120 W with low-voltage winding short-circuited
Compute :
(i) the equivalent circuit (approximate) constants,
(ii) efficiency and voltage regulation for an 80% lagging p.f. load,
(iii) the efficiency at half full-load and 80% lagging p.f. load.
(Electrical Engineering-I, Bombay Univ. 1988)
Soluion. It is seen from the O.C. test, that with primary open, the secondary draws a no-load current
of 4.2 A. Since K = 120/450 = 4/15, the corresponding no-load primary current I
0
= 4.2 × 4/15 = 1.12 A.
(i) Now, V
1
I
0
cos φ
0
=80 cos φ
0
= 80/450 × 1.12 = 0.159
∴φ
0
= cos
1
(0.159) = 80.9°; sin φ
0
= 0.987
I
w
= I
0
cos φ
0
= 1.12 × 0.159 = 0.178A and I
µ
= 1.12 × 0.987 = 1.1 A
R
0
= 450/0.178 = 2530
and X
0
= 450/1.1 = 409
During S.C. test, instruments have been placed in primary.
Z
01
= 9.65/22.2 = 0.435
R
01
= 120/22.2
2
= 0.243
X
01
=
22
0.435 0.243 0.361
−=
The equivalent circuit is shown in Fig. 32.52.
(ii) Total approximate voltage drop as referred to primary is I
1
(R
01
cos φ + X
01
sin φ).
* Assuming φ
1
= φ
2
= cos
1
(0.8).
Transformer
1159
Fig. 32.52
Now, full-load I
1
= 10,000/450 = 22.2 A
Drop = 22.2 (0.243 × 0.8 + 0.361 × 0.6) = 9.2 V
Regulation = 9.2 × 100/450 = 2.04%
F.L. losses = 80 + 120 = 200 W;
F.L. output = 10,000 × 0.8 = 8000 W
η = 8000/8200 = 0.9757 or 97.57%
(iii) Half-load
Iron loss = 80 W; Cu loss = (1/2)
2
× 120 = 30 W
Total losses = 110 W; Output = 5000 × 0.8 = 4000 W
∴η= 4000/4110 = 0.9734 or 97.34%
Example 32.45. Consider a 20 kVA, 2200/220 V, 50 Hz transformer. The O.C./S.C. test results
are as follows :
O.C. test : 220 V, 4.2 A, 148 W (1.v. side)
S.C. test : 86 V, 10.5 A, 360 W (h.v. side)
Determine the regulation at 0.8 p.f. lagging and at full load. What is the p.f. on short-circuit ?
(Elect. Machines Nagpur Univ. 1993)
Solution. It may be noted that O.C. data is not required in this question for finding the regulation.
Since during S.C. test instruments have been placed on the h.v. side i.e. primary side.
Z
01
= 86/10.5 = 8.19 ; R
01
= 360/10.5
2
= 3.26
X
01
=
22
8.19 3.26 7.5
−=
F.L. primary current, I
1
= 20,000/2200 = 9.09 A
Total voltage drop as referred to primary = I
1
(R
01
cos φ + X
01
sin φ)
Drop = 9.09 (3.26 × 0.8 + 7.5 × 0.6) = 64.6 V
% age regn. = 64.6 × 100/2200 = 2.9%, p.f. on short-circuit = R
01
/Z
01
= 3.26/8.19 = 0.4 lag
Example 32.46. A short-circuit test when performed on the h.v. side of a 10 kVA, 2000/400 V
single phase transformer, gave the following data ; 60 V, 4 A, 100 W.
If the 1.v. side is delivering full load current at 0.8 p.f. lag and at 400 V, find the voltage applied
to h.v. side. (Elect. Machines-I, Nagpur Univ. 1993)
Solution. Here, the test has been performed on the h.v. side i.e. primary side.
Z
01
= 60/4 = 15 ; R
01
= 100/4
2
= 6.25 ; X
01
=
22
15 6.25
= 13.63
F. L . I
1
= 10,000/2000 = 5A
Total transformer voltage drop as referred to primary is
I
1
(R
01
cos φ + X
01
sin φ) = 5 (6.25 × 0.8 + 13.63 × 0.6) = 67 V
Hence, primary voltage has to be raised from 2000 V to 2067 V in order to compensate for the total
voltage drop in the transformer. In that case secondary voltage on load would remain the same as on
no-load.
Example 32.47. A 250/500-V transformer gave the following test results :
Short-circuit test : with low-voltage winding short-circuited :
20 V; 12 A, 100 W
Open-circuit test : 250 V, 1 A, 80 W on low-voltage side.
Determine the circuit constants, insert these on the equivalent circuit diagram and calculate
applied voltage and efficiency when the output is 10 A at 500 volt and 0.8 power factor lagging.
(Elect. Machines, Nagpur Univ. 1993)
1160
Electrical Technology
Solution. Open-circuit Test :
V
1
I
0
cos φ
0
=80 cos φ
0
= 80/250 × I = 0.32
I
w
= I
0
cos φ
0
= I × 0.32 = 0.32 A, I
µ
=
22
(1 0 . 32 ) 0 . 95
−=
A
R
0
= V
1
/I
w
= 250/0.32 = 781.3 , X
0
= V
1
/ I
µ
= 250/0.95 = 263.8
The circuit is shown in Fig. 32.53 (a).
Short-circuit Test :
As the primary is short-circuited, all values refer to secondary winding.
Fig. 32.53 (a) Fig. 32.53 (b)
R
02
=
2
short-circuit power
100
F.L. secondary current
12
=
= 0.694
Z
02
= 20/12 = 1.667 ; X
02
=
22
(1.667 0.694 )
= 1.518
As R
0
and X
0
refer to primary, hence we will transfer these values to primary with the help of
transformation ratio.
K = 500/250 = 2 R
01
= R
02
/K
2
= 0.694/4 = 0.174
X
01
= X
02
/K
2
= 1.518/4 = 0.38 ; Z
01
= Z
02
/K
2
= 1.667/4 = 0.417
The equivalent circuit is shown in Fig. 32.53 (a).
Efficiency
Total Cu loss = I
2
2
R
02
= 100 × 0.694 = 69.4 W ; Iron loss = 80 W
Total loss = 69.4 + 80 = 149.4 W ∴η =
5000 0.8 100
4000 149.4
××
+
= 96.42%
The applied voltage V
1
is the vector sum of V
1
and I
1
Z
01
as shown in Fig. 32.53 (b).
I
1
= 20 A ; I
1
R
01
= 20 × 0.174 = 3.84 V ; I
1
X
01
= 20 × 0.38 = 7.6 V
Neglecting the angle between V
1
and V
1
, we have
V
1
2
= OC
2
= ON
2
+ NC
2
= (OM + MN)
2
+ (NB + BC)
2
= (250 × 0.8 + 3.48)
2
+ (250 × 0.6 + 7.6)
2
V
1
2
= 203.5
2
+ 157.6
2
V
1
= 257.4 V
Example 32.48. A 230/230 V, 3 kVA transformer gave the following results :
O.C. Test : 230 V, 2 amp, 100 W
S.C. Test : 15 V, 13 amp, 120 W
Determine the regulation and efficiency at full load 0.80 p.f. lagging.
(Sambalpur University, 1998)
Solution. This is the case of a transformer with turns ratio as 1 : 1. Such a transformer is mainly
required for isolation.
Transformer
1161
Rated Current =
3000
230
= 13 amp
Cu-losses at rated load = 120 watts, from S.C. test
Core losses = 100 watts, from O.C. test
At full load, VA output = 3000
At 0.8 lag p.f., Power output = 3000 × 0.8 = 2400 watts
Required efficiency =
2400
100%
2400 220
×
+
= 91.6%
From S.C. test, Z =
15
13
= 1.154 ohms
R =
120
15 15
×
= 0.53 ohm, X =
22
1.154 0.53
= 1.0251 ohm
Approximate voltage regulation
= IR cos φ + IX sin φ = 13[0.53 × 0.8 + 1.0251 × 0.6]
= 13[0.424 + 0.615] = 13.51 volts
In terms of %, the voltage regulation =
13.51
100%
230
×
= 5.874%
Example 32.49. A 10 kVA, 500/250 V, single-phase transformer has its maximum efficiency of
94% when delivering 90% of its rated output at unity p.f. Estimate its efficiency when delivering its
full-load output at p.f. of 0.8 lagging. (Nagpur University, November 1998)
Solution. Rated output at unity p.f. = 10000 W. Hence, 90% of rated output = 9,000 W
Input with 94% efficiency = 9000/0.94 W
Losses = 9000((1/0.94) 1) = 574 W
At maximum efficiency, variable copper-loss = constant = Core loss = 574/2 = 287 W
At rated current, Let the copper-loss = P
c
watts
At 90% load with unity p.f., the copper-loss is expressed as 0.90
2
× P
c
.
Hence, P
c
= 287/0.81 = 354 W
(b) Output at full-load, 0.8 lag p.f. = 10,000 × 0.80 = 8000 W
At the corresponding load, Full Load copper-loss = 354 W
Hence, efficiency = 8000/(8000 + 354 + 287) = 0.926 = 92.6%
Example 32.50. Resistances and Leakage reactance of 10 kVA, 50 Hz, 2300/230 V single phase
distribution transformer are r
1
= 3.96 ohms, r
2
= 0.0396 ohms, x
1
= 15.8 ohms, x
2
= 0.158 ohm.
Subscript 1 refers to HV and 2 to LV winding (a) transformer delivers rated kVA at 0.8 p.f. Lagging
to a load on the L.V. side. Find the H.V. side voltage necessary to maintain 230 V across Load-
terminals. Also find percentage voltage regulation. (b) Find the power-factor of the rated load-
current at which the voltage regulation will be zero, hence find the H.V. side voltage.
(Nagpur University, November 1997)
Solution. (a) Rated current on L.V. side = 10,000/230 = 43.5 A. Let the total resistance and total
leakage reactance be referred to L.V. side. Finally, the required H.V. side voltage can be worked out
after transformation.
Total resistance, r = r
1
+ r
2
= 3.96 × (230/2300)
2
+ 0.0396
= 0.0792 ohms
Total leakage-reactance, x = x
1
+ x
2
= 15.8 × (230/2300)
2
+ 0.158
= 0.316 ohm
1162
Electrical Technology
For purpose of calculation of voltage-magnitudes, approximate formula for voltage regulation can be
used. For the present case of 0.8 lagging p.f.
V
1
= V
2
+ I [r cos φ + x sin φ]
= 230 + 43.5 [(0.0792 × 0.8) + (0.316 × 0.6)]
= 230 + 43.5 [0.0634 + 0.1896] = 230 + 11 = 241 volts
Hence, V
1
= 241 × (2300/230) = 2410 volts.
It means that H.V. side terminal voltage must be 2410 for keeping 230 V at the specified load.
(b) Approximate formula for voltage regulation is : V
1
V
2
= I [r cos φ ± x sin φ]
With Lagging p.f., +ve sign is retained. With leading power-factor, the ve sign is applicable. For the
voltage-regulation to be zero, only leading P.f. condition can prevail.
Thus, r cos φ x sin φ =0
or tan φ = r/x = 0.0792/0.316 = 0.25
or φ = 14º, cos φ = 0.97 leading
Corresponding sin φ =sin 14º = 0.243
H.V. terminal voltage required is 2300 V to maintain 230 V at Load, since Zero regulation condition is
under discussion.
Example 32.51. A 5 kVA, 2200/220 V, single-phase transformer has the following parameters.
H.V. side : r
1
= 3.4 ohms, x
1
= 7.2 ohms
L.V. side : r
2
= 0.028 ohms, x
2
= 0.060 ohms
Transformer is made to deliver rated current at 0.8 lagging P.f. to a load connected on the L.V.
side. If the load voltage is 220 V, calculate the terminal voltage on H.V. side
(Neglect the exciting current). (Rajiv Gandhi Technical University, Bhopal, Summer 2001)
Solution. Calculations may be done referring all the parameters the L.V. side first. Finally, the
voltage required on H.V. side can be obtained after transformation.
Rated current ref. to L.V. side = 5000/220 = 22.73 A
Total winding resistance ref. to L.V. side = r
1
+ r
2
= (220/2200)
2
× 3.4 + 0.028
Total winding-leakage-reactance ref. to L.V. side = x
1
+ x
2
= (220/2200)
2
× 7.2 + 0.060 = 0.132 ohm
G
B
C
D
F
A
I
O
V
1
V
2
B
Fig. 32.53(c)
In the phasor diagram of Fig. 32.53 (c).
QA = V
2
= 220 volts, I = 22.73 A at lagging phase angle of 36.87º
AB = Ir, AD = Ir cos φ = 22.73 × 0.062 × 0.80 = 1.127 V
DC = Ix sin φ = 22.73 × 0.132 × 0.60 = 1.80 V
Transformer
1163
OC = 220 + 1.127 + 1.80 = 222.93 volts
BD = Ir sin φ = 0.85 V
BF = x cos φ = 2.40 V
CF = 2.40 0.85 = 1.55 V
V
1
= OF = (222.93
2
+ 1.55
2
)
0.50
= 222.935 volts
Required terminal voltage of H.V. side = V
1
= 222.935 × (2200/220) = 2229.35 volts
[Note. In approximate and fast calculations, CF is often ignored for calculation of magnitude of V
1
. The
concerned expression is : V
1
= V
2
+ Ir cos φ + Ix sin φ, for lagging P.f.]
Example 32.52. A 4-kVA, 200/400 V, single-phase transformer takes 0.7 amp and 65 W on Open-
circuit. When the low-voltage winding is short-circuited and 15 V is applied to the high-voltage
terminals, the current and power are 10 A and 75 W respectively. Calculate the full-load efficiency
at unity power factor and full-load regulation at 0.80 power-factor lagging.
(Nagpur University April 1999)
Solution. At a load of 4 kVA, the rated currents are :
L.V. side : 4000/200 = 20 amp
And H.V. side : 4000/400 = 10 amp
From the test data, full-load copper-loss = 75 W
And constant core-loss = 65 W
From S.C. test, Z = 15/10 = 1.5 ohms
R = 75/100 = 0.75 ohm
Hence x =
22
1.5 0.75
= 1.30 ohms
All these series-parameters are referred to the H.V. side, since the S.C. test has been conducted
from H.V. side.
Full-load efficiency at unity p.f. = 4000 / (4000 + 65 + 75)
= 0.966 = 96.6%
Full load voltage regulation at 0.80 lagging p.f.
= Ir cos φ + I x sin φ
= 10 (0.75 × 0.80 + 1.30 × 0.60) = 16.14 Volts
Thus, due to loading, H.V. side voltage will drop by 16.14 volts (i.e. terminal voltage for the load
will be 383.86 volts), when L.V. side is energized by 200-V source.
32.25. Percentage Resistance, Reactance and Impedance
These quantities are usually measured by the voltage drop at full-load current expressed as a
percentage of the normal voltage of the winding on which calculations are made.
(i) Percentage resistance at full-load
% R =
2
101 1 01
111
100 100
IR I R
VVI
×= ×
=
2
202
22
100
IR
VI
×
= % Cu loss at full-load
% R = % Cu loss = ν
r
...Art.32.16
(ii) Percentage reactance at full-load
% X =
101 202
12
100 100
IX IX
VV
×= ×
= ν
x
1164
Electrical Technology
(iii) Percentage impedance at full-load
% Z =
101 202
12
100 100
IZ IZ
VV
×= ×
(iv) %Z =
22
(% % )
RX
+
It should be noted from above that the reactances and resistances in ohm can be obtained thus :
R
01
=
11
11
%%Cu loss
100 100
RV V
II
××
=
××
; Similarly R
02
=
22
22
%%Cu loss
100 100
RV V
II
××
=
××
X
01
=
11
11
%
100 100
z
XV V
II
×ν×
=
××
; Similarly X
02
=
22
22
%
100 100
x
V
XV
II
ν×
×
=
××
It may be noted that percentage resistance, reactance and impedance have the same value whether
referred to primary or secondary.
Example 32.53. A 3300/230 V, 50-kVA, transformer is found to have impedance of 4% and a Cu
loss of 1.8% at full-load. Find its percentage reactance and also the ohmic values of resistance,
reactance and impedance as referred to primary. What would be the value of primary short-circuit
current if primary voltage is assumed constant ?
Solution. % X =
22 22
(% % ) (4 1.8 ) 3.57%
ZR−==
(Cu loss = % R)
Full load I
1
= 50,000/3300 = 15.2 A (assuming 100% efficiency). Considering primary winding, we
have
% R =
01 1
01
100
1.8 3300
1.8
100 15.2
L
RI
R
V
×
×
=∴= =
×
3.91
Similarly % X =
01 1
01
1
100
3.57 3300
3.57
100 15.2
XI
X
V
×
×
=∴= =
×
7.76
Similarly Z
01
=
43300
100 15.2
×
=
×
8.7
Now
Short-circuit current
Full load current
=
100
S.C. current = 15.2 25 =
4
∴×
380 A
Example 32.54. A 20-kVA, 2200/220-V, 50-Hz distribution transformer is tested for efficiency
and regulation as follows :
O.C. test : 220 V 4.2 A, 148 W –l.v side
S.C. test : 86 V 10.5 A, 360 W – l.v. side
Determine (a) core loss (b) equivalent resistance referred to primary (c) equivalent resistance
referred to secondary (d) equivalent reactance referred to primary (e) equivalent reactance referred
to secondary (f) regulation of transformer at 0.8 p.f. lagging current (g) efficiency at full-load and
half the full-load at 0.8 p.f. lagging current.
Solution. (a) As shown in Art 32.9, no-load primary input is practically equal to the core loss.
Hence, core loss as found from no-load test, is 148 W.
(b) From S.C. test, R
01
= 360/10.5
2
= 3.26
(c) R
02
= K
2
R
01
= (220/2200)
2
× 3.26 = 0.0326
* Short circuit I
SC
=
1
01
V
Z
Now, Z
01
=
1
1
%
100
VZ
I
×
×
I
SC
=
11 1
11
100 100
100
%% %
SC
I
VI I
VZ Z I Z
×× ×
=∴=
×
*
Transformer
1165
(d) Z
10
=
86
8.19
10.5
SC
SC
V
I
==
X
01
=
22
(8.19 3.26 )
−=
7.51
(e) X
02
= K
2
X
01
= (220/2200)
2
× 7.51 = 0.0751
(f ) We will use the definition of regulation as given in Art. 32.24 (3).
We will find the rise in primary voltage necessary to maintain the output terminal voltage constant from
no-load to full-load.
Rated primary current = 20,000/2200 = 9.1 A
V
1
=
22
[2200 0.8 9.1 3.26) (2200 0.6 9.1 7.51) ]
×+× + ×+× =
2265 V
% regn =
2265 2200
100
2200
×=
2.95%
We would get the same result by working in the secondary. Rated secondary current = 91 A.
0
V
2
=
22
(220 0.8 91 0.0326) (220 0.6 91 0.0751) ] 226.5
V×+× + ×+× =
% regns. =
226.5 220
100
220
×=
2.95%
(g) Core loss = 1.48 W. It will be the same for all loads.
Cu loss at full load = I
1
2
R
01
= 9.1
2
× 3.26 = 270 W
Cu loss at half full-load = 4.55
2
× 3.26 = 67.5 W (or F.L. Cu loss/4)
∴η at full-load =
22,000 0.8 100
20,000 0.8 148 270
××
=
×+ +
97.4%
∴η at half-load =
10,000 0.8 100
10,000 0.8 148 67.5
××
=
×+ +
97.3%
Example 32.55. Calculate the regulation of a transformer in which the ohmic loss is 1% of the
output and the reactance drop is 5% of the voltage, when the power factor is (i) 0.80 Lag (ii) unity
(iii) 0.80 Leading. (Madras University, 1997)
Solution. When 1% of output is the ohmic loss, p.u. resistance of the transformer, ε
r
= 0.01
When 5% is the reactance drop, p.u. reactance of the transformer ε
x
= 0.05
(i) Per Unit regulation of the transformer at full-load, 0.8 Lagging p.f.
= 0.01 × cos φ + 0.05 × sin φ = 0.01 × 0.8 + 0.05 × 0.06 = 0.038 or 3.8%
(ii) Per Unit regulation at unity p.f. = 0.01 × 1 = 0.01 or 1%
(iii) Per Unit regulation at 0.08 Leading p.f. = 0.01 × 0.8 0.05 × 0.6 = 0.022 or 2.2%
Example 32.56. The maximum efficiency of a 500 kVA, 3300/500 V, 50 Hz, single phase
transformer is 97% and occurs at 3/4
th
full-load u.p.f. If the impedance is 10% calculate the
regulation at fullload, 0.8 p.f. Lag. (Madurai Kamraj University, November 1997)
Solution. At unity p.f. with 3/4
th
full load, the output of the transformer
= 500 × 0.75 × 1 kW = 375 kW
0.97 =
375
375 2
i
P
+
where P
i
= core loss in kW, at rated voltage.
At maximum efficiency, x
2
P
c
= P
i
(0.75)
2
P
c
= P
i
where x = 0.75, i.e. 3/4
th
which is the fractional loading of the transformer
1166
Electrical Technology
P
c
= copper losses in kW, at rated current
P
i
=
()
{}
11 3
(375) 1 ½ 375 5.8 kW
20.97 97
×−=××=
P
c
= 5.8/(0.75)
2
= 10.3 kW
Full load current in primary (H.V.) winding =
500 1000
3300
×
=
151.5 amp
Total winding resistance ref. to primary
=
2
10.3 1000
0.44876 ohm
(151.5)
×
=
ε
r
= % resistance =
151.5 0.44876
100 %
3300
×
×=
2.06%
ε
z
= % Impedance = 10%
ε
x
= % reactance =
100 4.244
−=
9.7855%
By Approximate formula at 0.8 p.f. lag
% regulation = ε
r
cos φ + ε
x
sin φ
= 2.06 × 0.8 + 9.7855 × 0.6
= 1.648 + 5.87 = 7.52 %
Example 32.57. A transformer has copper-loss of 1.5% and reactance-drop of 3.5% when tested
at full-load. Calculate its full-load regulation at (i) u.p.f. (ii) 0.8 p.f. Lagging and (iii) 0.8 p.f.
Leading. (Bharathithasan Univ. April 1997)
Solution. The test-data at full-load gives following parameters :
p.u. resistance = 0.015, p.u. reactance = 0.035
(i) Approximate Voltage Regulation at unity p.f. full load
= 0.015 cos φ + 0.035 sin φ
= 0.015 per unit = 1.5%
(ii) Approximate Voltage Regulation at 0.80 Lagging p.f.
= (0.015 × 0.8) + (0.035 × 0.6) = 0.033 per unit = 3.3%
(iii) Approximate Voltage Regulation at 0.8 leading p.f.
= I
r
cos φ − I
x
sin φ
= (0.015 × 0.8) (0.035 × 0.6) = 0.009 per unit =
0.9%
32.26. Kapp Regulation Diagram
It has been shown that secondary terminal voltage falls as the load on the transformer is increased
when p.f. is lagging and it increases when the power factor is leading. In other words, secondary terminal
voltage not only depends on the load but on power factor also (Art. 32.16). For finding the voltage
drop (or rise) which is further used in determining the regulation of the transformer, a graphical
construction is employed which was proposed by late Dr. Kapp.
For drawing Kapp regulation diagram, it is necessary to know the equivalent resistance and
reactance as referred to secondary i.e. R
02
and X
02
. If I
2
is the secondary load current, then secondary
terminal voltage on load V
2
, is obtained by subtracting I
2
R
02
and I
2
X
02
voltage drops vectorially from
secondary no-load voltage
0
V
2
.
Now,
0
V
2
is constant, hence it can be represented by a circle of constant radius OA as in Fig. 32.54.
This circle is known as no-load or open-circuit e.m.f. circle. For a given load, OI
2
represents the load
current and is taken as the reference vector, CB represents I
2
R
02
and is parallel to OI
2
, AB represents I
2
X
02
and is drawn at right angles to CB. Vector OC obviously represents I
2
X
02
and is drawn at right
Transformer
1167
angles to CB. Vector OC obviously rep-
resents secondary terminal voltage V
2
.
Since I
2
is constant, the drop triangle
ABC remains constant in size. It is seen
that end point C of V
2
lies on another
circle whose centre is O. This point O
lies at a distance of I
2
X
02
vertically be-
low the point O and a distance of I
2
R
02
to its left as shown in Fig. 32.54.
Suppose it is required to find the
voltage drop on full-load at a lagging
power factor of cos φ, then a radius OLP
is drawn inclined at an angle of φ with
OX. LM = I
2
R
02
and is drawn horizon-
tal MN = I
2
X
02
and is drawn perpen-
dicular to LM. Obviously, ON is no-
load voltage
0
V
2
. Now, ON = OP =
0
V
2
. Similarly, OL is V
2
. The voltage
drop = OP OL = LP.
Hence, percentage regulation ‘down’ is =
100 100
OP OL
LP
OP OP
×=×
It is seen that for finding voltage drop, triangle LMN need not be drawn, but simply the radius OLP.
The diagram shows clearly how the secondary terminal voltage falls as the angle of lag
increases. Conversely, for a leading power factor, the fall in secondary terminal voltage decreases till for an
angle of φ
0
leading, the fall becomes zero; hence V
2
=
0
V
2
. For angles greater than φ
0
, the secondary
terminal voltage V
2
becomes greater than
0
V
2
.
The Kapp diagram is very helpful in determining the variation of regulation with power factor but it has
the disadvantage that since the lengths of the sides of the impedance triangle are very small as compared to
the radii of the circles, the diagram has to be drawn on a very large scale, if sufficiently accurate results are
desired.
32.27. Sumpner of Back-to-Back Test
This test provides data for finding the
regulation, efficiency and heating under load
conditions and is employed only when two
similar transformers are available. One
transformer is loaded on the other and both
are connected to supply. The power taken
from the supply is that necessary for supplying
the losses of both transformers and the
negligibly small loss in the control circuit.
As shown in Fig. 32.55, primaries of the
two transformers are connected in parallel
across the same a.c. supply. With switch S
open, the wattmeter W
1
reads the core loss
for the two transformers.
Fig. 32.55
Fig. 32.54
I
1
I
2
I
2
I
1
W
1
W
2
V
M
L
J
E
H
D
A
S
A
T
Supply
F
B
G
C
1168
Electrical Technology
The secondaries are so connected that their potentials are in opposition to each other. This would
so if V
AB
= V
CD
and A is joined to C whilst B is joined to D. In that case, there would be no secondary
current flowing around the loop formed by the two secondaries. T is an auxiliary low-voltage trans-
former which can be adjusted to give a variable voltage and hence current in the secondary loop
circuit. By proper adjustment of T, full-load secondary current I
2
can be made to flow as shown. It is
seen, that I
2
flows from D to C and then from A to B. Flow of I
1
is confined to the loop FEJLGHMF
and it does not pass through W
1
. Hence, W
1
continues to read the core loss and W
2
measures full-load
Cu loss (or at any other load current value I
2
). Obviously, the power taken in is twice the losses of a
single transformer.
Example 32.58. Two similar 250-kVA, single-phase transformers gave the following results
when tested by back-to-back method :
Mains wattmeter, W
1
= 5.0 kW
Primary series circuit wattmeter, W
2
= 7.5 kW (at full-load current).
Find out the individual transformer efficiencies at 75% full-load and 0.8 p.f. lead.
(Electrical Machines-III, Gujarat Univ. 1986)
Solution. Total losses for both transformers = 5 + 7.5 = 12.5 kW
F.L. loss for each transformer = 12.5/2 = 6.25 kW
Copper-loss at 75% load =
()
2
37.5
kW 2.11 kW
42
×=
Output of each transformer at 75% F.L. and 0.8 p.f. = (250 × 0.75) × 0.8 = 150 kW
η =
150
150 2.5 2.11
=
++
97%
32.28. Losses in a Transformer
In a static
transformer, there
are no friction or
windage losses.
Hence, the only
losses occuring
are :
(i) Core
or Iron Loss : It
includes both
hysteresis loss
and eddy cur-
rent loss. Be-
cause the core
flux in a trans-
former remains
practically con-
stant for all loads
(its variation being
1 to 3% from no-load to full-load). The core loss is practically the same at all loads.
Hysteresis loss W
h
= η B
1.6
max
f V
watt; eddy current loss W
e
= PB
2
max
f
2
t
2
watt
These losses are minimized by using steel of high silicon content for the core and by using very thin
Typical 75kVA Transformer Losses vs. Load
3,000
2,500
2,000
1,500
1,000
500
Total Loses (Watts)
0
Load (%)
0 5 10 15 20 25 30 35 40 45 50 55 60 65 70 75 80 85 90 95 100
150
0
Rise
110
0
Rise
80
0
Rise
TP-1 150
0
C Rise
Transformer
1169
laminations. Iron or core loss is found from the O.C. test. The input of the transformer when on no-
load measures the core loss.
(ii) Copper loss. This loss is due to the ohmic resistance of the transformer windings. Total Cu loss
= I
1
2
R
1
+ I
2
2
R
2
= I
1
2
R
01
+ I
2
2
R
02
. It is clear that Cu loss is proportional to (current)
2
or kVA
2
. In other
words, Cu loss at half the full-load is one-fourth of that at full-load.
The value of Cu loss is found from the short-circuit test (Art. 32.22).
32.29. Efficiency of a Transformer
As is the case with other types of electrical machines, the efficiency of a transformer at a particular load
and power factor is defined as the output divided by the input–the two being measured in the same units
(either watts or kilowatts).
Efficiency =
Output
Input
But a transformer being a highly efficient piece of equipment, has very small loss, hence it is impractical
to try to measure transformer, efficiency by measuring input and output. These quantities are nearly of the
same size. A better method is to determine the losses and then to calculate the efficiency from ;
Efficiency =
Output Output
Output + losses Output + Cu loss + iron loss
=
or η =
Input Losses
losses
1
Input Input
=−
It may be noted here that efficiency is based on power output in watts and not in volt-amperes, al-
though losses are proportional to VA. Hence, at any volt-ampere load, the efficiency depends on power
factor, being maximum at a power factor of unity.
Efficiency can be computed by determining core loss from no-load or open-circuit test and Cu loss
from the short-circuit test.
32.30. Condition for Maximum Efficiency
Cu loss = I
1
2
R
01
or I
2
2
R
02
= W
cu
Iron loss = Hysteresis loss + Eddy current loss = W
h
+ W
e
= W
i
Considering primary side,
Primary input = V
1
I
1
cos φ
1
η =
2
11 1 1 01
11 1
11 1 11 1
coscos losses
cos cos
i
VI I R WVI
VI VI
φ− φ−
=
φφ
=1
101
11111
cos cos
i
IR W
VVI
φφ
Differentiating both sides with respect to I
1
, we get
1
d
dI
η
=
01
2
11
11 1
0
cos
cos
i
RW
V
VI
−+
φ
φ
For η to be maximum,
1
d
dI
η
= 0. Hence, the above equation becomes
01
11
cos
R
V
φ
=
2
11 1
cos
i
W
VI
φ
or W
i
= I
1
2
R
01
or I
2
2
R
02
or Cu loss = Iron loss
1170
Electrical Technology
Fig. 32.56
The output current corresponding to maximum efficiency is I
2
=
02
(/ )
i
WR
.
It is this value of the output current which will make the Cu loss equal to the iron loss. By proper
design, it is possible to make the maximum efficiency occur at any desired load.
Note. (i) If we are given iron loss and full-
load Cu loss, then the load at which two losses
would be equal (i.e. corresponding to maximum
efficiency) is given by
= Full load ×
()
Iron loss
F.L. Cu loss
In Fig. 32.56, Cu losses are plotted as a
percentage of power input and the efficiency
curve as deduced from these is also shown. It is
obvious that the point of intersection of the Cu
and iron loss curves gives the point of maximum
efficiency. It would be seen that the efficiency
is high and is practically constant from 15%
full-load to 25% overload.
(ii) The efficiency at any load is given by
η =
full-load kVA p.f.
100
( full-load kVA p.f.) +
x
xWW
cu
i
××
×
××+
where x = ratio of actual to full-load kVA
W
i
= iron loss in kW ; W
cu
= Cu loss in kW.
Example 32.59. In a 25-kVA, 2000/200 V, single-phase transformer, the iron and full-load cop-
per losses are 350 and 400 W respectively. Calculate the efficiency at unity power factor on
(i) full load (ii) half full-load. (Elect. Engg. & Electronic, Bangalore Univ. 1990 and
Similar example in U.P. Technical University 2001)
Solution. (i) Full-load Unity p.f.
Total loss = 350 + 400 = 750 W
F.L. output at u.p.f. = 25 × 1 = 25 kW ; η = 25/25.75 = 0.97 or 97%
(ii) Half F.L. Unity p.f.
Cu loss = 400 × (1/2)
2
= 100 W. Iron loss remains constant at 350 W, Total loss = 100 + 350
= 450 W.
Half-load output at u.p.f. = 12.5 kW
∴η= 12.5/(12.5 + 0.45) = 96.52%
Example 32.60. If P
1
and P
2
be the iron and copper losses of a transformer on full-load, find the
ratio of P
1
and P
2
such that maximum efficiency occurs at 75% full-load.
(Elect. Machines AMIE Sec. B, Summer 1992)
Solution. If P
2
is the Cu loss at full-load, its value at 75% of full-load is = P
2
× (0.75)
2
= 9P
2
/16. At
maximum efficiency, it equals the iron loss P
1
which remains constant throughout. Hence, at maximum
efficiency.
P
1
=9P
2
/16 or P
1
/P
2
= 9/16.
Example 32.61. A 11000/230 V, 150-kVA, 1-phase, 50-Hz transformer has core loss of 1.4 kW
and F.L. Cu loss of 1.6 kW. Determine
(i) the kVA load for max. efficiency and value of max. efficiency at unity p.f.
(ii) the efficiency at half F.L. 0.8 p.f. leading (Basic Elect. Machine, Nagpur Univ. 1993)
Transformer
1171
Solution. (i) Load kVA corresponding to maximum efficiency is
= F.L. kVA ×
Iron loss 1.6
250
F.L. Cu loss 1.4
= 160 kVA
Since Cu loss equals iron loss at maximum efficiency, total loss = 1.4 + 1.4 = 2.8 kW ;
output = 160 × 1 = 160 kW
η
max
= 160/162.8 = 0.982 or 98.2%
(ii) Cu loss at half full-load = 1.6 × (1/2)
2
= 0.4 kW ; Total loss = 1.4 + 0.4 = 1.8 kW
Half F.L. output at 0.8 p.f.= (150/2) × 0.8 = 60 kW
Efficiency = 60/(60 + 1.8) = 0.97 or 97%
Example 32.62. A 5-kVA, 2,300/230-V, 50-Hz transformer was tested for the iron losses with
normal excitation and Cu losses at full-load and these were found to be 40 W and 112 W respectively.
Calculate the efficiencies of the transformer at 0.8 power factor for the following kVA outputs :
1.25 2.5 3.75 5.0 6.25 7.5
Plot efficiency vs kVA output curve. (Elect. Engg. -I, Bombay Univ. 1987)
Solution. F.L. Cu loss = 112 W ; Iron loss = 40 W
(i) Cu loss at 1.25 kVA = 112 × (1.25/5)
2
= 7 W
Total loss = 40 + 7 = 47 W Output = 1.25 × 0.8 = 1 kW = 1,000 W
η = 100 × 1,000/1,047 = 95.51 %
(ii) Cu loss at 2.5 kVA = 112 × (2.5/5)
2
= 28 W
Total loss = 40 + 28 = 68 W
Output = 2.5 × 0.8 = 2 kW
η = 2,000 × 100/2,068 = 96.71%
(iii) Cu loss at 3.75 kVA
= 112 × (3.75/5)
2
= 63 W
Total loss = 40 + 63 = 103 W
η = 3,000 × 100/3,103 = 96.68 %
(iv) Cu loss at 5 kVA
= 112 W
Total loss = 152 W = 0.152 kW
Output = 5 × 0.8 = 4 kW
η =4 × 100/4.142 = 96.34 %
(v) Cu loss at 6.25 kVA
= 112 × (6.25/5)
2
= 175 W
Total loss = 125 W = 0.125 kW ; Output = 6.25 × 0.8 = 5 kW
η =5 × 100/5.215 = 95.88 %
(vi) Cu loss at 7.5 kVA = 112 × (7.5/5)
2
= 252 W
Total loss = 292 W = 0.292 kW ; Output = 7.5 × 0.8 = 6 kW
η =6 × 100/6.292 = 95.36 %
The curve is shown in Fig. 32.57.
Example 32.63. A 200-kVA transformer has an efficiency of 98% at full load. If the max.
efficiency occurs at three quarters of full-load, calculate the efficiency at half load. Assume negli-
gible magnetizing current and p.f. 0.8 at all loads. (Elect. Technology Punjab Univ. Jan. 1991)
Fig. 32.57
98
97
96
95
94
93
92
1.25 2.5 3.75 5 6.25 7.5
Efficiency
Load, kVA
1172
Electrical Technology
Solution. As given, the transformer has a F.L. efficiency of 98 % at 0.8 p.f.
F.L. output = 200 × 0.8 = 160 kW ; F.L. input = 160/0.98 = 163.265 kW
F.L. losses = 163.265 160 = 3.265 kW
This loss consists of F.L. Cu loss x and iron loss y.
x + y = 3.265 kW ...(i)
It is also given that η
max
occurs at three quarters of full-load when Cu loss becomes equal to iron
loss.
Cu loss at 75 % of F.L. = x (3/4)
2
=9 x/16
Since y remains constant, hence 9 x/16 = y ...(ii)
Substituting the value of y in Eqn. (i), we get x + 9 x/16 = 3265 or x = 2090 W; y = 1175 W
Half-load Unity p.f.
Cu loss = 2090 × (1/2)
2
= 522 W ; total loss = 522 + 1175 = 1697 W
Output = 100 × 0.8 = 80 kW ; η = 80/81.697 = 0.979 or 97.9 %
Example 32.64. A 25-kVA, 1-phase transformer, 2,200 volts to 220 volts, has a primary resistance
of 1.0
and a secondary resistance of 0.01 . Find the equivalent secondary resistance and the
full-load efficiency at 0.8 p.f. if the iron loss of the transformer is 80% of the full-load Cu loss.
(Elect. Technology, Utkal Univ. 1998)
Solution. K = 220/2,200 = 1/10 ; R
02
= R
2
+ K
2
R
1
= 0.01 + 1/100 = 0.02
Full-load I
2
= 25,000/220 = 113.6 A ; F.L. Cu loss = I
2
2
R
02
= 113.6
2
× 0.02 = 258 W.
Iron loss = 80% of 258 = 206.4 W ; Total loss = 258 + 206.4 = 464.4 W
F.L. output = 25 × 0.8 = 20 kW = 20,000 W
Full-load η = 20,000 × 100/(20,000 + 464.4) = 97.7 %
Example 32.65. A 4-kVA, 200/400-V, 1-phase transformer has equivalent resistance and reactance
referred to low-voltage side equal to 0.5
and 1.5
respectively. Find the terminal voltage on the
high-voltage side when it supplies 3/4th full-load at power factor of 0.8, the supply voltage being 220
V. Hence, find the output of the transformer and its efficiency if the core losses are 100 W.
(Electrical Engineering ; Bombay Univ. 1985)
Solution. Obviously, primary is the low-voltage side and the secondary, the high voltage side.
Here, R
01
= 0.5 and X
01
= 1.5 . These can be transferred to the secondary side with the help of
the transformation ratio.
K = 400/200 = 2 ; R
02
= K
2
R
01
= 2
2
× 0.5 = 2 ; X
02
= K
2
X
01
= 4 × 1.5 = 6
Secondary current when load is 3/4 the, full-load is = (1,000 × 4 × 3/4)/400 = 7.5 A
Total drop as referred to transformer secondary is
= I
2
(R
02
cos φ + X
02
sin φ)* = 7.5 (2 × 0.8 + 6 × 0.6) = 39 V
Terminal voltage on high-voltage side under given load condition is
= 400 39 = 361 V
Cu loss = I
2
2
R
02
= 7.5
2
× 2 = 112.5 W Iron loss = 100 W
Total loss = 212.5 W output = (4 × 3/4) × 0.8 = 2.4 kW
Input = 2,400 + 212.5 = 2,612.5 W η = 2,400 × 100/2,612.5 = 91.87 %
Example 32.66. A 20-kVA, 440/220 V, I-φ, 50 Hz transformer has iron loss of 324 W. The Cu
loss is found to be 100 W when delivering half full-load current. Determine (i) efficiency when
* Assuming a lagging power factor
Transformer
1173
delivering full-load current at 0.8 lagging p.f. and (ii) the percent of full-load when the efficiency will
be maximum. (Electrotechnique-II, M.S. Univ., Baroda 1987)
Solution. F.L. Cu loss = 2
2
× 100 = 400 W ; Iron loss = 324 W
(i) F.L. efficiency at 0.8 p.f. =
20 0.8
100
(20 0.8) 0.724
×
×
×+
= 95.67 %
(ii)
kVA for maximum
F.L. kVA
=
Iron loss 324
F.L. Cu loss 400
=
= 0.9
Hence, efficiency would be maximum at 90 % of F.L.
Example 32.67. Consider a 4-kVA, 200/400 V single-phase transformer supplying full-load
current at 0.8 lagging power factor. The O.C./S.C. test results are as follows :
O.C. test : 200 V, 0.8 A, 70 W (I.V. side)
S.C. test : 20 V, 10 A, 60 (H.V. side)
Calculate efficiency, secondary voltage and current into primary at the above load.
Calculate the load at unity power factor corresponding to maximum efficiency.
(Elect. Machines Nagpur Univ. 1993)
Solution. Full-load, I
2
= 4000/400 = 10 A
It means that S.C. test has been carried out with full secondary flowing. Hence, 60 W represents
full-load Cu loss of the transformer.
Total F.L. losses = 60 + 70 = 130 W ; F.L. output = 4 × 0.8 = 3.2 kW
F.L. η = 3.2/3.33 = 0.96 or 96 %
S.C. Test
Z
02
= 20/10 = 2 ; I
2
2
R
02
= 60 or R
02
= 60/10
2
= 0.6 ; X
02
=
22
20.6
= 1.9
Transformer voltage drop as referred to secondary
= I
2
(R
02
cos φ + X
02
sin φ) = 10 (0.6 × 0.8 + 1.9 × 0.6) = 16.2 V
V
2
= 400 16.2 = 383.8 V
Primary current = 4000/200 = 20 A
kVA corresponding to η
max
= 4 ×
70/ 60
= 4.32 kVA
Load at u.p.f. corresponding to η
max
= 4.32 × 1 = 4.32 kW
Example 32.68. A 600 kVA, 1-phase transformer has an efficiency of 92 % both at full-load and
half-load at unity power factor. Determine its efficiency at 60 % of full-load at 0.8 power factor lag.
(Elect. Machines, A.M.I.E. Sec. B, 1992)
Solution.
η =
2
cos
100
()cos
iCu
xkVA
xkVA W xW
××φ
×
××φ++
where x represents percentage of full-load
W
i
is iron loss and W
Cu
is full-load Cu loss.
At F.L. u.p.f. Here x =1
92 =
2
1
16001
16001
Cu
WIW
××
××++
× 100, W
i
+ W
Cu
= 52.174 kW ...(i)
At half F.L. UPF. Here x = 1/2
92 =
2
1/2 600 1
(1 / 2) 6 00 1 (1 / 2)
iCu
WW
××
××++
× 100 ;
1174
Electrical Technology
W
i
+ 0.25 W
Cu
= 26.087 kW ...(ii)
From (i) and (ii), we get, W
i
= 17.39 kW, W
Cu
= 34.78 kW
60 % F.L. 0.8 p.f. (lag) Here, x = 0.6
η =
2
0.6 600 0.8 100
(0.6 600 0.8) 17.39 (0.6) 34.78
×××
××+ +
= 85.9 %
Example 32.69. A 600-kVA, 1-ph transformer when working at u.p.f. has an efficiency of 92 %
at full-load and also at half-load. Determine its efficiency when it operates at unity p.f. and 60 % of
full-load. (Electric. Machines, Kerala Univ. 1987)
Solution. The fact that efficiency is the same i.e. 92 % at both full-load and half-load will help us
to find the iron and copper losses.
At full-load
Output = 600 kW ; Input = 600/0.92 = 652.2 kW ; Total loss = 652.2 600 = 52.2 kW
Let x = Iron loss It remains constant at all loads.
y = F.L. Cu loss It is (kVA)
2
. x + y = 52.2 ...(i)
At half-load
Output = 300 kW ; Input = 300/0.92 Losses = (300/0.92 300) = 26.1 kW
Since Cu loss becomes one-fourth of its F.L. value, hence
x + y/4 = 26.1
Solving for x and y, we get x = 17.4 kW ; y = 34.8 kW ...(ii)
At 60 % full-load
Cu loss = 0.62 × 34.8 = 12.53 kW ; Total loss = 17.4 + 12.53 = 29.93 kW
Output = 600 × 0.6 = 360 kW ∴η = 360/389.93 = 0.965 or 96.5 %
Example 32.70. The maximum efficiency of a 100-kVA, single phase transformer is 98% and
occurs at 80% of full load at 8 p.f. If the leakage impedance of the transformer is 5 %, find the
voltage regulation at rated load of 0.8 power factor lagging.
(Elect. Machines-I, Nagpur Univ. 1993)
Solution. Since maximum efficiency occurs at 80 percent of full-load at 0.8 p.f.,
Output at η
max
= (100 × 0.8) × 0.8 = 64 kW ; Input = 64/0.98 = 65.3 kW
Total loss = 65.3 64 = 1.3 kW. This loss is divided equally between Cu and iron.
Cu loss at 80 % of full-load = 1.3/2 = 0.65 kW
Cu loss at full-load = 0.65/0.8
2
= 1 kW
% R =
22
Cu loss 100
100 1
100
VI
×=×
= 1 % = v
r
; v
x
= 5 %
% age regn. = (1 × 0.8 + 5 × 0.6) +
1
200
(5 × 0.8 1 × 0.6)
2
= 0.166 %
Example 32.71. A 10 kVA, 5000/440-V, 25-Hz single phase transformer has copper, eddy current
and hysteresis losses of 1.5, 0.5 and 0.6 per cent of output on full load. What will be the percentage
losses if the transformer is used on a 10-kV, 50-Hz system keeping the full-load current constant ?
Assume unity power factor operation. Compare the full load efficiencies for the two cases.
(Elect. Machines, A.M.I.E., Sec. B, 1991)
Solution. We know that E
1
= 4,44 f N
1
B
1
A.. When both excitation voltage and frequency are
doubled, flux remains unchanged.
F.L. output at upf = 10 kVA × 1 = 10 kW
Transformer
1175
F.L. Cu loss = 1.5 × 10/100 = 0.15 kW ; Eddy current loss
=0.5 × 10/100 = 0.05 kW ; Hysteresis loss = 0.6 × 10/100 = 0.06 kW
Now, full-load current is kept constant but voltage is increased from 5000 V to 10,000 V. Hence,
output will be doubled to 20 kW. Due to constant current, Cu loss would also remain constant.
New Cu loss = 0.15 kW, % Cu loss = (0.15/20) × 100 = 0.75 %
Now, eddy current loss f
2
and hysteresis loss f.
New eddy current loss = 0.05 (50/25)
2
= 0.2 kW, % eddy current loss = (0.2/20) × 100 = 1 %
Now, W
h
= 0.06 × (50/25) = 0.12 kW, % W
h
= (0.12/20) × 100 = 0.6 %
η
1
=
10
100
10 0.15 0.05 0.06
×
+++
= 87.4 %
η
2
=
20
100
20 0.15 0.2 0.12
×
+++
= 97.7 %
Example 32.72. A 300-kVA, single-phase transformer is designed to have a resistance of 1.5 %
and maximum efficiency occurs at a load of 173.2 kVA. Find its efficiency when supplying full-load
at 0.8 p.f. lagging at normal voltage and frequency. (Electrical Machines-I, Gujarat Univ. 1985)
Solution. % R =
22
F.L. Cu loss
100
Full-load
VI
×
; 1.5 =
F.L. Cu loss
100
300 1000
×
×
F.L. Cu loss = 1.5 × 300 × 1000/100 = 4500 W
Also, 173.2 = 300
Iron Loss
4500
; Iron loss = 1500 W
Total F.L. loss = 4500 + 1500 = 6 kW
F.L. η at 0.8 p.f. =
300 0.8
100
(300 0.8) 6
×
×
×+
= 97.6 %
Example 32.73. A single phase transformer is rated at 100-kVA, 2300/230-V, 50 Hz. The
maximum flux density in the core is 1.2 Wb/m
2
and the net cross-sectional area of the core is 0.04 m
2
.
Determine
(a) The number of primary and secondary turns needed.
(b) If the mean length of the magnetic circuit is 2.5 m and the relative permeability is 1200,
determine the magnetising current. Neglect the current drawn for the core loss.
(c) On short-circuit with full-load current flowing, the power input is 1200 W and an open-
circuit with rated voltage, the power input was 400 W. Determine the efficiency of the transformer
at 75 % of full-load with 0.8 p.f. lag.
(d) If the same transformer is connected to a supply of similar voltage but double the frequency
(i.e., 100 Hz). What is the effect on its efficiency ? (Elect. Engg., Bombay Univ. 1988)
Solution. (a) Applying e.m.f. equation of the transformer to the primary, we have
2300 = 4.44 × 50 × N
1
× (1.2 × 0.0.4) N
1
= 216
K = 230/2300 = 1/10 N
2
= KN
1
= 216/10 = 21.6 or 22
(b) AT = H × l =
7
0
1.2 2.5
1989
4 10 1200
r
B
l
×
×= =
µµ
π× ×
I =
1989
216
= 9.21 A
(c) F.L. Cu loss = 1200 W S.C. test ; Iron loss = 400 W O.C. test
Cu loss at 75 % of F.L. = (0.75)
2
× 1200 = 675 W
Total loss = 400 + 675 = 1075 kW
Output = 100 × (3/4) × 0.8 = 60 kW ; η = (60/61.075) × 100 = 98.26 %
1176
Electrical Technology
(d) When frequency is doubled, iron loss is increased because
(i) hysteresis loss is doubled W
h
f
(ii) eddy current loss is quadrupled W
e
f
2
Hence, efficiency will be decreased.
Example 32.74. A transformer has a resistance of 1.8 % and a reactance of 5.4 %. (a) At full
load, what is the power-factor at which the regulation will be : (i) Zero, (ii) positive-maximum ?
(b) If its maximum efficiency occurs at full-load (at unity p.f.), what will be the efficiency under these
conditions ?
Solution : Approximate percentage regulation is given, in this case, by the relationship
1.8 cos φ ± 5.4 sin φ.
(a) Regulation :
(i) If regulation is zero, negative sign must be applicable. This happens at leadings p.f.
Corresponding p.f. = tan φ = 1.8/5.4 = 0.333 leading
φ = 18.44
0
leading
(ii) For maximum positive regulation, lagging p.f. is a must. From phasor diagram, the result can
be obtained.
Corresponding tan φ = 5.4/1.8 = 3, φ = 71.56 lagging
% Voltage regulation = 1.8 cos φ + 5.4 sin φ = 5.7 %
(b) Efficiency : Maximum efficiency occurs at such a load when
Iron losses = Copper losses
This means Iron-losses are 1.8 %.
Efficiency = 100/(100 + 1.8 + 1.8) = 96.52 %
Example 32.75. A 10 kVA, 1 phase, 50 Hz, 500/250 V transformer gave following test results :
OC test (LV) side : 250 V, 3.0 A, 200 W
SC test (LV) side : 15 V, 30 A, 300 W.
Calculate efficiency and regulation at full load, 0.8 p.f. lagging.
(Nagpur University, Summer 2000)
Solution. For efficiency calculations, full load current should be calculated, on the L.V. side in
this case,
F.L. Current =
10,000
250
= 40 amp
Short-circuit test data have been given at 30 A current on the L.V. side.
i
2
r losses at 40 A L.V. side =
()
2
40
300
30
×
Watts = 533.3 Watts
At rated voltage, iron losses (from O.C. test) = 200 Watts
F.L. Output at 0.8 P.F. = 10,000 × 0.8 = 8000 Watts
Hence, η =
8000
100 %
8000 4733.3
×
+
= 91.6 %
For regulation, series resistance and reactance parameters of the equivalent circuit have to be
evaluated, from the S.C. test.
Series Impedance, Z =
15
30
= 0.5 ohm
Series resistance, r =
300
30 30
×
= 0.333 ohm
Transformer
1177
Series reactance, x =
22
0.5 0.333
= 0.373 ohm
By Approximate formula,
p.u. regulation at full load, 0.8 p.f. lagging
=
40
250
[0.333 × 0.8 0.373 × 0.6] = 6.82 × 10
3
p.u.
When converted into volts, this is 6.82 × 10
3
× 250 = 1.70 volt
Example 32.76. A 40 kVA, 1-ph, transformer has an iron loss of 400 W, and full copper loss of
800 W. Find the load at which maximum efficiency is achieved at unity power factor.
(Amravati University, Winter 1999)
Solution. If x = fraction of rated load at which the efficiency is maximum.
P
i
= Ironloss = 400 W
P
c
= F.L. copperloss = 800 W
Then x
2
P
c
= P
i
On substitution of numerical values of P
i
and P
c
, we get
x = 0.707
Hence, the efficiency is maximum, at unity p.f. and at 70.7 % of the rated Load. At this load,
copperloss = Ironloss = 0.40 kW
Corresponding output = 40 × 0.707 × 1
= 28.28 kW
Corresponding efficiency =
28.28
28.28 0.4 0.4
++
= 97.25 %
Extension to Question : (a) At what load (s) at unity p.f. the efficiency will be 96.8 % ?
40 70 125
96
96.5
96.8
97
97.25
97.5
98
X
X
0.90
P.f.
Unity
P.f.
/
%
%
Load
0
0
Fig. 32.58. Efficiency variation with load
Solution. Let x = Fractional load at which the concerned efficiency occurs, at unity p.f.
2
40
40 0.8 0.40
x
xx++
= 0.968
This gives the following values of x : x
1
= 1.25 x
2
= 0.40
1178
Electrical Technology
Thus, at 40 % and at 125 % of the rated load, the efficiency will be 96.8 % as marked on the graph,
in Fig. 32.58.
(b) How will maximum-efficiency condition be affected if the power factor is 0.90 lagging ?
Solution.
The condition for efficiency-variation-statement is that the power factor remains constant. Thus, for
0.90 lagging p.f., another curve (Lower curve in Fig. 32.58) will be drawn for which the maximum efficiency
will occur at the same value of x (= 0.707), but
Maximum efficiency =
2
40 cos
40 cos 0.80 0.40
x
x
φ
φ+ +
=
28.28 0.90
(28.28 0.90) 0.80
×
×+
= 97 %
Example 32.77. A 10 kVA, 500/250 V, single-phase transformer gave the following test results:
S.C. Test (H.V. side) : 60 V, 20 A, 150 W
The maximum efficiency occurs at unity power factor and at 1.20 times full-load current. De-
termine full-load efficiency at 0.80 p.f. Also calculate the maximum efficiency.
(Rajiv Gandhi Technical University, Bhopal, Summer 2001)
Solution. Full-load current on H.V. side = 10,000/500 = 20 Amp
S.C. test has been conducted from H.V. side only. Hence, full-load copper-loss, at unity
p.f. = 150 watts
(a) Maximum efficiency occurs at 1.2 times full-load current, at unity p.f. corresponding copper-
loss = (1.2)
2
× 150 = 216 watts
At maximum efficiency, copper-loss = core-loss = 216 watts
Corresponding Power-output = 1.2 × 10,000 × 1.0 = 12 kW
Hence, maximum efficiency at unity P.f. = (12)/(12 + 0.216 + 0.2160) = 0.9653 = 96.53 %
(b) Full-load efficiency at 0.80 P.f.
Output Power at full-load, 0.80 P.f. = 10,000 × 0.8 = 8000 W, constant core-loss = 216 W
Corresponding copper-loss = 150 W
Total losses = 366 W
Hence, efficiency = (8000/8366) × 100 % = 95.63 %.
32.31. Variation of Efficiency with Power Factor
The efficiency of a transformer is given by
η =
Input - Losses
Output
Input Input
=
=1
22
Losses Losses
1
Input ( cos losses)
VI
=−
φ+
Let, losses/V
2
I
2
= x
∴η=1
22
22
losses /
cos (losses / )
VI
VI
φ+
=1
/cos
1
(cos ) 1 ( / cos )
x
x
xx
φ
=−
φ+ + φ
The variations of efficiency with power factor at
different loadings on a typical transformer are shown in
Fig. 32.59.
Fig. 32.59
0.99
0.98
0.97
0.96
00.51.0 1.5
1
.
0
p
.
f
.
0
.
8
p
.
f
.
0
.
6
p
.
f
.
Efficiency
Full Load
Transformer
1179
Tutorial Problems 32.4
1. A 200-kVA transformer has an efficiency of 98 % at full-load. If the maximum efficiency occurs at
three-quarters of full-load, calculate (a) iron loss at F.L. (b) Cu loss at F.L. (c) efficiency at half-load.
Ignore magnetising current and assume a p.f. of 0.8 at all loads.
[(a) 1.777 kW (b) 2.09 kW (c) 97.92%]
2. A 600 kVA, 1-ph transformer has an efficiency of 92 % both at full-load and half-load at unity power
factor. Determine its efficiency at 60 % of full load at 0.8 power factor lag.
[90.59%] (Elect. Machines, A.M.I.E. Sec. B, 1992)
3. Find the efficiency of a 150 kVA transformer at 25 % full load at 0.8 p.f. lag if the copper loss at full
load is 1600 W and the iron loss is 1400 W. Ignore the effects of temperature rise and magnetising
current. [96.15%] (Elect. Machines, A.M.I.E. Sec. B, 1991)
4. The F.L. Cu loss and iron loss of a transformer are 920 W and 430 W respectively. (i) Calculate the
loading of the transformer at which efficiency is maximum (ii) what would be the losses for giving
maximum efficiency at 0.85 of full-load if total full-load losses are to remain unchanged ?
[(a) 68.4% of F.L. (ii) W
i
= 565 W ; W
cu
= 785 W]
5. At full-load, the Cu and iron losses in a 100-kVA transformer are each equal to 2.5 kW. Find the
efficiency at a load of 65 kVA, power factor 0.8. [93.58%] (City & Guilds London)
6. A transformer, when tested on full-load, is found to have Cu loss 1.8% and reactance drop 3.8%.
Calculate its full-load regulation (i) at unity p.f. (ii) 0.8 p.f. lagging (iii) 0.8 p.f. leading.
[(i) 1.80% (ii) 3.7 % (iii)
0.88%]
7. With the help of a vector diagram, explain the significance of the following quantities in the open-
circuit and short-circuit tests of a transformer (a) power consumed (b) input voltage (c) input current.
When a 100-kVA single-phase transformer was tested in this way, the following data were obtained
: On open circuit, the power consumed was 1300 W and on short-circuit the power consumed was
1200 W. Calculate the efficiency of the transformer on (a) full-load (b) half-load when working at
unity power factor. [(a) 97.6% (b) 96.9%] (London Univ.)
8. An 11,000/230-V, 150-kVA, 50-Hz, 1-phase transformer has a core loss of 1.4 kW and full-load Cu
loss of 1.6 kW. Determine (a) the kVA load for maximum efficiency and the minmum efficiency (b)
the efficiency at half full-load at 0.8 power factor lagging. [140.33 kVA, 97.6% ; 97%]
9. A single-phase transformer, working at unity power factor has an efficiency of 90 % at both half-load
and a full-load of 500 kW. Determine the efficiency at 75 % of full-load. [90.5%] (I.E.E. London)
10. A 10-kVA, 500/250-V, single-phase transformer has its maximum efficiency of 94 % when delivering
90 % of its rated output at unity power factor. Estimate its efficiency when delivering its full-load
output at p.f. of 0.8 lagging. [92.6%] (Elect. Machinery, Mysore Univ, 1979)
11. A single-phase transformer has a voltage ratio on open-circuit of 3300/660-V. The primary and
secondary resistances are 0.8 and 0.03 respectively, the corresponding leakage reactance being
4 and 0.12 . The load is equivalent to a coil of resistance 4.8 and inductive reactance 3.6 .
Determine the terminal voltage of the transformer and the output in kW. [636 V, 54 kW]
12. A 100-kVA, single-phase transformer has an iron loss of 600 W and a copper loss of 1.5 kW at full-
load current. Calculate the efficiency at (a) 100 kVA output at 0.8 p.f. lagging (b) 50 kVA output at
unity power factor. [(a) 97.44% (b) 98.09%]
13. A 10-kVA, 440/3300-V, 1-phase transformer, when tested on open circuit, gave the following figures
on the primary side : 440V ; 1.3 A ; 115 W.
When tested on short-circuit with full-load current flowing, the power input was 140 W. Calculate
the efficiency of the transformer at (a) full-load unity p.f. (b) one quarter full-load 0.8 p.f.
[(a) 97.51% (b) 94.18%] (Elect. Engg-I, Sd. Patel Univ. June 1977)
14. A 150-kVA single-phase transformer has a core loss of 1.5 kW and a full-load Cu loss of 2 kW.
Calculate the efficiency of the transformer (a) at full-load, 0.8 p.f. lagging (b) at one-half full-load
1180
Electrical Technology
unity p.f. Determine also the secondary current at which the efficiency is maximum if the secondary
voltage is maintained at its rated value of 240 V. [(a) 97.17% (b) 97.4% ; 541 A]
15. A 200-kVA, 1-phase, 3300/400-V transformer gave the following results in the short-circuit test. With
200 V applied to the primary and the secondary short-circuited, the primary current was the full-load
value and the input power 1650 W. Calculate the secondary p.d. and percentage regulation when the
secondary load is passing 300 A at 0.707 p.f. lagging with normal primary voltage.
[380 V ; 480%]
16. The primary and secondary windings of a 40-kVA, 6600/250-V, single-phase transformer have
resistances of 10 and 0.02 respectively. The leakage reactance of the transformer referred to the
primary is 35 . Calculate
(a) the primary voltage required to circulate full-load current when the secondary is short-circuited.
(b) the full-load regulations at (i) unity (ii) 0.8 lagging p.f. Neglect the no-load current.
[(a) 256 V (b) (i) 2.2% (ii) 3.7%] (Elect. Technology, Kerala Univ. 1979)
17. Calculate :
(a) F.L. efficiency at unity p.f.
(b) The secondary terminal voltage when supplying full-load secondary current at p.f. (i) 0.8 lag
(ii) 0.8 lead for the 4-kVA, 200/400 V, 50 Hz, 1-phase transformer of which the following are the
test figures :
Open circuit with 200 V supplied to the primary winding-power 60 W. Short-circuit with 16 V
applied to the h.v. winding-current 8 A, power 40 W. [0.97 ; 383 V ; 406 V]
18. A 100-kVA, 6600/250-V, 50-Hz transformer gave the following results :
O.C. test : 900 W, normal voltage.
S.C. test (data on h.v. side) : 12 A, 290 V, 860 W
Calculate
(a) the efficiency and percentage regulation at full-load at 0.8 p.f. lagging.
(b) the load at which maximum efficiency occurs and the value of this efficiency at p.f. of unity, 0.8
lag and 0.8 lead. [(a) 97.3%, 4.32% (b) 81 kVA, 97.8%, 97.3% ; 97.3%]
19. The primary resistance of a 440/110-V transformer is 0.5 and the secondary resistance is 0.04 .
When 440 V is applied to the primary and secondary is left open-circuited, 200 W is drawn from the
supply. Find the secondary current which will give maximum efficiency and calculate this efficiency
for a load having unity power factor.
[53 A ; 93.58%] (Basic Electricity & Electronics. Bombay Univ. 1981)
20. Two tests were performed on a 40-kVA transformer to predetermine its efficiency. The results were:
Open circuit : 250 V at 500 W
Short circuit : 40 V at F.L. current, 750 W both tests from primary side.
Calculate the efficiency at rated kVA and 1/2 rated kVA at (i) unity p.f. (ii) 0.8 p.f.
[96.97% ; 96.68% ; 96.24% ; 95.87%]
21. The following figures were obtained from tests on a 30-kVA, 3000/110-V transformer :
O.C. test : 3000 V 0.5 A 350 W ; S.C. test ; 150 V 10 A 500 W
Calculate the efficiency of the transformer at
(a) full-load, 0.8 p.f. (b) half-load, unity p.f.
Also, calculate the kVA output at which the efficiency is maximum. [96.56% ; 97% ; 25.1 kVA]
22. The efficiency of a 400 kVA, 1-phase transformer is 98.77 % when delivering full load at 0.8 power
factor, and 99.13 % at half load and unity power factor. Calculate (a) the iron loss, (b) the full load
copper loss. [(a) 1012 W (b) 2973 W] (Rajiv Gandhi Technical University, 2000)
Transformer
1181
32.32. All-day Efficiency
The ordinary or commercial efficiency of
a transformer is given by the ratio
Output in watts
Input in watts
.
But there are certain types of transformers
whose performance cannot be judged by this
efficiency. Transformers used for supplying
lighting and general network i.e., distribution
transformers have their primaries energised all
the twenty-four hours, although their
secondaries supply little or no-load much of
the time during the day except during the house
lighting period. It means that whereas core
loss occurs throughout the day, the Cu loss
occurs only when the transformers are loaded. Hence, it is considered a good practice to design such
transformers so that core losses are very low. The Cu losses are relatively less important, because they
depend on the load. The performance of such is compared on the basis of energy consumed during a
certain time period, usually a day of 24 hours.
∴η
all-day
=
Output in kWh
Input in kWh
(For 24 hours)
This efficiency is always less than the commercial efficiency of a transformer.
To find this all-day efficiency or (as it is also called) energy efficiency, we have to know the load cycle
on the transformer i.e., how much and how long the transformer is loaded during 24 hours. Practical
calculations are facilitated by making use of a load factor.
Example 32.78. Find the all-day efficiency of 500-kVA distribution transformer whose copper
loss and iron loss at full load are 4.5 kW and 3.5 kW respectively. During a day of 24 hours, it is
loaded as under :
No. of hours Loading in kW Power factor
6 400 0.8
10 300 0.75
4 100 0.8
40
(Elect. Machines, Nagpur Univ. 1993)
Solution. It should be noted that a load of 400 kW at 0.8 p.f. is equal to 400/0.8 = 500 kVA.
Similarly, 300 kW at 0.75 p.f. means 300/0.75 = 400 kVA and 100 kW at 0.8 p.f. means 100/0.8 = 125 kVA
i.e., one-fourth of the full-load.
Cu loss at F.L. of 500 kVA = 4.5 kW
Cu loss at 400 kVA = 4.5 × (400/500)
2
= 2.88 kW
Cu loss at 125 kVA = 4.5 × (125/500)
2
= 0.281 kW
Total Cu loss in 24 hrs = (6 × 4.5) + (10 × 2.88) + (4 × 0.281) + (4 × 0)
= 56.924 kWh
The iron loss takes place throughout the day irrespective of the load on the transformer because
its primary is energized all the 24 hours.
The world first 5,000 KVA amorphous transformer
commissioned in August 2001 in Japan
1182
Electrical Technology
Iron loss in 24 hours = 24 × 3.5 = 84 kWh
Total transformer loss = 56.924 + 84 = 140.924 kWh
Transformer output is 24 hrs = (6 × 400) + (10 × 300) + (4 × 100) = 5800 kWh
∴η
all-day
=
output
5800
output + losses 5800 140.924
=
+
= 0.976 or 97.6 %
Example 32.79. A 100-kVA lighting transformer has a full-load loss of 3 kW, the losses being
equally divided between iron and copper. During a day, the transformer operates on full-load for 3
hours, one half-load for 4 hours, the output being negligible for the remainder of the day. Calculate
the all-day efficiency. (Elect. Engg. Punjab Univ. 1990)
Solution. It should be noted that lighting transformers are taken to have a load p.f. of unity.
Iron loss for 24 hour = 1.5 × 24 = 36 kWh ; F.L. Cu loss = 1.5 kW
Cu loss for 3 hours on F.L. = 1.5 × 3 = 4.5 kWh
Cu loss at half full-load = 1.5/4 kW
Cu loss for 4 hours at half the load = (1.5/4) × 4 = 1.5 kWh
Total losses = 36 + 4.5 + 1.5 = 42 kWh
Total output = (100 × 3) + (50 × 4) = 500 kWh
∴η
all-day
= 500 × 100/542 = 92.26 %
Incidentally, ordinary or commercial efficiency of the transformer is
= 100/(100 + 3) = 0.971 or 97.1 %
Example 32.80. Two 100-kW transformers each has a maximum efficiency of 98 % but in one the
maximum efficiency occurs at full-load while in the other, it occurs at half-load. Each transformer
is on full-load for 4 hours, on half-load for 6 hours and on one-tenth load for 14 hours per day.
Determine the all-day efficiency of each transformer. (Elect. Machines-I, Vikram Univ. 1988)
Solution. Let x be the iron loss and y the full-load Cu loss. If the ordinary efficiency
is a maximum at 1/m of full-load, then x = y/m
2
.
Now, output = 100 kW ; Input = 100/0.98
Total losses = 100/0.98 100 = 2.04 kW
y + x/m
2
=2.04
Ist Transformer
Here m = 1 ; y + y = 2.04 ; y = 1.02 kW and x = 1.02 kW
Iron loss for 24 hours = 1.02 × 24 = 24.48 kWh
Cu loss for 24 hours = 4 × 1.02 + 6 × (1.02/4) + 14 (1.02/10
2
) = 5.73 kWh
Total loss = 24.48 + 5.73 = 30.21 kWh
=4 × 100 + 6 × 50 + 14 × 10 = 840 kWh
∴η
all-day
= 840/870.21 = 0.965 or 96.5 %
2nd Transformer
Here 1/m =1/2 or m = 2 y + y/4 = 2.04
or y = 1.63 kW ; x = 0.14 kW
Output = 840 kWh ...as above
Iron loss for 24 hours = 0.41 × 24 = 9.84 kWh
Cu loss for 24 hours = 4 × 1.63 + 6 (1.63/4) + 14 (1.63/10
2
) = 9.19 kWh
Total loss = 9.84 + 9.19 = 19.03 kWh
∴η
all-day
= 840/859.03 = 0.978 or 97.8 %
Transformer
1183
Example 32.81. A 5-kVA distribution transformer has a full-load efficiency at unity p.f. of 95
%, the copper and iron losses then being equal. Calculate its all-day efficiency if it is loaded throughout
the 24 hours as follows :
No load for 10 hours Quarter load for 7 hours
Half load for 5 hours Full load for 2 hours
Assume load p.f. of unity. (Power Apparatus-I, Delhi Univ. 1987)
Solution. Let us first find out the losses from the given commercial efficiency of the transformer.
Output = 5 × 1 = 5 kW ; Input = 5/0.95 = 5.264 kW
Losses = (5.264 5.000) = 0.264 kW = 264W
Since efficiency is maximum, the losses are divided equally between Cu and iron.
Cu loss at F.L. of 5 kVA = 264/2 = 132 W ; Iron loss = 132 W
Cu loss at one-fourth F.L. = (1/4)
2
× 132 = 8.2 W
Cu loss at one-half F.L. = (1/2)
2
× 132 = 33 W
Quarter load Cu loss for 7 hours = 7 × 8.2 = 57.4 Wh
Half-load Cu loss for 5 hours = 5 × 33 = 165 Wh
F.L. Cu loss for 2 hours = 2 × 132 = 264 Wh
Total Cu loss during one day = 57.4 + 165 + 264 = 486.4 Wh = 0.486 kWh
Iron loss in 24 hours = 24 × 132 = 3168 Wh = 3.168 kWh
Total losses in 24 hours = 3.168 + 0.486 = 3.654 kWh
Since load p.f. is to be assumed as unity.
F.L. output = 5 × 1 = 5 kW ; Half F.L. output = (5/2) × 1 = 2.5 kW
Quarter load output = (5/4) × 1 = 1.25 kW
Transformer output in a day of 24 hours = (7 × 1.25) + (5 × 2.5) + (2 × 5) = 31.25 kWh
η
all-day
=
31.25
100
(31.25 3.654)
×
+
= 89.53 %
Example 32.82. Findall day” efficiency of a transformer having maximum efficiency of 98 %
at 15 kVA at unity power factor and loaded as follows :
12 hours – 2 kW at 0.5 p.f. lag
6 hours – 12 kW at 0.8 p.f. lag
6 hours – at no load. (Elect. Machines-I, Nagpur Univ. 1993)
Solution. Output = 15 × 1 = 15 kW, input = 15/0.98
Losses = (15/0.98 15) = 0.306 kW = 306 W
Since efficiency is maximum, the losses are divided equally between Cu and iron.
Cu loss at 15 kVA = 306/2 = 153 W, Iron loss = 153 W
2 kW at 0.5 p.f. = 2/0.5 = 4 kVA, 12 kW at 0.8 p.f. = 12/0.8 = 15 kVA
Cu loss at 4 kVA = 153 (4/15)
2
= 10.9 W ; Cu loss at 15 kVA = 153 W.
Cu loss in 12 hrs = 12 × 10.9 = 131 Wh ; Cu loss in 6 hr = 6 × 153 = 918 Wh.
Total Cu loss for 24 hr = 131 + 918 = 1050 Wh = 1.05 kWh
Iron loss for 24 hrs = 24 × 153 = 3,672 Wh = 3.672 kWh
Output in 24 hrs = (2 × 12) + (6 × 12) = 96 kWh
Input in 24 hrs = 96 + 1.05 + 3.672 = 100.72 kWh
∴η
all-day
= 96 × 100/100.72 = 95.3 %
1184
Electrical Technology
Example 32.83. A 150-kVA transformer is loaded as follows :
Load increases from zero to 100 kVA in 3 hours from 7 a.m. to 10.00 a.m., stays at 100 kVA from
10 a.m. to 6 p.m. and then the transformer is disconnected till next day. Assuming the load to be
resistive and core-loss equal to full-load copper loss of 1 kW, determine the all-day efficiency and
the ordinary efficiency of the transformer. (Electrical Machines-II, Indore Univ. 1990)
Solution. Since load is resistive, its p.f. is unity.
Average load from 7 a.m. to 10 a.m. = (0 + 100)/2 = 50 kVA i.e., one-third F.L.
Load from 10 a.m. to 6 p.m. = 100 kVA i.e., 2/3 of F.L.
Ordinary Efficiency
In this case, load variations are not relevant.
Output = 150 × 1 = 150 kW ; Iron loss = Cu loss = 1 kW ; Total loss = 2 kW.
Ordinary η = 150/(150 + 2) = 0.9868 or 98.68 %
All-day Efficiency
Cu loss from 7-10 a.m. = 3 × (1/3)
2
× 1 = 0.333 kWh
Cu loss from 10 a.m. to 6.p.m. = 8 × (2/3)
2
× 1 = 3.555 kWh
Total Cu loss for 24 hrs = 0.333 + 3.555 = 3.888 kWh
Total iron loss for 24 hrs = 24 × 1 = 24 kWh
Losses for a day of 24 hrs = 27.888 kWh
Output for 24 hrs = 3 × (50 × 1) + 8 (100 × 1) = 950 kWh
∴η
all-day
=
950 100
(950 27.888)
×
+
= 97.15 %
Example 32.84. Find the all-day efficiency of a 50 kVA distribution transformer having full load
efficiency of 94 % and full-load copper losses are equal to the constant iron losses. The loading of
the transformer is as follows, the power factor being 1.0.
(i) No load for 10 hours (ii) Half load for 5 hours
(iii)25 % load for 6 hours (iv) Full load for 3 hours. (Sambalpur University, 1998)
Solution. At full load unity p.f.
efficiency = 94 % =
50,000
50,000 2
i
P
+
2P
i
=
50,000
50,000
0.94



,orP
i
=
1 1 0.94
50,000
20.94

×


,
P
i
= 25,000 ×
0.06
0.94
= 1596 Watts
Hence, full load Cu-losses = 1596 Watts
(a) Energy required in overcoming Cu-losses, during 24 hours
(i) No load for 10 hours : zero
(ii) At half load, Cu-losses = (0.5)
2
× 1596 Watts = 399
Energy in 5 hours =
399 5
1000
×
kWh = 1.995 kWh
(iii) At 25 % load, Cu-loss = (0.25)
2
× 1596 = 99.75 Watts
Energy in 6 hours =
6 99.75
1000
×
= 0.5985 kWh
(iv) Energy lost during 3 hours of full load =
1596 3
1000
×
= 4.788 kWh
(b) Energy lost in constant core-losses for 2 hours =
1596
24
1000
×
= 38.304 kWh
Transformer
1185
(c) Energy required by the load = 25 × 5 + 12.5 × 6 + 50 × 3 = 125 + 75 + 150 = 350 kWh
All-day efficiency =
350
100
350 38.304 7.3815
×
++
= 88.454 %
Example 32.85. A 10 kVA, 1-ph transformer has a core-loss of 40 W and full load ohmic loss of
100 W. The daily variation of load on the transformer is as follows :
6 a.m. to 1 p.m. 3 kW at 0.60 p.f.
1 p.m. to 5 p.m.
8kW at 0.8 p.f.
5 p.m. to 1 a.m. full load at u.p.f.
1 a.m. to 6 a.m. no load
Determine all day efficiency of the transformer (Amravati University, 1999)
Solution. Fractional loading (= x) and the output kWh corresponding to load variations can be
worked out in tabular form, as below :
S.N. Number x = x
2
P
c
in kW Output in Copper Loss
of hours kWh in kWh
17
3/0.6
10
= 0.5 0.50
2
× 0.10 = 0.025 3 × 7 = 21 0.025 × 7 = 0.175
24
8/0.8
10
= 1.0 0.10 8 × 4 = 32 0.1 × 4 = 0.40
38
10/1
10
= 1.0 0.10 10 × 8 = 80 0.1 × 8 = 0.8
4 5 Zero Zero Zero Zero
Output in kWh 21 + 32 + 80
= 133
Ohmic Loss, in kWh 0.175 + 0.40 + 0.80
= 1.375
Core loss during 24 Hrs =
400
1000
× 24 = 0.96 kWh
Hence, Energy efficiency (= All day Efficiency) =
133
133 +1.375 + 0.96
× 100 = 98.3 %
Example 32.86. A transformer has its maximum efficiency of 0.98 at 15 kVA at unity p.f.
During a day, it is loaded as follows :
12 hours : 2 kW at 0.8 p.f.
6 hours : 12 kW at 0.8 p.f.
6 hours : 18 kW at 0.9 p.f.
Find the all day efficiency. (Manomaniam Sundaranar Univ. April 1998)
Solution. Let 15 kVA be treated as full load.
Output at maximum efficiency = 15000 × 1 watts
Input = 15000/0.98 watts
Losses = Input Output = 15000 (1/0.98 1) = 15000 × 2/98 = 306 Watts
At maximum efficiency, since the variable copper-loss and constant core-loss are equal.
Full load copper-loss = Constant core-loss = 306/2 = 153 Watts
Let the term x represents the ratio of required Load/Full load.
Output = 15 × cos φ
load kVA
Xmer Rating
1186
Electrical Technology
Following tabular entries simplify the calculations for all-day efficiency.
S.N. x Hrs x
2
P
c
in kW Energy in Copper Output during the
-loss in kWh period in kWh
1
2/0.5
15
=
15
4
12 0.01088 0.131 24
2
12/0.8
1.0
15
=
6 0.153 0.918 72
3
18/0.9
4/3
15
=
6 0.272 1.632 108
Total output during the day = 204 kWh
Total copper-loss during the day = 2.681 kWh
Total core-loss during the day = 0.153 × 24 = 3.672
All day efficiency = (204/210.353) × 100 = 96.98 %
Tutorial Problems 32.5
1. A 100-kVA distribution transformer has a maximum efficiency of 98 % at 50 % full-load and unity
power factor. Determine its iron losses and full-load copper losses.
The transformer undergoes a daily load cycle as follows :
Load Power factor Load duration
100 kVA 1.0 8 hrs
50 kVA 0.8 6 hrs
No load 10 hrs
Determine its all-day efficiency. (Electrical Engineering, MS Univ. Baroda 1979)
2. What is meant by energy efficiency of a transformer ?
A 20-kVA transformer has a maximum efficiency of 98 percent when delivering three-fourth full-load
at u.p.f. If during the day, the transformer is loaded as follows :
12 hours No load
6 hours 12 kWh, 0.8 p.f.
6 hours 20 kW, u.p.f.
Calculate the energy efficiency of the transformer.
(Electrical Technology-III, Gwalior Univ., 1980)
32.33. Auto-transformer
It is a transformer with one winding only, part of this being common to both primary and
secondary. Obviously, in this transformer the primary and secondary are not electrically isolated from
each other as is the case with a 2-winding transformer. But its theory and operation are similar to those
of a two-winding transformer. Because of one winding, it uses less copper and hence is cheaper. It is
used where transformation ratio differs little from unity. Fig. 32.60 shows both step down and step-up
auto-transformers.
As shown in Fig. 32.60 (a), AB, is primary winding having N
1
turns and BC is secondary winding
having N
2
turns. Neglecting iron losses and no-load current.
221
112
VN I
VNI
==
=K
Transformer
1187
The current in section CB is vector
difference* of I
2
and I
1
. But as the two
currents are practically in phase oppo-
sition, the resultant current is (I
2
I
1
)
where I
2
is greater than I
1
.
As compared to an ordinary 2-
winding transformer of same output, an
auto-transformer has higher efficiency
but smaller size. Moreover, its voltage
regulation is also superior.
Saving of Cu
Volume and hence weight of Cu, is
proportional to the length and area of
the cross-section of the conductors.
Now, length of conductors is proportional to the number of turns and cross-section depends on current.
Hence, weight is proportional to the product of the current and number of turns.
With reference to Fig. 32.60,
Wt. of Cu in section AC is (N
1
N
2
) I
1
; Wt. of Cu in section BC is N
2
(I
2
I
1
).
Total Wt. of Cu in auto-transformer (N
1
N
2
) I
1
+ N
2
(I
2
I
1
)
If a two-winding transformer were to perform the same duty, then
Wt. of Cu on its primary N
1
I
1
; Wt. of Cu on secondary N
2
I
2
Total Wt. of Cu N
1
I
1
+ N
2
I
2
Wt. of Cu in auto-transformer
Wt. of Cu in ordinary transformer
=
121221
11 2 2
() ()
NNINII
NI N I
−+
+
= I
2
112
22
11
11
2
21
11 ;
2
1
N
NNI
K
KK
NI
NIK
NI

=− = = =


Q
Wt. of Cu in auto-transformer
(W
a
) = (1 K) × (Wt. of Cu in ordinary
transformer W
0
)
Saving = W
0
W
a
= W
0
(1 K) W
0
= KW
0
Saving = K × (Wt. of Cu in ordi-
nary transformer)
Hence, saving will increase as K ap-
proaches unity.
It can be proved that power trans-
formed inductively is input (1 K).
The rest of the power = (K × input) is
conducted directly from the source to the
load i.e., it is transferred conductively to
the load.
Fig. 32.60
V
2
I
1
I=(I I)
1c2
I
2
Load
C
B
I
1
A
I=(I I)
2c1
V
1
V
2
V
1
C
I
1
I
2
Load
I
2
I
1
I
2
Step up auto-transformer
* In fact, current flowing in the common winding of the auto-transformer is always equal to the difference
between the primary and secondary currents of an ordinary transformer.
(a) (b)
1188
Electrical Technology
Uses
As said earlier, auto-transformers are used when K is nearly equal to unity and where there is no
objection to electrical connection between primary and secondary. Hence, such transformers are
used :
1. to give small boost to a distribution cable to correct the voltage drop.
2. as auto-starter transformers to give upto 50 to 60 % of full voltage to an induction motor during
starting.
3. as furnace transformers for getting a convenient supply to suit the furnace winding from a 230-V
supply
4. as interconnecting transformers in 132 kV/330 kV system.
5. in control equipment for 1-phase and 3-phase electrical locomotives.
Example 32.87. An auto-transformer supplies a load of 3 kW at 115 volts at a unity power
factor. If the applied primary voltage is 230 volts, calculate the power transferred to the load
(a) inductively and (b) conductively. (Basic Elect. Machines, Nagpur Univ, 1991)
Solution. As seen from Art 32.33
Power transferred inductively = Input (1 K)
Power transferred conductively = Input × K
Now, K = 115/230 = 1/2, input output = 3 kW
Inductively transferred power = 3 (1 1/2) = 1.5 kW
Conductivley transferred power = (1/2) × 3 = 1.5 kW
Example 32.88. The primary and secondary voltages
of an auto-transformer are 500 V and 400 V respectively.
Show with the aid of diagram, the current distribution in the
winding when the secondary current is 100 A and calculate
the economy of Cu in this particular case.
Solution. The circuit is shown in Fig, 30.61.
K = V
2
/V
1
= 400/500 = 0.8
I
1
= KI
2
= 0.8 × 100 = 80 A
The current distribution is shown in Fig. 32.61.
Saving = KW
0
= 0.8 W
0
- Art 32.33
Percentage saving = 0.8 × 100 = 80
Example 32.89. Determine the core area, the number of turns and the position of the tapping
point for a 500-kVA, 50-Hz, single-phase, 6,600/5,000-V auto-transformer, assuming the following
approximate values : e.m.f. per turn 8 V. Maximum flux density 1.3 Wb/m
2
.
Solution. E = 4.44 f Φ
m
N volt
Φ
m
=
/8
4.44 4.44 50
EN
f
=
×
= 0.03604 Wb
Core area = 0.03604/1.3 = 0.0277 m
2
= 277 cm
2
Turns of h.v. side = 6600/8 = 825 ; Turns of L.V. side = 5000/8 = 625
Hence, tapping should be 200 turns from high voltage end or 625 turns from the common end.
32.34. Conversion of 2-Winding Transformer into Auto-transformer
Any two-winding transformer can be converted into an auto-transformer either step-down or
step-up. Fig. 32.62 (a) shows such a transformer with its polarity markings. Suppose it is a 20-kVA,
Fig. 32.61
500 V
100
A
20 A
100 A
400 V
80 A
80
A
Transformer
1189
2400/240 V transformer. If we employ additive polarity between the high-voltage and low-voltage sides,
we get a step-up auto-transformer. If, however, we use the subtractive polarity, we get a step-down auto-
transformer.
Fig. 32.62
(a) Additive Polarity
Connections for such a polarity are shown in Fig. 32.62 (b). The circuit is re-drawn in Fig. 32.62 (c)
showing common terminal of the transformer at the top whereas Fig. 32.62 (d) shows the same circuit
with common terminal at the bottom. Because of additive polarity, V
2
= 2400 + 240 = 2640 V and V
1
is
2400 V. There is a marked increase in the kVA of the auto-transformer (Ex. 32.90). As shown in
Fig. 32.62 (d), common current flows towards the common terminal. The transformer acts as a step-up
transformer.
(b) Subtractive Polarity
Such a connection is shown in Fig. 32.63 (a). The circuit has been re-drawn with common polarity
at top in Fig. 32.63 (b) and at bottom in Fig. 32.63 (c). In this case, the transformer acts as a step-down
auto-transformer.
Fig. 32.63
The common current flows away from the common terminal. As will be shown in Example 32.91, in
this case also, there is a very large increase in kVA rating of the auto-transformer though not as marked as
in the previous case. Here, V
2
= 2400 240 = 2160 V.
Example 32.90. For the 20-kVA, 2400/240-V two-winding step-down transformer shown in
Fig. 32.63 (a) connected as an auto-transformer with additive polarity as shown in Fig. 30.61 (d),
compute
1190
Electrical Technology
(i) original current capacity of HV-windings.
(ii) original current capacity of LV-windings.
(iii) k VA rating of auto-transformer using current capacity of current LV winding as calculated
in (ii) above.
(iv) per cent increase in kVA capacity of auto-transformer as compared to original two-winding
transformer.
(v) values of I
1
and I
c
in Fig. 30.61 (d) from value of I
2
used in (iii) above.
(vi) per cent overload of 2400-V winding when used as an auto-transformer.
(vii) comment on the results obtained.
Solution. (i) I
1
= 20 × 10
3
/2400 = 8.33 A (ii) I
2
= I
1
/K = 8.33 × 10 = 83.3 A
(iii) kVA rating of auto-transformer V
2
I
2
= 2640 × 83.3 × 10
3
= 220 kVA
(iv) Per cent increase in kVA rating =
220
100
20
×=
1100%
(v) I
1
= 220 × 10
3
/2400 = 91.7A, I
c
= I
1
I
2
= 91.7 83.3 = 8.4 A
(vi) Per cent overload of 2400 V winding = 8.4 × 100/8.33 = 100.8%
(vii) As an auto-transformer, the kVA has increased tremendously to 1100% of its original value with
LV coil at its rated current capacity and HV coil at negligible overload i.e. 1.008 × rated load.
Example 32.91. Repeat Example 30.64 for subtractive polarity as shown in Fig. 32.62 (c).
Solution. (i) I
1
= 8.33 A (ii) I
2
= 83.3 A
(iii) New kVA rating of auto-transformer is 2160 × 83.3 × 10
3
= 180 kVA
(iv) Per cent increase in kVA rating =
180
100
20
×=
900%
(v) I
1
= 180 × 10
3
/2400 = 75 A, I
c
= I
2
I
1
= 83.3 75 = 8.3 A
(vi) Per cent overload of 2400V winding = 8.3 × 100/8.33 = 100%
(vii) In this case, kVA has increased to 900% of its original value as a two-winding transformer with
both low-voltage and high-voltage windings carrying their rated currents.
The above phenomenal increase in kVA capacity is due to the fact that in an auto-transformer energy
transfer from primary to secondary is by both conduction as well as induction whereas in a
2-winding transformer it is by induction only. This extra conductive link is mainly responsible for the
increase in kVA capacity.
Example 32.92. A 5-kVA, 110/110-V, single-phase, 50-Hz transformer has full-load efficiency of
95% and an iron loss of 50 W. The transformer is now connected as an auto-transformer to a
220-V supply. If it delivers a 5-kW load at unity power factor to a 110-V circuit, calculate the
efficiency of the operation and the current drawn by the high-voltage side.
(Electric Machinery-II, Banglore Univ. 1991)
Solution. Fig. 32.64 (a)
shows the normal connection
for a 2-winding transformer. In
Fig. 32.64 (b) the same unit
has been connected as an auto
transformer. Since the two
windings are connected in se-
ries, voltage across each is 110
V.
The iron loss would
Fig. 32.64
110
V
Prim
ary
Secondary
110
V
(
)
2-W
inding
Transform
er
a
220
V
110
V
(
)
Auto-Transform
er
b
Load
Transformer
1191
remain the same in both connections. Since the auto-transformer windings will each carry but half the
current as compared to the conventional two-winding transformer, the copper loss will be one-fourth of the
previous value.
Two-winding Transformer
η = 0.95 0.95 =
output
5,000
output + losses 5,000 50 Cu loss
=
++
Cu loss = 212 W
Auto-transformer
Cu loss = 212/4 = 53 W; Iron loss = 50 W ∴ η =
5,000
0.9797 or . %
5,000 53 50
=
++
97 97
Current of the h.v. side = 5103/220 = 23.2 A
Example 32.93. A transformer has a primary voltage rating of 11500 volts and secondary volt-
age rating of 2300 volts. Two windings are connected in series and the primary is connected to a
supply of 11500 volts, to act as a step-up auto transformer. Determine the voltage output of the
transformer.
Question extended : If the two winding transformer is rated at 115 kVA, what will be the kVA
raitng of the auto-transformer ? (Madras University, 1997)
Solution. As in Fig. 32.65 (a), 115 kVA, 11500/2300 V, transformer has the current ratings of 10 A
and 50 A.
Referring to Fig. 32.65 (b), the step-up connections have been shown. Winding currents have to
be at the same rated values. As in Fig. 32.65 (b), the voltage obtainable at B
1
B
2
is 13800 V, and from
b
1
a load-current of 50 A can be supplied.
kVA rating = 13800 × 50 × 10
3
= 690
(b) 2 winding transformer Rating : 115kVA (b) Auto transformer
Fig. 32.65
Example 32.94. An 11500/2300 V transformer is rated at 100 kVA as a 2 winding transformer.
If the two windings are connected in series to form an auto-transformer, what will be the possible
voltage ratios ? (Manonmaniam Sundaranar Univ. April 1998)
Solution. Fig. 32.66 (a) shows this 2-winding transformer with
rated winding currents marked .
Rated current of 11.5 kV winding = 100 × 100/11500 = 8.7
Amp
Rated current of 2300 V winding = 43.5 Amp
Fig. 32.66 (b) and Fig. 32.66 (c) show autotransformer
Fig. 32.66 (
a
).
2 winding transformer
10A
11500/2300V
a
1
a
2
b
1
b
2
PRI SEC 50
am
p
1192
Electrical Technology
connections. On H.V. side, they have a rating of 13.8 kV. On L.V. side, with connection as in Fig. 32.66
(b), the rating is 2300 V. On L.V. side of Fig. 32.66 (c), the output is at 11.5 kV.
Thus, possible voltage ratios are : 13800/2300 V and 13800/11500 V.
With both the connections, step-up or step-down versions are possible.
Extension of Question : Calculate kVA ratings in the two cases.
Fig. 32.66
Windings will carry the rated currents, while working out kVA outputs.
In Fig. 32.66 (b), Input current (into terminal A
1
of windings A
1
-A
2
) can be 8.7 Amp with H.V.-side-
voltage ratings as 13.8 kV. Transformation ratio = 13800/2300 = 6
Hence, kVA rating = 13.8 × 8.7 = 120
Output current = 120 × 1000/2300 = 52.2 Amp
Current in the winding B
1
-B
2
= Difference of Output current and Input current
= 52.2 8.7 = 43.5 A, which is the rated current of the winding B
1
-B
2
.
In Fig. 32.66 (c). Similarly, transformation ratio = 13800/11500 = 1.2
kVA rating = 13800 × 43.5 × 10
`3
= 600
Output current = 600 × 1000/11500 = 52.2 Amp
Current carried by common winding = 52.2 43.5 = 8.7 A, which is rated current for the winding
A
1
-A
2
. Thus, with the same two windings give, a transformation ratio closer to unity gives higher kVA
rating as an auto transformer.
Thus, a 100 kVA two winding transformer is reconnected as an autotransformer of 120 kVA with
transformation ratio as 6, and becomes a 600 kVA autotransformer with transformation ratio as 1.2.
Example 32.95. A two-winding transformer is
rated at 2400/240 V, 50-kVA. It is re-connected as a
step-up auto-transformer, with 2400 V input. Calcu-
late the rating of the auto-transformer and the induc-
tively and conductively transferred powers while deliv-
ering the rated output at unity power-factor.
(Nagpur University, Winter 1999)
Solution. With 50 kVA as the rating, the rated currents
on the two sides are 20.8 A (2400-V side) and 208 A
(240 V side). With the required re-connection, the 2400-
V winding will work as a common winding. As shown in
Fig. 32.67, the winding common to input and output can
carry 20.8 A, the output current can be 208 A with a
voltage of 2640 V, which means that the output of this auto
Fig. 32.67
Transformer
1193
transformer is (2640 × 208) = 550 kVA.
The corresponding input-current is
(208 × 2640/2400) = 229 A.
The ratio of turns in this case is given by
k = 2640/2400 = 1.1
With the step-up job, and k =1.1
Rating of Auto-transformer
Rating as two-winding transformer
=
1.1
~1 0.1
=
k
k
This gives the rating as auto-transformer of 550 kVA.
At unity power-factor, the rated load = 550 kW.
Out of this, the “inductively” transferred power
= Power handled by the common winding
= (2400 V) × (20.8 A) × 10
3
= 50 kW.
= Rated output as a two-winding transformer.
Remaining Power = 550 kW 50 kW = 500 kW.
This power of 500 kW is “conductively” transferred as is clear from the division of currents at the
input node. Out of the total current of 229 A from the source, 208 A goes straight to the output. The
remaining current of 20.8 A is through the common and inductive path, as marked in the
Fig. 32.67.
32.35. Parallel Operation of Single-phase Transformers
For supplying a load in excess of the rating of
an existing transformer, a second transformer may
be connected in parallel with it as shown in Fig.
32.68. It is seen that primary windings are
connected to the supply bus bars and secondary
windings are connected to the load bus-bars. In
connecting two or more than two transformers in
parallel, it is essential that their terminals of similar
polarities are joined to the same bus-bars as in Fig.
32.68. If this is not done, the two e.m.fs. induced
in the secondaries which are paralleled with incorrect
polarities, will act together in the local secondary
circuit even when supplying no load and will hence
produce the equivalent of a dead short-circuit as
shown in Fig. 32.69.
There are certain definite conditions which
must be satisfied in order to avoid any local circulating currents and to ensure that the transformers share the
common load in proportion to their kVA ratings. The conditions are :
1. Primary windings of the transformers should be suitable for the supply system voltage and fre-
quency.
2. The transformers should be properly connected with regard to polarity.
3. The voltage ratings of both primaries and secondaries should be identical. In other words, the
transformers should have the same turn ratio i.e. transformation ratio.
4. The percentage impedances should be equal in magnitude and have the same X/R ratio in order to
avoid circulating currents and operation at different power factors.
Fig. 32.68
1194
Electrical Technology
5. With transformers having different kVA ratings, the equivalent impedances should be
inversely proportional to the individual kVA rating if circulating currents are to be avoided.
Of these conditions, (1) is easily comprehended ; condition (2) is absolutely essential (otherwise
paralleling with incorrect polarities will result in dead short-circuit). There is some lattitude possible
with conditions (3) and (4). If condition (3) is not exactly satisfied i.e. the two transformers have
slightly different transformation or voltage ratios, even then parallel operation is possible. But due to
inequality of induced e.m.fs. in secondaries, there will be even on no-load, some circulating current between
them (and therefore between the primary windings also) when secondary terminals are connected in parallel.
When secondaries are loaded, this localized circulating current will tend to produce unequal loading condition.
Hence, it may be impossible to take full kVA output
from the parallel connected group without one of the
transformers becoming over-heated.
If condition (4) is not exactly satisfied i.e.
impedance triangles are not identical in shape and
size, parallel operation will still be possible, but the
power factors at which the two transformers operate
will be different from the power factor of the common
load. Therefore, in this case, the two transformers
will not share the load in proportion to their kVA
ratings.
It should be noted that the impedances of two
transformers may differ in magnitude and in quality (i.e. ratio of equivalent resistance to reactance). It is
worthwhile to distinguish between the percentage and numerical value of an impedance. For example,
consider two transformers having ratings in the ratio 1 : 2. It is obvious that to carry double the current, the
latter must have half the impedance of the former for the same regulation. For parallel operation, the
regulation must be the same, this condition being enforced by the very fact of their being connected in
parallel. It means that the currents carried by the two transformers are proportional to their ratings provided
their numerical impedances are inversely proportional to these ratings and their percentage impedances
are identical.
If the quality of the two percentage impedances is different (i.e. ratio of percentage resistance to
reactance is different), then this will result in divergence of phase angle of the two currents, with the
result that one transformer will be operating with a higher and the other with a lower power factor than
that of the combined load.
(a) Case 1. Ideal Case
We will first consider the ideal case of two transformers having the same voltage ratio and having
impedance voltage triangles identical in size and shape.
Let E be the no-load secondary voltage of each transformer and V
2
the terminal voltage ; I
A
and
I
B
the currents supplied by them and I-the total current, lagging behind V
2
by an angle φ (Fig. 32.70(a)
I
A
V
2
I
B
I
0
E
A
C
B
V
1
V
2
Z
B
Z
A
I
I
A
I
B
Load
E
E
Fig. 32.70 (a) Fig. 32.70 (b)
Fig. 32.69
V
1
V
1
Load Busbars
Primary Busbars
V
2
S
Transformer
1195
In Fig. 32.70 (b) a single triangle ABC represents the identical impedance voltage triangles of both the
transformers. The currents I
A
and I
B
of the individual transformers are in phase with the load current I and
are inversely proportional to the respective impedances. Following relations are obvious.
I = I
A
+ I
B
; V
2
= E I
A
Z
A
= E I
B
Z
B
= E IZ
AB
Also I
A
Z
A
= I
B
Z
B
or I
A
/I
B
= Z
B
/Z
A
I
A
= IZ
B
/(Z
A
+ Z
B
) and I
B
= IZ
A
/(Z
A
+ Z
B
)
(b) Case 2. Equal Voltage Ratios
Let us assume that no-load voltages of both secondaries is the same i.e. E
A
= E
B
= E, and that the two
voltages are coincident i.e. there is no phase difference between E
A
and E
B
, which would be true if the
magnetising currents of the two
transformers are not much different
from each other. Under these
conditions, both primaries and
secondaries of the two transformers
can be connected in parallel and there
will circulate no current between them
on on-load.
However, if we neglect the
magnetising admittances, the two transformers can be connected as shown by their equivalent circuits in Fig.
32.71. The vector diagram is shown in Fig. 32.72.
From Fig. 32.71 (a) or (b) it is seen that it represents two impedances in parallel. Considering all
values consistently with reference to secondaries, let
Z
A
, Z
B
= impedances of the transformers
I
A
, I
B
= their respective currents
V
2
= common terminal voltage
I = combined current
It is seen that I
A
Z
A
= I
B
Z
B
= IZ
AB
...(i)
where Z
AB
is the combined impedance of Z
A
and Z
B
in parallel.
1/Z
AB
=1/Z
A
+ 1/Z
B
Hence Z
AB
= Z
A
Z
B
/(Z
A
+ Z
B
) ...(ii)
From equation (i), we get
Z
A
= IZ
AB
Z
A
= IZ
B
/(Z
A
+ Z
B
) and I
B
= IZ
AB
/Z
B
= IZ
A
/ (Z
A
+ Z
B
)
Multiplying both sides by common terminal voltage V
2
, we have
V
2
I
A
=
B
2
AB
+
Z
VI
ZZ
; Similarly V
2
I
B
=
A
2
AB
+
Z
VI
ZZ
Let V
2
I ×10
3
= S-the combined load kVA. Then, the kVA carried by each transformer is
B
A
AB
=
+
Z
SS
ZZ
=
AB
1
1+ /
S
ZZ
and
A
B
AB BA
1
=
+1+/
=
Z
SS S
ZZ ZZ
...(iii)
Hence, S
A
and S
B
are obtained in magnitude as well as in phase from the above vectorial
expressions.
The above problem may be solved graphically, although somewhat more laboriously. As shown in Fig.
32.72, drawn I
A
and I
B
with an angular difference of (φ
A
−φ
B
) and magnitude (according to some suitable
scale) inversely proportional to the respective impedances. Vector sum of I
A
and I
B
gives total combined
current I. The phase angle and magnitude of I will be known from the conditions of loading, so that angle φ
Fig. 32.72
0
E
=
E
=
E
AB
V
2
I
A
I
B
I
A
B
IZ =
I
Z
AA BB
IX
AA
IR
AA
IR
BB
IX
BB
Fig. 32.71
V
2
V
2
V
1
I=
I
+I
AB
E
=
E
=E
AB
()a ()b
Z
B
Z
B
I
B
E
B
E
A
I
A
I
A
I
B
Z
A
Z
A
Load
Load
I
1196
Electrical Technology
between V
2
and I will be known. Inserting this, the transformer currents I
A
and I
B
become known in
magnitude and phase with respect to V
2
.
Note. (a) In equation (iii) above, it is not necessary to use the ohmic values of resistances and reactances,
because only impedance ratios are required.
(b) The two percentage impedances must be adjusted to the same kVA in the case of transformers of
different rating as in Ex. 32.101.
(c) From equation (iii) above, it is seen that if two transformers having the same rating and the same
transformation ratio are to share the load equally, then their impedances should be equal i.e. equal resistances and
reactances and not numerical equality of impedances. In general, for transformers of different ratings but same
transformation ratio, their equivalent impedances must be inversely proportional to their ratings if each transformer
is to assume a load in proportion to its rating. For example, as said earlier, a transformer operating in parallel with
another of twice the rating, must have an impedance twice that of the large transformer in order that the load may
be properly shared between them.
Example 32.96. Two 1-phase transformers with equal turns have impedances of (0.5 + j3) ohm
and (0.6 + j10) ohm with respect to the secondary. If they operate in parallel, determine how they
will share a total load of 100 kW at p.f. 0.8 lagging ? (Electrical Technology, Madras Univ. 1987)
Solution. Z
A
= 0.5 + j 3 = 3.04 80.6° Z
B
= 0.6 + j 10 = 10.02 86.6°
Z
A
+ Z
B
=1.1 + j13 = 13.0585.2°
Now, a load of 100 kW at 0.8 p.f. means a kVA of 100/0.8 = 125. Hence,
S = 125 ∠ − 36.9°
S
A
=
A
AB
+
Z
S
ZZ
=
125 36.9 10.02 86.6
13.05 85.2
∠− °× °
∠°
= 96 ∠ − 35.5°
= a load of 96 × cos 35.5° = 78.2 kW
A
B
AB
=
+
Z
SS
ZZ
=
125 36.9 3.04 80.6
13.05 85.2
∠− °× °
∠°
= 29.1 ∠− 41.5°
= a load of 29.1 × cos 41.5° = 21.8 kW
Note. Obviously, transformer A is carrying more than its due share of the common load.
Example 32.97. Two single-phase transformers A and B are connected in parallel. They have
same kVA ratings but their resistances are respectively 0.005 and 0.01 per unit and their leakage
reactances 0.05 and 0.04 per unit. If A is operated on full-load at a p.f. of 0.8 lagging, what will be
the load and p.f. of B ? (A.C. Machines-I, Jadavpur Univ. 1985)
Solution. In general, S
A
=
B
AB
+
Æ
S
ZZ
and S
B
=
A
AB
+
Æ
S
ZZ
where S is the total kVA supplied and Z
A
and Z
B
are the percentage impedances of these transformers.
B
A
S
S
=
A
B
Z
Z
Now, Z
A
= 0.005 + j 0.05 per unit ; % Z
A
= 0.5 + j 5 and %Z
B
= 1 + j 4
Let S
A
=S
A
∠ − 36.87°
where S
A
represents the rating of transformer A (and also of B).
S
B
= S
A
∠− 36.87° ×
(0.5 5)
(1 4)
j
j
+
+
= S
A
∠− 36.87°×
20.7
17
8.3° = 1.22 S
A
∠− 28.57°
Transformer
1197
It is obvious that transformer B is working 22% over-load and its power factor is
cos 28.57° = 0.878 (lag)
Example 32.98. Two 1-phase transformers A and B rated at 250 kVA each are operated in
parallel on both sides. Percentage impedances for A and B are (1 + j 6) and (1.2 + j 4.8) respectively.
Compute the load shared by each when the total load is 500 kVA at 0.8 p.f. lagging.
(Electrical Machines-II, Indore Univ. 1989)
Solution.
A
AB
+
Æ
ZZ
=
16
2.2 10.8
j
j
+
+
=0.552.1° ;
B
AB
1.2 4.8
+ 2.2 10.8
j
j
+
=
+
Æ
ZZ
= 0.45∠− 2.5°
B
A
AB
+
=
Æ
SS
ZZ
= 500∠− 36.9° × 0.45∠− 2.5° = 225
∠−∠−
∠−∠−
∠−39.4°
A
B
AB
+
=
Æ
SS
ZZ
=500∠− 36.9° × 0.552.1° = 275
∠−∠−
∠−∠−
∠− 34.8°
Obviously, transformer B is overloaded to the extent of (275 250) × 100/250 = 10%. It carries
(275/500) × 100 = 55% of the total load.
Example 32.99. Two 100-kW, single-phase transformers are connected in parallel both on the
primary and secondary. One transformer has an ohmic drop of 0.5% at full-load and an inductive
drop of 8% at full-load current. The other has an ohmic drop of 0.75% and inductive drop of 2%.
Show how will they share a load of 180 kW at 0.9 power factor.
(Elect. Machines-I, Calcutta Univ. 1988)
Solution. A load of 180 kW at 0.9 p.f. means a kVA of 180/0.9 = 200
Load S = 200 25.8°
1
12
+
Æ
ZZ
=
22
(0.5 8) (0.5 8)(1.25 12)
(1 .2 5 1 2)
1.25 12
jjj
j
++
=
+
+
=
96.63 4
96.65 2.4
145.6 145.6
j+
∠°
=
= 0.6642.4°
2
12
+
Æ
ZZ
=
(0.75 4)(1.25 12)
145.6
jj+−
=
48.94 4 49.1 5
145.6 145.6
j−∠°
=
= 0.337∠−
S
1
=
2
12
+
Z
S
ZZ
= 200∠− 25.8° × 0.337 ∠−5° = 67.4∠−30.8°
kW
1
= 67.4 × cos 30.8° = 67.4 × 0.859 = 57.9 kW
S
2
= 200 ∠−25.8° × 0.664 2.4° = 132.8 ∠−23.4
kW
2
= 132.8 × cos 23.4° = 132.8 × 0.915 = 121.5 kW
Note. Second transformer is working 21.5% over-load. Also, it shares 65.7% of the total load.
Example 32.100. A load of 200 kW at 0.85 power factor lagging is to be shared by two transform-
ers A and B having the same ratings and the same transformation ratio. For transformer A, the full-
load resistive drop is 1% and reactance drop 5% of the normal terminal voltage. For transformer
B the corresponding values are : 2% and 6%. Calculate the load kVA supplied by each transformer.
Solution. Z
A
=1 + j 5; Z
B
= Z + j 6
1198
Electrical Technology
15
26
j
j
+
=
+
A
B
Z
Z
=0.8 + j 0.1 ;
A
26
15
j
j
+
=
+
B
Z
Z
= 1.23 j 0.514
Load kVA = kW/p.f. = 200/0.85 = 235
kVA = S = 235(0.85 j 0.527)
= 200 j 123.8 ( cos φ = 0.85; sin φ = 0.527)
S
A
=
B
AB AB
Z
1
S=S
Z+Z 1+(Z/Z)
=
200 123.8 200 123.8
1 (0.8 0.1) 1.8 0.1
jj
jj
−−
=
++ +
= 107.3 j 74.9
S
A
=
22
(107.3 74.9 )
+
= 131; cos φ
A
= 107.3/131 = 0.82 (lag)
Similarly S
B
=
A
AB
Z
S
Z+Z
=
BA
1
S
1+(Z /Z )
=
200 123.8 200 123.8
1 (1.23 0.514) 2.23 0.154
jj
jj
−−
=
+−
= 93 j 49
S
B
=
22
(93 49 )
+
= 105; cos φ
B
= 90/105 = 0.888 (lag)
(As a check, S = S
A
+ S
B
= 131 + 105 = 236. The small error is due to approximations made in
calculations.)
Example 32.101. Two 2,200/110-V, transformers are operated in parallel to share a load of
125 kVA at 0.8 power factor lagging. Transformers are rated as below :
A : 100 kVA ; 0.9% resistance and 10% reactance
B : 50 kVA ; 1.0% resistance and 5% reactance
Find the load carried by each transformer. (Elect. Technology, Utkal Univ. 1989)
Solution. It should be noted that the percentages given above refer to different ratings. As
pointed out in Art. 32.35, these should be adjusted to the same basic kVA, say, 100 kVA.
% Z
A
= 0.9 + j 10 ; % Z
B
= (100/50)(1 + j 5) = (2 + j 10)
A
AB
Z
Z+Z
=
22
0.9 10 (0.9 10) (2.9 20)
(2.9 20)
2.9 20
jjj
j
++
=
+
+
=
(202.6 11)
202.9
408.4 08.4
j+
∠3.1°
=
4
= 0.4968 3.1°
B
AB
Z
Z+Z
=
(2 10)(2.9 20) 206 11
408.4 08.4
jj j+−
=
4
=
206.1
408.4
∠ − 3.1°
= 0.504 ∠− 3.1°
Also cos φ = 0.8, φ = cos
1
(0.8) = 36.9°
S
A
=
B
AB
Z
S
Z+Z
= 125 ∠− 36.9° × 0.504 ∠− 3.1° = 63 ∠ − 40°
S
B
=
A
AB
Z
S
Z+Z
= 125 ∠− 36.9° × 0.4968 3.1° = 62.1 ∠ − 33.8°
Transformer
1199
Example 32.102. A 500-kVA transformer with 1% resistance and 5% reactance is connected in
parallel with a 250-kVA transformer with 1.5% resistance and 4% reactance. The secondary
voltage of each transformer is 400 V on no-load. Find how they share a load of 750-kVA at a p.f. of
0.8 lagging. (Electrical Machinery-I, Madras Univ. 1987)
Solution. It may be noted that percentage drops given above refer to different ratings. These should
be adjusted to the same basic kVA i.e. 500 kVA.
% Z
A
= 1 + j 5 = 5.1 78.7° ; % Z
B
=
()
500
250
(1.5 + j 4) = 3 + j 8 = 8.55 69.4°
%(Z
A
+Z
B
) =4 + j 13 = 13.6 72.9° ; S = 750 ∠− 36.9°
S
A
=
B
AB
Z
S
Z+Z
=
750 36.9 8.55 69.4
13.6 72.9
∠− °× °
∠°
= 470 ∠ − 40.4°
= 470 kVA at p.f. of 0.762 lagging
S
B
=
A
AB
Z
S
Z+Z
=
750 36.9 5.1 78.7
13.6 72.9
∠− °× °
∠°
= 280 ∠ − 31.1°
= 280 kVA at p.f. 0.856 lagging
Note. The above solution has been attempted vectorially, but in practice, the angle between I
A
and I
B
is so
small that if instead of using vectorial expressions, arithmetic expressions were used, the answer would not be
much different. In most cases, calculations by both vectorial and arithmetical methods generally yield results that
do not differ sufficiently to warrant the more involved procedure by the vector solution. The above example will
now be attempted arithmetically :
Z
A
= 5.1 , Z
B
= 8.55 ; I
A
/I
B
= Z
B
/Z
A
= 8.55/5.1 = 1.677 I
A
= 1.677 I
B
Total current = 750,000/400 = 1875 A
I = I
A
+ I
B
; 1875 = 1.677I
B
+ I
B
= 2.677 I
B
I
B
= 1875/2.677
S
B
= 400 × 1875/2.677 × 100 = 280 kVA
I
A
= 1.677 × 1875/2.677
S
A
= 400 × 1.677 × 1875/2.677 × 1000 = 470 kVA
Example 32.103. Two single-phase transformers A and B of equal voltage ratio are running in
parallel and supply a load of 1000 A at 0.8 p.f. lag. The equivalent impedances of the two transformers
are (2 + j3) and (2.5 + j5) ohms respectively. Calculate the current supplied by each transformer
and the ratio of the kW output of the two transformers.
(Electrical Machines-I, Bombay Univ. 1986)
Solution. Z
A
= (2 + j 3), Z
B
= (2.5 + j 5)
Now, =
AB
BA
IZ
IZ
=
2.5 5
23
j
j
+
+
= (1.54+ j 0.2) ; I
A
= I
B
(1.54 + j 0.2)
Taking secondary terminal voltage as reference vector, we get
I = 1000(0.8 j0.6) = 800 j 600 = 200 (4 j 3)
Also, I = I
A
+ I
B
= I
B
(1.54 + j 0.2) + I
B
= I
B
(2.54 + j 0.2)
200(4 j 3) = I
B
(2.54 + j 0.2) ; I
B
= 294.6 j 259.5 = 392.6
∠−∠−
∠−∠−
∠− 41.37°
I
A
= I
B
(1.54 + j 0.2) = (294.6 j 259.5) (1.54 + j 0.2) = 505.6 j 340.7
= 609.7 ∠− 33.95°
The ratio of the kW output is given by the ratio of the in-phase components of the two currents.
output of
output of
A
B
=
505.6 1.7
294.6 1
=
Note. Arithmetic solution mentioned above could also be attempted.
1200
Electrical Technology
Example 32.104. Two transformers A and B, both of no-load ratio 1,000/500-V are connected
in parallel and supplied at 1,000 V. A is rated at 100 kVA, its total resistance and reactance being
1% and 5% respectively, B is rated at 250 kVA, with 2% resistance and 2% reactance. Determine
the load on each transformer and the secondary voltage when a total load of 300 kVA at 0.8 power
factor lagging is supplied.
Solution. Let the percentage impedances be adjusted to the common basic kVA of 100.
Then %Z
A
=(1+ j 5) ; %Z
B
= (100/250)(2 + j 2) = (0.8 + j 0.8)
A
AB
Z
Z+Z
=
15
(1 .8 5 .8)
j
j
+
+
= 0.839 5.9
B
AB
Z
Z+Z
=
0.8 0.8
(1 .8 5 .8)
j
j
+
+
= 0.1865 ∠ − 27.6°
Now, S = 300 ∠ − 36.9° = 240 j 180
S
A
=
B
AB
Z
S.
Z+Z
= 300 ∠− 36.9° × 0.1865 ∠ − 27.6° = 55.95 ∠ − 64.5°
S
B
=
A
AB
Z
S
Z+Z
= 300 ∠ − 36.9° × 0.839 5.9° = 251.7 ∠ − 31°
Since Z
A
and Z
B
are in parallel, their combined impedance on 100 kVA basis is
Z
AB
=
AB
AB
ZZ
Z+Z
=
(1 5)(0.8 0.8)
(1 .8 5 .8)
jj
j
++
+
= 0.6 + j 0.738
Percentage drop over Z
AB
is = (0.6 × 240/100) + (0.738 × 180/100) = 1.44 + 1.328 = 2.768%
V
2
=
500 2.768
500
100
×



= 486.12 V
Example 32.105. Two 1-φ transformers are connected in parallel at no-load. One has a turn
ratio of 5,000/440 and a rating of 200 kVA, the other has a ratio of 5,000/480 and a rating of
350 kVA. The leakage reactance of each is 3.5%.
What is the no-load circulation current expressed as a percentage of the nominal current of the
200 kVA transformer.
Solution. The normal currents are
200 × 10
3
/440 = 455 A and 350 × 10
3
/480 = 730 A
Reactances seen from the secondary side are
3.5 440
100 455
×
= 0.034 ,
3.5 480
100 730
×
= 0.023
The difference of induced voltage is 40 V. The circulating current is
I
C
= 40/0.057 = 704 A = 1.55 times the normal current of 200 kVA unit.
Tutorial Problems 32.6
1. Two single-phase transformers A and B of equal voltage ratio are running in parallel and supplying a
load requiring 500 A at 0.8 power factor lagging at a terminal voltage of 400 V. The equivalent
impedances of the transformers, as referred to secondary windings, are (2 + j 3) and (2.5 + j 5) ohm.
Calculate the current supplied by each transformer.
(Note. The student is advised to try by arithmetic method also). [I
A
= 304 A ; I
B
= 197 A]
2. Two single-phase transformers A and B are operating in parallel and supplying a common load of
1000 kVA at 0.8 p.f. lagging. The data regarding the transformers is as follows :
Transformer
1201
Transformer Rating %Resistance % Reactance
A 750 kVA 3 5
B 500 kVA 2 4
Determine the loading of each transformer. [S
A
= 535
∠− ∠−
∠− ∠−
∠− 34.7° ; S
B
= 465
∠−∠−
∠−∠−
∠− 39.3°]
3. Two transformers A and B give the following test results. With the low-tension side short-circuited,
A takes a current of 10 A at 200 V, the power input being 1000 W. Similarly, B takes 30 A at 200 V
; the power input being 1,500 W. On open circuit, both transformers give a secondary voltage of 2200
when 11,000 volts are applied to the primary terminals. These transformers are connected in parallel
on both high tension and low tension sides Calculate the current and power in each transformer when
supplying a load of 200 A at 0.8 power factor lagging. The no-load currents may be neglected.
(Hint : Calculate Z
A
and Z
B
from S.C. test data)
[I
A
= 50.5 A, P
A
= 100 kW ; I
B
= 151 A, P
B
= 252 kW] (London University)
4. Two 6600/250-V transformers have the following short-circuit characteristics : Applied voltage
200 V, current 30 A, power input 1,200 W for one of the transformers ; the corresponding data for the
other transformer being 120 V, 20 A and 1,500 W. All values are measured on the H.V. side with the
L.V. terminals short circuited. Find the approximate current and the power factor of each transformer
when working in parallel with each other on the high and low voltage sides and taking a total load of
150 kW at a p.f. of 0.8 lagging from the high voltage bus-bars.
[I
A
= 13.8 A, cos
φφ
φφ
φA = 0.63 ; I
B
= 15.35 A , cos
φφ
φφ
φB = 0.91] (Electrical Engg-IV, Baroda Univ. 1978)
5. Two 11,000/2,200-V, 1-phase transformers are connected in parallel to supply a total load of 200 at 0.8
p.f. lagging at 2,200 V. One transformer has an equivalent resistance of 0.4 and equivalent reac-
tance of 0.8 referred to the low-voltage side. The other has equivalent resistance of 0.1 and a
reactance of 0.3 . Determine the current and power supplied by each transformer.
[52 A ; 148 A ; 99 A ; 252 kW]
6. A 2,000-kVA transformer (A) is connected in parallel with a 4,000-kVA transformer (B) to supply a
3-phase load of 5,000 kVA at 0.8 p.f. lagging. Determine the kVA supplied by each transformer
assuming equal no-load voltages. The percentage volt drops in the windings at the rated loads are as
follows :
Transformer A : resistance 2% ; reactance 8%
Transformer B : resistance 1.6% ; reactance 3%
[A : 860 kVA, 0.661 lag ; B : 4170 kVA, 0.824 lag] (A.C. Machines-I, Jadavpur Univ. 1979)
7. Two single-phase transformers work in parallel on a load of 750 A at 0.8 p.f. lagging. Determine
secondary voltage and the output and power factor of each transformer. Test data are :
Open circuit : 11,00/13,300 V for each transformer
Short circuit : with h.v. winding short-circuit
Transformer A : secondary input 200 V, 400 A, 15 kW
Transformer B : secondary input 100 V, 400 A, 20 kW
[3,190 VA : 80 kVA, 0.65 lag ; B : 1,615 kVA ; 0.86 lag]
(c) Case 3. Unequal Voltage Ratios
In this case, the voltage ratios (or transformation
ratios) of the two transformers are different. It means
that their no-load secondary voltages are unequal. Such
cases can be more easily handled by phasor algebra
than graphically.
Let E
A
, E
B
= no-load secondary e.m.f.s of the
two transformers.
Z
L
= load impedance across the secondary.
The equivalent circuit and vector diagram are also
shown in Fig. 32.73 and 32.74.
Fig. 32.73
V
1
V
2
Z
B
Z
A
I=I +I
AB
I
A
I
B
E
A
E
B
Load
Z
L
1202
Electrical Technology
It is seen that even when secondaries are on no-load, there will be some cross-current in them because
of inequality in their induced e.m.fs. This circulating current I
C
is given by
I
C
= (E
A
E
B
)/(Z
A
+ Z
B
) ...(i)
As the induced e.m.fs. of the two transformers are equal to the total drops in their respective circuits.
Z
A
=I
A
Z
A
+ V
2
; E
B
= I
B
Z
B
+ V
2
Now, V
2
=IZ
L
= (I
A
+ I
B
) Z
L
where Z
L
= load impedance
E
A
= I
A
Z
A
+ (I
A
+ I
B
) Z
L
...(ii)
E
B
= I
B
Z
B
+ (I
A
+ I
B
) Z
L
...(iii)
E
A
E
B
=I
A
Z
A
I
B
Z
B
...(iv)
I
A
= [(E
A
E
B
) + I
B
Z
B
]/Z
A
Substituting this value of I
A
in equation (iii),
we get
E
B
=I
B
Z
B
+ [{(E
A
E
B
) + I
B
Z
B
}/Z
A
+I
B
]/Z
L
I
B
=[E
B
Z
A
(E
A
E
B
)Z
L
]/[Z
A
Z
B
+ Z
L
.(Z
A
+ Z
B
)] ...(v)
From the symmetry of the expression, we get
I
A
= [E
A
Z
B
+ (E
A
E
B
)Z
L
[Z
A
Z
B
+ Z
L
(Z
A
+ Z
B
)] ...(vi)
Also, I = I
A
+ I
B
=
Á Â Â Á
Á Â L Á Â
Å Æ + Å Æ
Æ Æ + Æ + Æ )
By multiplying the numerator and denominator of this equation by 1/Z
A
Z
B
and the result by Z
L
we get
V
2
= IZ
L
=
/
Á A Â B
Á B L
ÅÆ+ Å Æ
1/
Æ + 1/Æ + 1/ Æ
The two equations (v) and (vi) then give the values of secondary currents. The primary currents may
be obtained by the division of transformation ratio i.e. K and by addition (if not negligible) of the no-load
current. Usually, E
A
and E
B
have the same phase (as assumed above) but there may be some phase
difference between the two due to some difference of internal connection in parallel of a star/star and a star/
delta 3-phase transformers.
If Z
A
and Z
B
are small as compared to Z
L
i.e. when the transformers are not operated near short-
circuit conditions, then equations for I
A
and I
B
can be put in a simpler and more easily under-standable
form. Neglecting Z
A
Z
B
in comparison with the expression Z
L
(Z
A
+ Z
B
), we have
I
A
=
)
+
Á B A B
LA B A B
Å Æ Å E
Æ + Æ Z + Z
...(vii)
I
B
=
)
BA A B
LA B A B
Å Æ Å E
Æ + Æ Z + Z
...(viii)
The physical interpretation of the second term in equations (vii) and (viii) is that it represents the
cross-current between the secondaries. The first term shows how the actual load current divides
between the loads. The value of current circulating in transformer secondaries (even when there is no-
load) is given by* I
C
= (E
A
E
B
)/(Z
A
+ Z
B
) assuming that E
A
> E
B
. It lags behind E
A
by an angle α
given by tan α = (X
A
+ X
B
)/(R
A
+ R
B
). If E
A
+ E
B
the ratios of the currents are inversely as the
impedances (numerical values).
* Under load conditions, the circulating current is
I
C
=
/
AB
ABABL
EE
ZZZZZ
++
If Z
L
= i.e. on open-circuit, the expression reduces to that given above.
Fig. 32.74
E
B
I
B
I
A
E
A
V
2
IR
AA
IX
AA
IX
BB
IZ
BB
IR
BB
IZ
AA
0
B
A
I
Transformer
1203
If in Eq. (iv) we substitute I
B
= I
I
A
and simplify, we get
I
A
=
BAB
AB AB
IZ E E
+
Z+Z Z+Z
...(ix)
Similarly, if we substitute I
A
= I
I
B
and simplify, then
I
B
=
AAB
AB AB
IZ E E
+
Z+Z Z+Z
...(x)
In a similar manner, value of terminal voltage V
2
is given by
V
2
= IZ
L
=
+−
AB BA AB
AB
EZ EZ IZZ
Z+Z
...(xi)
These expressions give the values of transformer currents and terminal voltage in terms of the load
current. The value of V
2
may also be found as under :
As seen from Fig. 30.70.
I
A
=(E
A
V
2
)/Z
A
= (E
A
V
2
)Y
A
;I
B
= (E
B
V
2
)Y
B
I=V
2
Y
L
= I
A
+ I
B
or V
2
Y
L
= (E
A
V
2
)Y
A
+ (E
B
V
2
)Y
B
V
2
(Y
L
+ Y
A
+ Y
B
)=E
A
Y
A
+ E
B
Y
B
or V
2
=
+
+
AA BB
LAB
EY EY
Y+Y Y
...(xii)
Eq. (xi) gives V
2
in terms of load current. But if only load kVA is given, the problem becomes more
complicated and involves the solution of a quadratic equation in V
2
.
Now, S = V
2
I. When we substitute this value of I in Eq. (xi), we get
V
2
=
()
+−
+
AB BA AB 2
AB
EZ EZ SZZ/V
ZZ
or V
2
2
(Z
A
+ Z
B
)
V(E
A
Z
B
E
B
Z
A
) + SZ
A
Z
B
= 0
When V
2
becomes known, then I
A
and I
B
may be directly found from
V
2
= E
A
I
A
Z
A
and V
2
= E
B
I
B
Z
B.
Note. In the case considered above, it is found more convenient to work with numerical values of imped-
ances instead of % values.
Example 32.106. Two transformers A and B are joined in parallel to the same load. Determine
the current delivered by each transformer having given : open-circuit e.m.f. 6600 V for A and 6,400
V for B. Equivalent leakage impedance in terms of the secondary = 0.3 + j3 for A and 0.2 + j1 for B.
The load impedance is 8 + j6. (Elect. Machines-I, Indore Univ. 1987)
Solution. I
A
=
()
+−
+
AB A B L
AB L A B
EZ E E Z
ZZ Z(Z +Z)
Here E
A
= 6,600 V ; E
B
= 6,400 V ; Z
L
= 8 + j 6; Z
A
= 0.3 + j 3 ; Z
B
= 0.2 + j 1
I
A
=
6600(0.2 1) (6600 6400) (8 6)
(0.3 3)(0.2 1) (8 6) (0.3 3 0.2 1)
++ +
++++ +++
jj
jj j j j
117 j 156 = 195A in magnitude
Similarly, I
B
=
()
−−
+
BA A B L
AB L A B
EZ E E Z
ZZ Z(Z +Z)
=
6400 (0.3 3) (6600 6400) (8 6)
(0. 3 3) (0. 2 1) (8 6 ) (0.5 4)
jj
jjjj
+− +
+++ +++
= 349 j 231 = 421 A (in magnitude)
1204
Electrical Technology
Example 32.107. Two 1-φ transformers, one of 100 kVA and the other of 50 kVA are connected
in parallel to the same bus-bars on the primary side, their no-load secondary voltages being 1000 V
and 950 V respectively. Their resistances are 2.0 and 2.5 per cent respectively and their reactances 8
and 6 percent respectively. Calculate no-load circulating current in the secondaries.
(Adv. Elect Machines, A.M.I.E. Sec. B, 1991)
Solution. The circuit connections are shown in Fig. 32.75.
Ist transformer
Normal secondary current = 100,000/1000 = 100 A
R
A
=
1000 2.0
100 100
×
×
= 0.2 ; X
A
=
1000 8
100 100
×
×
= 0.8
2nd Transformer
Normal secondary current = 50,000/950 = 52.63 A
R
B
= 950 × 2.5/100 × 52.63 = 0.45
X
B
= 850 × 6/100 × 52.63 = 1.08
Combined impedance of the two secondaries is
Z=
22
()( )
AB A B
RR X X
+++
=
22
0.65 1.88 1.99
+=
I
c
= (1000 950)/1.99 = 25.1 A ; α = tan
1
(1.88/
0.65) = 71°
Example 32.108. Two single-phase transformers, one of 1000-kVA and the other of 500-kVA
are connected in parallel to the same bus-bars on the primary side ; their no-load secondary voltages
being 500 V and 510 V respectively. The impedance voltage of the first transformer is 3% and that
of the second 5%. Assuming that ratio of resistance to reactance is the same and equal to 0.4 in
each. What will be the cross current when the secondaries are connected in parallel ?
(Electrical Machines-I, Madras Univ. 1985)
Solution. Let us first determine the ohmic value of the two impedances. Also, let the secondary
voltage be 480 V*.
Full-load I
A
= 1000 × 1000/480 = 2083 A ; F.L. I
B
= 500 × 1000/480 = 1042 A
Z
A
=
%
9500
100 100 2083
AA
A
ZE
I
×
×
=
××
= 0.0072
Z
B
=
5510
100 1042
×
×
= 0.0245
I
C
=
510 500
(0.0072 0.0245)
BA
AB
EE
ZZ
=
++
= 315.4 A
Note. Since the value of X/R is the same for the two transformers, there is no phase difference between E
A
and E
B
.
Example 32.109. Two transformers A and B of ratings 500 kVA and 250 kVA are supplying a
load kVA of 750 at 0.8 power factor lagging. Their open-circuit voltages are 405V and 415 V
respectively. Transformer A has 1% resistance and 5% reactance and transformer B has 1.5% resis-
tance and 4% reactance. Find (a) cross-current in the secondaries on no-load and (b) the load shared
by each transformer.
* Though it is chosen arbitrarily, its value must be less than either of the two no-load e.m.fs.
Fig. 32.75
E
A
I
C
Z
A
Z
B
E
B
I
C
Load Bus Bars
Supply Bus Bars
Transformer
1205
Solution. As said earlier, it is more convenient to work with ohmic impedances and for that purpose,
we will convert percentage value into numerical values by assuming 400 volt as the terminal voltage (this
value is arbitrary but this assumption will not introduce appreciable error).
Now I
A
R
A
= 1% of 400 R
A
=
1400
100 1250
×
= 0.0032
where I
A
= 500,000/400 = 1250 A
I
A
X
A
= 5% of 400 ; X
A
=
5 400
100 1250
×
= 0.016 (i.e. X
A
= 5R
A
)
In a similar way, we can find R
B
and X
B
; R
B
= 0.0096 ; X
B
= 0.0256
Z
A
= 0.0032 + j 0.016 = 0.0163 78.5° ; Z
B
= 0.0096 + j 0.0256 = 0.0275 69.4°
Z
A
+ Z
B
= 0.0128 + j 0.0416 = 0.0436 72.9°
Next step is to calculate load impedance. Let Z
L
be the load impedance and V
2
the terminal voltage
which has been assumed as 400 V.
(V
2
2
/Z
L
) = 750 ∠− 36.9°
Z
L
= 400
2
× 10
3
/750 ∠− 36.9° = 0.214 36.9° = (0.171 + j 0.128)
(a)I
C
=
AB
AB
EE
Z+Z
=
(405 415)
0.0436 72.9
∠°
= 230 ∠− 72.9°
(b)I
A
=
405 0.0275 69.4 (405 415) 0.214 36.9
0.0163 78.5 0.0275 69.4 0.214 36.9 0.0436 72.9
×∠°+×°
°×∠°+°×∠°
= 970 ∠ − 35°
Similarly, I
B
=
415 0.0163 78.5 (405 415) 0.214 36.9
0.0163 78.5 0.0275 69.4 214 369 0.0436 72.9
×∠°×°
∠°× ∠°+ °× ∠°
S
A
= 400 × 970 × 10
3
∠− 35° = 388 ∠− 35° kVA ; cos φ
A
= cos 35° = 0.82 (lag)
S
B
= 400 × 875 × 10
3
42.6° = 350 ∠− 42.6° kVA
cos φ
B
= cos 42.6° = 0.736 (lag)
Example 32.110. Two transformers A and B are connected in parallel to a load of (2 + j 1.5)
ohms. Their impedances in secondary terms are Z
A
= (0.15 + j 0.5) ohm and Z
B
= (0.1 + j 0.6) ohm.
Their no-load terminal voltages are E
A
= 207 0° volt and E
B
=205 0° volt. Find the power
output and power factor of each transformer. (Elect. Machines-I, Punjab Univ. 1991)
Solution. Using the equations derived in Art. 30.34 (c), we have
I
A
=
()
)
+−
AB A B L
AB L A B
EZ E E Z
ZZ +Z(Z +Z
Z
A
= (0.15 + j 0.5) ; Z
B
= (0.1 +j 0.6) ; Z
L
= (2 + j 1.5) = 2.5 36.9°
I
A
=
207(0.1 0.6) (207 205) (2 1.5)
(0.15 0.5)(0.1 0.6) (2 1.5)(0.25 1.1)
jj
jj j j
++ +
++++ +
=
24.7 127.2 129.7 79
1.435 2.715 3.07 117.9
j
j
+∠°
=
−+ °
= 42.26 ∠− 38.9°A = (32.89 j 26.55)A
I
B
=
()
)
−−
EZ E E Z
ZZ +Z(Z +Z
BA A B L
AB L A B
=
205(0.15 0.5) 2(2 1.5) 103 75
1.435 2.715 3.07 117.9
jj
j
+−+ °
=
−+ °
= 33.56 ∠ − 42.9° = (24.58 j 22.84) A
Now V
2
= IZ
L
= (I
A
+ I
B
) Z
L
= (57.47 j 49.39) (2 + j 1.5) = 189 j 12.58 = 189.4 ∠− 3.9°
1206
Electrical Technology
p.f. angle of transformer A = 3.9° (38.9°) = 35°
p.f. of A = cos 35° = 0.818 (lag) ; p.f. of B = cos [3.9° (42.9°)] = 0.776 (lag)
Power output of transformer A is P
A
= 189.4 × 42.26 × 0.818 = 6,548 W
Similarly, P
B
= 189.4 × 33.56 × 0.776 = 4,900 W
Example 32.111. Two transformers have the following particulars :
Transformer A Transformer B
Rated current 200 A 600 A
Per unit resistance 0.02 0.025
Per unit reactance 0.05 0.06
No-load e.m.f. 245 V 240 V
Calculate the terminal voltage when they are connected in parallel and supply a load impedance
of (0.25 + j 0.1) Ω.(Elect. Machines-I, Sd. Patel Univ. 1981)
Solution. Impedance in ohms = Z
pu
× N.L. e.m.f./full-load current
Z
A
= (245/200)(0.02 + j 0.05) = 0.0245 + j 0.0613 = 0.066 68.2°
Z
B
= (240/600)(0.025 + j 0.06) = 0.01 + j 0.024 = 0.026 67.3°
Z = (0.25 + j 0.1) = 0.269 21.8° ; Z
A
+ Z
B
= 0.0345 + j 0.0853 = 0.092 68°
Z
L
(Z
A
+ Z
B
) = 0.269 × 0.092 89.8° = 0.0247 89.8° = (0 + j 0.0247)
Z
A
Z
B
= 0.066 × 0.026 135.5° = (0.001225 + j 0.001201)
Z
A
Z
B
+ Z
L
(Z
A
+ Z
B
) = (
0.00125 + j 0.259) = 0.0259 92.7°
Let us take E
A
as reference quantity.
Also E
b
is in phase with E
A
because transformers are in parallel on both sides.
E
A
Z
B
= 245(0.01 + j 0.0245) = 2.45 + j 5.87
E
B
Z
A
= 240(0.0245 + j 0.0613) = 5.88 + j 14.7
E
A
Z
B
+ E
B
Z
A
= 8.33 + j 20.57 = 22.15 67.9°
Now, I =
22.15 67.9
0.0259 92.7
∠°
=
∠°
AB BA
AB L A B
EZ +EZ
Z Z +Z (Z +Z )
= 855 ∠ − 24.8°
V
2
= IZ
L
= 885 ∠ − 24.8° × 0.26921.8° = 230
Tutorial Problems 29.1
1. A 1000-kVA and a 500-kVA, 1-phase transformers are connected to the same bus-bars on the primary
side. The secondary e.m.fs. at no-load are 500 and 510 V respectively. The impedance voltage of the
first transformer is 3.4% and of the second 5%. What cross-current will pass between them when the
secondaries are connected together in parallel ? Assuming that the ratio of resistance to reactance is
the same in each, what currents will flow in the windings of the two transformers when supplying a
total load of 1200 kVA.
[(i) 290 A (ii) 1577 and 900 A] (City & Guilds, London)
2. Two transformers A and B are connected in parallel to supply a load having an impedance of
(2 + j 1.5). The equivalent impedances referred to the secondary windings are 0.15 + j 0.5 and
0.1 + j 0.6 respectively. The open-circuit e.m.f. of A is 207 V and of B is 205 V. Calculate (i) the
voltage at the load (ii) the power supplied to the load (iii) the power output of each transformer and
(iv) the kVA input to each transformer.
[(i) 189 ∠− 3.8°V (ii) 11.5 kW (iii) 6.5 kW, 4.95 kW (iv) 8.7 kVA, 6.87 kVA]
Transformer
1207
QUESTIONS AND ANSWERS ON TRANSFORMERS
Q.1. How is magnetic leakage reduced to a minimum in commerical transformers ?
Ans. By interleaving the primary and secondary windings.
Q.2. Mention the factors on which hysteresis loss depends ?
Ans. (i) Quality and amount of iron in the core (ii) Flux density and (iii) Frequency.
Q.3. How can eddy current loss be minimised ?
Ans. By laminating the core.
Q.4. In practice, what determines the thickness of the laminae or stampings ?
Ans. Frequency.
Q.5. Does the transformer draw any current when its secondary is open ?
Ans. Yes, no-load primary current.
Q.6. Why ?
Ans. For supplying no-load iron and copper losses in primary.
Q.7. Is Cu loss affected by power factor ?
Ans. Yes, Cu loss varies inversely with power factor.
Q.8. Why ?
Ans. Cu loss depends on current in the primary and secondary windings. It is well-known that current
required is higher when power factor is lower.
Q.9. What effects are produced by change in voltage ?
Ans. 1. Iron loss.........varies approximately as V
2
.
2. Cu loss.........it also varies as V
2
but decreases with an increase in voltage if constant kVA
output is assumed.
3. Efficiency.........for distribution transformers, efficiency at fractional loads decreases with in-
crease in voltage while at full load or overload it increases with increase in voltage and vice-
versa.
4. Regulation.........it varies as V
2
but decreases with increase in voltage if constant kVA output
is assumed.
5. Heating.........for constant kVA output, iron temperatures increase whereas Cu temperatures
decrease with increase in voltages and vice-versa.
Q. 10. How does change in frequency affect the operation of a given transformer ?
Ans. 1. Iron loss .........increases with a decrease in frequency. A 60-Hz transformer will have nearly
11% higher losses when worked on 50Hz instead of 60 Hz. However, when a 25-Hz trans-
former is worked on 60 Hz, iron losses are reduced by 25%.
2. Cu loss.........in distribution transformers, it is independent of frequecy.
3. Efficiency.........since Cu loss is unaffected by change in frequency, a given transformer effi-
ciency is less at a lower frequency than at a higher one.
4. Regulation.........regulation at unity power factor is not affected because IR drop is independent
of frequency. Since reactive drop is affected, regulation at low power factors decreases with
a decrease in frequency and vice-versa. For example, the regulation of a 25-Hz transformer
when operated at 50-Hz and low power factor is much poorer.
5. Heating.........since total loss is greater at a lower frequency, the temperature is increased with
decrease in frequency.
1208
Electrical Technology
OBJECTIVE TESTS – 32
1. A transformer transforms
(a) frequency
(b) voltage
(c) current
(d) voltage and current.
2. Which of the following is not a basic element of
a transformer ?
(a) core
(b) primary winding
(c) secondary winding
(d) mutual flux.
3. In an ideal transformer,
(a) windings have no resistance
(b) core has no losses
(c) core has infinite permeability
(d) all of the above.
4. The main purpose of using core in a transformer
is to
(a) decrease iron losses
(b) prevent eddy current loss
(c) eliminate magnetic hysteresis
(d) decrease reluctance of the common mag-
netic circuit.
5. Transformer cores are laminated in order to
(a) simplify its construction
(b) minimise eddy current loss
(c) reduce cost
(d) reduce hysteresis loss.
6. A transformer having 1000 primary turns is
connected to a 250-V a.c. supply. For a
secondary voltage of 400 V, the number of
secondary turns should be
(a) 1600 (b) 250
(c) 400 (d) 1250
7. The primary and secondary induced e.m.fs. E
1
and E
2
in a two-winding transformer are always
(a) equal in magnitude
(b) antiphase with each other
(c) in-phase with each other
(d) determined by load on transformer second-
ary.
8. A step-up transformer increases
(a) voltage (b) current
(c) power (d) frequency.
9. The primary and secondary windings of an
ordinary 2-winding transformer always have
(a) different number of turns
(b) same size of copper wire
(c) a common magnetic circuit
(d) separate magnetic circuits.
10. In a transformer, the leakage flux of each wind-
ing is proportional to the current in that winding
because
(a) Ohm’s law applies to magnetic circuits
(b) leakage paths do not saturate
(c) the two windings are electrically isolated
(d) mutual flux is confined to the core.
11. In a two-winding transformer, the e.m.f. per turn
in secondary winding is always.......the induced
e.m.f. power turn in primary.
(a) equal to K times
(b) equal to 1/K times
(c) equal to
(d) greater than.
12. In relation to a transformer, the ratio 20 : 1 indi-
cates that
(a) there are 20 turns on primary one turn on
secondary
(b) secondary voltage is 1/20th of primary
voltage
(c) primary current is 20 times greater than the
secondary current.
(d) for every 20 turns on primary, there is one
turn on secondary.
13. In performing the short circuit test of a trans-
former
(a) high voltage side is usually short circuited
(b) low voltage side is usually short circuited
(c) any side is short circuited with preference
(d) none of the above.
(Elect. Machines, A.M.I.E. Sec. B, 1993)
14. The equivalent resistance of the primary of a
transformer having K = 5 and R
1
= 0.1 ohm
when referred to secondary becomes.......ohm.
(a) 0.5
(b)0.02
(c) 0.004
(d) 2.5
15. A transformer has negative voltage regulation
when its load power factor is
(a) zero
(b) unity
(c) leading
(d) lagging.
Transformer
1209
16. The primary reason why open-circuit test is
performed on the low-voltage winding of the
transformer is that it
(a) draws sufficiently large on-load current for
convenient reading
(b) requires least voltage to perform the test
(c) needs minimum power input
(d) involves less core loss.
17. No-load test on a transformer is carried out to
determine
(a) copper loss
(b) magnetising current
(c) magnetising current and no-load loss
(d) efficiency of the transformer.
18. The main purpose of performing open-circuit
test on a transformer is to measure its
(a) Cu loss
(b) core loss
(c) total loss
(d) insulation resistance.
19. During short-circuit test, the iron loss of a
transformer is negligible because
(a) the entire input is just sufficient to meet Cu
losses only
(b) flux produced is a small fraction of the nor-
mal flux
(c) iron core becomes fully saturated
(d) supply frequency is held constant.
20. The iron loss of a transformer at 400 Hz is 10
W. Assuming that eddy current and hysteresis
losses vary as the square of flux density, the
iron loss of the transformer at rated voltage but
at 50 Hz would be....... watt.
(a)80 (b) 640
(c)1.25 (d) 100
21. In operating a 400 Hz transformer at 50 Hz
(a) only voltage is reduced in the same propor-
tion as the frequency
(b) only kVA rating is reduced in the same pro-
portion as the frequency
(c) both voltage and kVA rating are reduced in
the same proportion as the frequency
(d) none of the above.
22. The voltage applied to the h.v. side of a trans-
former during short-circuit test is 2% of its rated
voltage. The core loss will be.......percent of
the rated core loss.
(a)4 (b) 0.4
(c)0.25 (d)0.04
23. Transformers are rated in kVA instead of kW
because
(a) load power factor is often not known
(b) kVA is fixed whereas kW depends on load
p.f.
(c) total transformer loss depends on volt-
ampere
(d) it has become customary.
24. When a 400-Hz transformer is operated at 50
Hz its kVA rating is
(a) raduced to 1/8
(b) increased 8 times
(c) unaffected
(d) increased 64 times.
25. At relatively light loads, transformer efficiency
is low because
(a) secondary output is low
(b) transformer losses are high
(c) fixed loss is high in proportion to the output
(d) Cu loss is small.
26. A 200 kVA transformer has an iron loss of 1 kW
and full-load Cu loss of 2kW. Its load kVA cor-
responding to maximum efficiency is ....... kVA.
(a) 100 (b) 141.4
(c)50 (d) 200
27. If Cu loss of a transformer at 7/8th full load is
4900 W, then its full-load Cu loss would be
.......watt.
(a) 5600 (b) 6400
(c) 375 (d) 429
28. The ordinary efficiency of a given transformer
is maximum when
(a) it runs at half full-load
(b) it runs at full-load
(c) its Cu loss equals iron loss
(d) it runs slightly overload.
29. The output current corresponding to maximum
efficiency for a transformer having core loss of
100 W and equivalent resistance referred to
secondary of 0.25 is ....... ampere.
(a)20
(b)25
(c)5
(d) 400
30. The maximum efficiency of a 100-kVA trans-
former having iron loss of 900 kW and F.L. Cu
loss of 1600 W occurs at ....... kVA.
(a) 56.3 (b) 133.3
(c)75 (d) 177.7
1210
Electrical Technology
ANSWERS
1. d 2. d 3. d 4. d 5. b 6. a 7. c 8. a 9. c 10. b 11. c 12. d 13. b 14. d
15. c 16. a 17. c 18. b 19. b 20. b 21. b 22. d 23. c 24. a 25. c 26. b 27. b 28. c 29. a 30. c
31. d 32. d 33. c 34. a 35. b 36. a 37. d 38. c
31. The all-day efficiency of a transformer depends
primarily on
(a) its copper loss
(b) the amount of load
(c) the duration of load
(d) both (b) and (c).
32. The marked increase in kVA capacity produced
by connecting a 2 winding transformer as an
autotransfomer is due to
(a) increase in turn ratio
(b) increase in secondary voltage
(c) increase in transformer efficiency
(d) establishment of conductive link between
primary and secondary.
33. The kVA rating of an ordinary 2-winding
transformer is increased when connected as an
autotransformer because
(a) transformation ratio is increased
(b) secondary voltage is increased
(c) energy is transferred both inductively and
conductivity
(d) secondary current is increased.
34. The saving in Cu achieved by converting a
2-winding transformer into an autotransformer
is determined by
(a) voltage transformation ratio
(b) load on the secondary
(c) magnetic quality of core material
(d) size of the transformer core.
35. An autotransformer having a transformation ratio
of 0.8 supplies a load of 3 kW. The power
transferred conductively from primary to
secondary is.......kW.
(a) 0.6 (b) 2.4
(c) 1.5 (d)0.27
36. The essential condition for parallel opearation of
two 1-φ transformers is that they should have
the same
(a) polarity
(b) kVA rating
(c) voltage ratio
(d) percentage impedance.
37. If the impedance triangles of two transformers
operating in parallel are not identical in shape and
size, the two transformers will
(a) share the load unequally
(b) get heated unequally
(c) have a circulatory secondary current even
when unloaded
(d) run with different power factors.
38. Two transformers A and B having equal outputs
and voltage ratios but unequal percentage
impedances of 4 and 2 are operating in parallel.
Transformer A will be running over-load by .......
percent.
(a)50 (b)66
(c)33 (d)25
GO To FIRST